You are on page 1of 107

El-Zohry

SCE Endocrinology

sce.practicaldiabetes.com

Khalid Yusuf
Sohag Teaching Hospital 2017
(Egypt)
Elzohryxp@Yahoo.Com
FB: Sohag Endocrine Group
SCE Endocrinology (sce.practicaldiabetes.com), 2017

SCE Endocrinology
sce.practicaldiabetes.com
Dr. Khalid Yusuf, 2017

Q1 inappropriate. Again, there is no indication of


this in the history.
A 40-year-old man with type 2 diabetes
mellitus was referred to the clinic by his Further reading
general practitioner. He is currently on National Institute for Health and Clinical
maximum tolerated doses of metformin and Excellence. Type 2 Diabetes: newer agents.
gliclazide for his diabetes management. NICE CG87, May 2009.
National Institute for Health and Clinical
2
His body mass index is 35.4 kg/m and HbA1c is Excellence. Liraglutide for the treatment of
9.5% (80mmol/mol). He had dietary advice type 2 diabetes mellitus. NICE TA203, October
from a professional dietitian on weight 2010.
management two months ago. He engages in a
moderate level of exercise of 30 minutes a day Q2
for five days a week.
According to NICE guidance what would be the A 28-year-old woman attends the antenatal
correct next step in his management? diabetes clinic in her second pregnancy at 16
weeks of gestation. She had gestational
1- Start exenatide diabetes in her previous pregnancy and was
2- Start insulin treated with insulin. She has not attended
fasting blood glucose tests annually.
3- Continue with lifestyle changes
What should be the next step in management
4- Start sitagliptin of this woman?
5- Start pioglitazone 1- Arrange OGTT (oral glucose tolerance
test) at 24–28 weeks
Answer & Comments
2- Arrange fasting glucose
2- Start insulin 3- Arrange OGTT as soon as possible

Current NICE guidance suggests the use of GLP- 4- Start on metformin


1 mimetics only if BMI is above 35 and there 5- Start on insulin
are specific medical or psychological problems
associated with high body weight. There is no
Answer & Comments
indication of this in the given history. NICE
guidance also allows the use of 3- Arrange OGTT as soon as possible
thiazolidinedione or DPP-IV inhibitors as third
line therapy if insulin is unacceptable or

2| Dr. Khalid Yusuf (FB: Sohag Endocrine Group)


SCE Endocrinology (sce.practicaldiabetes.com), 2017

For women who have had previous gestational 5- Add short-acting insulin analogue with
diabetes, NICE guidance recommends early meals
self-monitoring of blood glucose or a two-hour
75g glucose tolerance test (OGTT) at 16–18 Answer & Comments
weeks of gestation. Screening for gestational
diabetes should not be performed using fasting 5- Add short-acting insulin analogue with
plasma glucose, random blood glucose, meals
glucose challenge test or urinalysis for glucose.
This gentleman is taking large amounts of
Further reading insulin but is not getting the correct dose due
National Institute for Health and Clinical to insulin leak, which is likely to be a reflection
Excellence. Diabetes in Pregnancy. NICE CG63, of the large volumes that he is injecting. This is
March 2008. reflected in his poor glycaemic control. GLP-1
mimetics are not licensed for use with insulin
Q3 at the current time, and the replacement of
insulin with a GLP-1 mimetic would be unwise
A 58-year-old plumber with type 2 diabetes as the doses of insulin needed are so high.
mellitus attends the diabetes clinic. His BM Adding short-acting prandial injections to basal
36.5kg/m2 and his weight has been static over insulin should reduce the volumes that he will
the past two years. His HbA1c was checked just need to inject. The addition of short-acting
prior to clinic and was 10% (86mmol/mol). He insulin to intensify glycaemic control in
maintains that he takes all his tablets and patients already on basal insulin has been
insulin regularly and is currently on the shown to be an effective strategy in the
following medications: Treating To Target in Type 2 diabetes (4-T)
study.
Metformin 1g tds
Further reading
Gliclazide 160mg bd Rury R, et al; for the 4-T Study Group. Three-year
efficacy of complex insulin regimens in type 2
Simvastatin 40mg daily diabetes. New Engl J Med 2009; 361: 1736–47.

Long-acting insulin analogue 140 units bd Q4

He complains of insulin leak following injection.


A 38-year-old man who has undergone
He has already tried changing injection sites
adrenalectomy for a phaeochromocytoma
and using long needles according to the advice
attends his first follow-up clinic visit. He has
given by the diabetes specialist nurse, but with
done some research on the internet and
no benefit.
inquires about genetic screening.
What should be the next step in managing this
Which of the following genetic mutations is
patient?
least likely to be associated with
1- Stop insulin, continue oral phaeochromocytoma?
hypoglycaemics and start exenatide
1- RET
2- Continue current regimen and accept
2- VHL
that current control is best that can be
achieved 3- SDHB
3- Add exenatide together with insulin 4- SDHD
4- Add pioglitazone 45mg 5- SDHA

3| Dr. Khalid Yusuf (FB: Sohag Endocrine Group)


SCE Endocrinology (sce.practicaldiabetes.com), 2017

Answer & Comments Q5

5- SDHA
A 48-year-old male was discovered to have a
pituitary lesion when he underwent a CT scan
It is recommended to test the following groups
of his head after trauma. A subsequent
of people who present with
enhanced MRI scan showed a 1.5cm
phaeochromocytoma for germline mutations.
hypointense area in the anterior pituitary. No
optic chiasm involvement or invasion of
1. Patients with personal or family history of
surrounding structures was noted. His serum
the following syndromes:
TSH was 3.5mU/L. T3 and T4 were within
normal limits. Serum prolactin was 590mU/L
MEN2 (c-RET gene protooncogene) ,
(45–375).
medullary thyroid Ca 90%,
phaeochromocytoma 50%, primary IGF1 was 31nmol/L (14–47). After an overnight
hyperparathyroidism 5-10% dexamethasone suppression test, his cortisol
Von Hippel Lindau syndrome (VHL gene)
was 28nmol/L. His FSH was 6IU/L (1.4–18) and
retinal angiomas (55%), central nervous
his LH was 7IU/L (3–8). Serum testosterone was
system haemangioblastoma (55%),
7nmol/L (8.4–28). He had a normal short
phaeochromocytoma (30%), renal cysts
synacthen test. His GH level was <2mU/L
(75%), renal cell carcinoma (25%),
following a 75g glucose tolerance test. His
pancreatic cysts (15%), pancreatic islet-
visual fields were normal on formal testing. He
cell tumour (3%)
feels well in himself and clinical examination
was normal. After all of the above
Neurofibromatosis type 1 (NF1)
investigations he returns to clinic and inquires
Paraganglioma syndrome type 1 (SDHD about the future management.
gene) phaeochromocytoma, Which of the following is true?
paraganglioma, gastric stromal tumours
1- There is more than a 75% chance that
Paraganglioma syndrome type 2 (SDHC the lesion will regress with time
gene) phaeochromocytoma, 2- Radiotherapy will be an essential part of
paraganglioma, gastric stromal tumours management

Paraganglioma syndrome type 3 (SDHB 3- If untreated the most likely pituitary


gene) phaeochromocytoma, hormone deficiency to develop next
paraganglioma, gastric stromal tumours, would be thyroid deficiency
renal cell Ca 4- His pituitary function is likely to return to
normal if treated with trans-sphenoidal
2. Onset before 40 surgery
3. Multiple tumours
4. Extra adrenal tumours 5- Conservative management with close
5. Malignant tumours follow up is the preferred management
option
Further reading
Erlic Z, Hartmut NPH. When should genetic testing Answer & Comments
be obtained in a patient with
phaeochromocytoma or paraganglioma? Clin 5- Conservative management with close
Endocrinol (Oxf) 2009; 70(3): 354–7. follow up is the preferred management
option

Clinically non-functioning adenomas constitute


the majority of pituitary macroadenomas.

4| Dr. Khalid Yusuf (FB: Sohag Endocrine Group)


SCE Endocrinology (sce.practicaldiabetes.com), 2017

Around 10% of tumours show spontaneous 2- Advise patient to take thyroxine on an


regression and about 50% can show empty stomach and one hour before
progression over five years. GH deficiency is taking other medication
present in about 85% and gonadal deficiency in
3- Change to a natural porcine extract of
about 75% of all patients, whereas
thyroxine
corticotroph (38%) and thyrotroph deficiencies
(32%) are present to a lesser degree. 4- Use a combination of T3 and T4
5- Arrange an MRI of her pituitary
Surgical treatment usually comprises trans-
sphenoidal resection (TSS) and is indicated in
patients in whom the tumour is in danger of Answer & Comments
encroaching the optic chiasm or who have
2- Advise patient to take thyroxine on an
visual impairment due to chiasmal
empty stomach and one hour before
involvement. TSS will improve visual function
taking other medication
in about 80%, but hypopituitarism will persist
in a large number of patients. Postoperative The average levothyroxine requirement for
radiotherapy can be used in patients with most individuals is around 1.6–1.8µg/kg.
residual tumour bulk postoperatively. A ‘wait Causes of elevated TSH levels despite being on
and see’ policy is an option in tumours which a reasonable dose of thyroxine include poor
are not in danger of reaching the optic chiasm. compliance, malabsorption and interaction
with other medication. The most common
Further reading drugs which interfere with absorption include
Dekkers OM, et al. Treatment and follow up of
iron and calcium supplements. It is suggested
clinically nonfunctioning pituitary
macroadenomas. J Clin Endocrinol Metab
that thyroxine should be taken on an empty
2008; 93: 3717–26. stomach and taking other medication should
be avoided for one hour. Drugs such as
phenytoin, carbamazepine and rifampicin
Q6 increase the metabolism of levothyroxine. In
addition, conditions such as thyroid hormone
A 59-year-old woman was referred by her GP resistance or TSH producing pituitary adenoma
because of difficulty in managing her can cause a persistently raised TSH.
hypothyroidism. She has a past history of
depression, hypertension and osteoporosis. Use of natural thyroid extracts is currently not
She initially presented with tiredness and endorsed by the British Thyroid Society.
weight gain and at presentation her initial TSH Combination treatment with T3 and T4 has not
was 61mU/L (0.35–5.5), and free T4 was been shown to confer any additional benefit in
6pmol/L (11.5–22.7) with a TPO antibody level most trials.
of >1000IU/ml. Pre-clinic blood tests reveal
TSH of 32 and free T4 of 7, despite being on Further reading
225µg of levothyroxine. She is also taking Morris JC. How do you approach the problem of
citalopram, calcium supplements and ramipril. TSH elevation in a patient on high-dose thyroid
She continues to complain of tiredness and hormone replacement? Clin Endocrinol 2009;
lethargy, and claims full compliance with her 70: 671–3.
medication. Armour Thyroid (USP) and combined thyroxine/
tri-iodothyronine as Thyroid Hormone
Replacement. A Statement from the British
Which of the following would be the most Thyroid Association Executive Committee,
appropriate next step in management? November 2007.

1- Arrange supervised administration of


weekly oral thyroxine

5| Dr. Khalid Yusuf (FB: Sohag Endocrine Group)


SCE Endocrinology (sce.practicaldiabetes.com), 2017

Q7 >6mmol/L, bicarbonate level <5mmol/L,


venous/arterial pH <7.1, hypokalaemia on
An 18-year-old girl with type 1 diabetes is admission (<3.5mmol/L), GCS <12 or abnormal
admitted feeling unwell and with recurrent AVPU scale, oxygen saturation <92% on air,
vomiting. She takes insulin glargine 20 units at systolic BP <90mmHg, pulse >100 or <60bpm
night and 8 units of insulin aspart with meals. and an anion gap >16.
She has just started university and had been
drinking large amounts of alcohol prior to Increased fat metabolism in DKA causes
admission. At the time of admission her serum release of ketones (acetone, acetoacetate and
glucose was 44mmol/L and her pH was 6.88. β-hydroxybutyrate). Suppression of
Her urine had +4 of ketones. Her GCS was ketonaemia is a vital part of DKA management.
15/15. Urine ketone sticks do not measure β-hydroxy-
butyrate and therefore measurement of blood
Which of the following is true regarding her ketones represents best practice in DKA.
management? Intravenous insulin and fluids should be
1- Insulin glargine should be omitted until continued until venous pH is >7.3 and blood
normal dietary intake is established ketones <0.3mmol/L.

2- Boluses of 8.4% sodium bicarbonate Further reading


should be administered until her pH is Joint British Diabetes Societies Inpatient Care
above 7 Group. The Management of Diabetic
Ketoacidosis in Adults. March 2010.
3- Checking venous pH would be
misleading and should not be used Kitabchi AE, et al. Hyperglycaemic crises in adult
patients with diabetes. Diabetes Care
4- She does not need to be managed in 2009;32:1335–43.
HDU as her GCS is 15
5- She should be continued on intravenous Q8
insulin and fluids until the plasma levels
of β-hydroxybutyrate are brought back A 51-year-old obese gentleman with type 2
to within normal limits diabetes attends clinic. He has hypertension,
laser treated retinopathy and
Answer & Comments microalbuminuria. He is taking aspirin 75mg
od, metformin 1g tds, ramipril 10mg od,
5- She should be continued on intravenous bisoprolol 5mg od and simvastatin 40mg od
insulin and fluids until the plasma levels and fenofibrate 267mg od. His most recent
of β-hydroxybutyrate are brought back HbA1c is 6.7%. His lipid profile is as follows:
to within normal limits
Total cholesterol 4.0mmol/L
Current national guidance suggests that long-
acting insulin analogues should be continued LDL 1.9mmol/L
while being treated with intravenous insulin.
HDL 1.0mmol/L
Sodium bicarbonate treatment is not routinely
advocated in DKA in the UK national guidelines. TG 5.7mmol/L
Which would be the most appropriate
The difference between arterial and venous pH
treatment modification with regard to his
is 0.02; therefore arterial sampling is not
necessary to monitor the acid-base status. serum lipids?
1- Change simvastatin to atorvastatin
Indications to manage the patient in a Level 2 80mg
setting include presence of blood ketones

6| Dr. Khalid Yusuf (FB: Sohag Endocrine Group)


SCE Endocrinology (sce.practicaldiabetes.com), 2017

2- No changes needed - continue on 2- 2.0%


current medication
3- 6.0%
3- Advise lifestyle changes and refer to
4- 8.0%
dietitian
5- 10.0%
4- Add modified release nicotinic acid
5- Add Omacor Answer & Comments

Answer & Comments 3- 6.0%

3- Advise lifestyle changes and refer to The overall lifetime risk in a caucasian
dietitian developing type 1 diabetes is 0.4%; however,
this rises to:
NICE guidance states that highly concentrated
omega-3 fish oils (such as Omacor) are o 4% if the mother has it and gives birth
indicated if there are high serum triglycerides before she is 25 and 1% if she gives
(>4.5mmol/L) and if: birth after she is 25
o 5–6% if the father has it
o Underlying causes such as poor o 10–25% if both parents have it
glycaemic control are excluded
o Lifestyle measures have proved Further reading
ineffective Genetics of Diabetes – American Diabetes
o Fibrate therapy has proved ineffective Association. [accessed 14/11/2010].
The Genetic Landscape of Diabetes. The National
In this gentleman, his glycaemic control is Institute of Health. [accessed 14/11/2010].
clearly good and he is taking fibrate therapy.
Concentrated omega-3 fish oil (Omacor) is a
management option but only after it is ensured Child
that lifestyle measures have been optimised. Type 1 Father 1 in 17 (5–6%)
Mother < 25 yrs 1 in 25 (4%)
Further reading Mother > 25 yrs 1 in 100
National Collaborating Centre for Chronic Father + between 1 in
Conditions. Type 2 diabetes: national clinical mother 10 and 1 in 4
guideline for management in primary and
Type 1 Diabetes: Your Child's Risk
secondary care (update). London: Royal
College of Physicians, 2008. NICE CG66.
In general, if you are a man with type 1
diabetes, the odds of your child developing
Q9 diabetes are 1 in 17.
A 28-year-old man with newly diagnosed type
1 diabetes attends clinic. He is accompanied by If you are a woman with type 1 diabetes and
his female partner who does not have your child was born before you were 25, your
child's risk is 1 in 25; if your child was born after
diabetes. They are planning to start a family
you turned 25, your child's risk is 1 in 100.
and want to know the risk of their offspring
having type 1 diabetes in the future.
Your child's risk is doubled if you developed
Considering genetics in type 1 diabetes, what is diabetes before age 11. If both you and your
the overall lifetime risk of a baby born to this partner have type 1 diabetes, the risk is
couple having T1DM? between 1 in 10 and 1 in 4.

1- 0.4%

7|
SCE Endocrinology (sce.practicaldiabetes.com), 2017

There is an exception to these numbers. About have type 2 diabetes, your child's risk is about
1 in every 7 people with type 1 diabetes has a 1 in 2.
condition called type 2 polyglandular
autoimmune syndrome. In addition to having People with certain rare types of type 2
diabetes, these people also have thyroid diabetes have different risks. If you have the
disease and a poorly working adrenal gland. rare form called maturity-onset diabetes of the
Some also have other immune system young (MODY), your child has almost a 1-in-2
disorders. If you have this syndrome, your chance of getting it, too.
child's risk of getting the syndrome — including
type 1 diabetes — is 1 in 2. - See more at:
http://www.diabetes.org/diabetes-
Researchers are learning how to predict a basics/genetics-of-
person's odds of getting diabetes. For example, diabetes.html#sthash.f3Rm9DfQ.dpuf
most whites with type 1 diabetes have genes
called HLA-DR3 or HLA-DR4. If you and your
child are white and share these genes, your Q 10
child's risk is higher. (Suspect genes in other
ethnic groups are less well studied. The HLA- A 22-year-old girl was referred by the GP when
DR7 gene may put African Americans at risk, she was found to have TSH of 8mU/L (0.35–
and the HLA-DR9 gene may put Japanese at 5.5). Her free T4 was 13.6pmol/L (11.5–22.7).
risk.) Her serum TPO antibodies were strongly
positive. A repeat TSH level a few months
Other tests can also make your child's risk afterwards remained elevated at 8.2. She feels
clearer. A special test that tells how the body well in herself. Her mother is currently on
responds to glucose can tell which school-aged thyroxine treatment.
children are most at risk. Which of the following is false?

Another more expensive test can be done for 1- She needs to start thyroxine treatment
children who have siblings with type 1 2- She is at risk of progressing to overt
diabetes. This test measures antibodies to hypothyroidism compared to the
insulin, to islet cells in the pancreas, or to an general population
enzyme called glutamic acid decarboxylase.
High levels can indicate that a child has a higher 3- She requires annual follow up
risk of developing type 1 diabetes. 4- If she wishes to start a family soon, she
needs treatment with thyroxine
Type 2 Diabetes: Your Child's Risk
5- Her TSH may normalise in the future
Type 2 diabetes runs in families. In part, this
tendency is due to children learning bad habits Answer & Comments
— eating a poor diet, not exercising — from
1- She needs to start thyroxine treatment
their parents. But there is also a genetic basis.
This patient has subclinical hypothyroidism.
In general, if you have type 2 diabetes, the risk There is no clear evidence to support use of
of your child getting diabetes is 1 in 7 if you thyroxine in patients who are asymptomatic
were diagnosed before age 50 and 1 in 13 if you with a TSH of <10mU/L. In women, the annual
were diagnosed after age 50. risk of spontaneous overt hypothyroidism is 4%
in those who have both high serum TSH and
Some scientists believe that a child's risk is TPO antibody concentrations. If the serum TSH
greater when the parent with type 2 diabetes is mildly increased between 4 and 10mU/L, and
is the mother. If both you and your partner the patient is TPO antibody positive, annual

8| Dr. Khalid Yusuf (FB: Sohag Endocrine Group)


SCE Endocrinology (sce.practicaldiabetes.com), 2017

monitoring of TSH is recommended. If the TSH Free T3 – 8pmol/L (normal range 3.7–6pmol/L)
is >10 or if the patient is symptomatic,
treatment with thyroxine is indicated in Thyroid stimulating hormone – <0.1mU/L
subclinical hypothyroidism. (normal range 0.35–5.5mU/L)

Spontaneous recovery has been described in Thyroid peroxidase antibody test – 1.0IU/ml
subjects with a mildly raised serum TSH. In one (normal range <2.0IU/ml)
study, 37% of patients normalised their serum
TSH levels over a mean follow-up time of 32 Radio-iodine uptake scan showed decreased
months. uptake
What would be the best management for this
Early correction of maternal
patient?
hypothyroxinaemia is recommended, aiming
to maintain the serum TSH in the lower half of 1- Stop amiodarone
the reference range prior to conception if 2- Watch and wait
possible.
3- Start prednisolone
Further reading 4- Start carbimazole
Vanderpump MPJ. How should we manage
patients with mildly increased serum 5- Refer for radio-active iodine treatment
thyrotrophin concentrations? Clin Endocrinol
2010;72:436–40. Answer & Comments

Q 11 3- Start prednisolone

Amiodarone has high concentration of iodine


A 65-year-old Asian man with past history of
(39% by weight). Abnormalities in thyroid
atrial flutter and schizophrenia is referred to
function occur in 50% of patients on
the endocrine clinic. He has no symptoms
amiodarone. Thyrotoxicosis resulting from the
except for 5kg weight loss over the last 2
high content of iodine in amiodarone is
months. He is currently on warfarin,
referred to as amiodarone induced
simvastatin 40mg od, amiodarone 200mg daily
thyrotoxicosis type 1. Features of this condition
and clozapine 200mg daily. His blood tests
include the presence of a goitre, positive
show:
thyroid antibodies, normal inflammatory
Hb – 13.5g/dl markers, increased or normal flow on thyroid
colour flow Doppler, and normal or increased
Na – 132mmol/L uptake of iodine in isotope scan. Thyrotoxicosis
due to direct toxic effect of amiodarone
WBC – 12.5x109/L (leading to destructive thyroiditis) is known as
amiodarone induced thyrotoxicosis type 2.
K – 3.8mmol/L Features of this condition include high
inflammatory markers, negative thyroid
ESR – 60mm/hr antibody test, decreased iodine uptake in
isotope scan, and decreased flow seen on
Urea – 6.5mmol/L thyroid colour flow Doppler.

Creatinine – 82µmol/L This gentleman has got amiodarone induced


thyrotoxicosis (AIT) type 2. High dose
Free T4 – 30.2pmol/L (normal range 11.5– glucocorticoid therapy is the treatment of
22.7pmol/L) choice for AIT type 2 due to anti-inflammatory
and membrane stabilising effects.

9| Dr. Khalid Yusuf (FB: Sohag Endocrine Group)


SCE Endocrinology (sce.practicaldiabetes.com), 2017

Further reading Further reading


Tsang W, Houlden RL. Amiodarone induced Berruti A, et al. Adjuvant therapy in patients with
thyrotoxicosis: a review. Can J Cardiol adrenocortical carcinoma: a position of an
2009;25(7):421–4. international panel. J Clin Oncol
2010;28(23):e401-2; author reply e403.
Q 12 Allolio B, Fassnacht M. Adrenocortical carcinoma:
clinical update. J Clin Endocrinol Metab
2006;91:2027–37.
A 56-year-old lady presented with excessive
hair growth, weight gain and easy bruising. She
was found to have the following biochemical Q 13
and radiological abnormalities:
A 58-year-old heavy goods vehicle driver
Serum testosterone – 5.6nmol/L (normal range (LGV/PCV driver) who has had type 2 diabetes
0.5–2.6nmol/L) for 21 years attends the diabetes clinic. He is
also known to have well-controlled mild
Dehydroepiandrosterone sulphate (DHEAS) – congestive cardiac failure. His oral
20µmol/L (normal range 1.9–9.4µmol/L) hypoglycaemic medication consists of
metformin 1g tds and gliclazide 160mg bd and
Overnight dexamethasone suppression test – he has already informed the DVLA previously of
125nmol/L (normal <50nmol/L) this. His HbA1c is 9.2% (77mmol/mol) and his
BMI is 32kg/m2. He has decided to retire at the
CT abdomen – 12.5cm tumour in right adrenal age of 60 and is unwilling to start on insulin
gland until after this age due to the potential effect
on his occupation. After discussion with you, he
She underwent surgical resection with
agrees to start a DPP-IV inhibitor (gliptin) in
complete removal of the tumour with regional
conjunction with his existing tablets.
lymph node clearance. Histology confirmed the
diagnosis of adrenocortical carcinoma with Which of the following statements is correct?
regional lymph node invasion
1- He does not need to inform the DVLA as
Which of the following would increase her he has previously informed them
survival? 2- He does not need to inform DVLA as he
1- Ketaconazole was started on an oral medication with a
low/negligible risk of hypoglycaemia
2- Mitotane
3- He should inform the DVLA again in view
3- Metyrapone of his medication change
4- Etomidate 4- He cannot hold an LGV/PCV licence
5- None of the above while on a DPP-IV inhibitor
5- He should inform the DVLA if he had
Answer & Comments been commenced on a GLP-1 mimetic,
but not a DPP-IV inhibitor
2- Mitotane

Mitotane is the only adrenal specific agent Answer & Comments


available for the treatment of adrenocortical
3- He should inform the DVLA again in view
carcinoma. This lady is at high risk of
of his medication change
recurrence despite complete tumour resection
(tumour size >12cm). Adjuvant chemotherapy Group 2 (LGV or PCV) drivers are required to
with mitotane has shown higher survival rates notify the DVLA if they have diabetes treated
in two retrospective studies. with tablets. If they are then started on GLP-1

10 | Dr. Khalid Yusuf (FB: Sohag Endocrine Group)


SCE Endocrinology (sce.practicaldiabetes.com), 2017

mimetics (exenatide or liraglutide) or a DPP-IV amitriptyline and gabapentin, increasing


inhibitor (gliptin), they are only required to both to maximal tolerated doses
notify the DVLA if this is in combination with a
5- Commence strong opiate (such as
sulphonylurea. There is a significantly
morphine or oxycodone) and discuss
increased risk of hypoglycaemia from DPP-IV
referral to the specialist pain service
inhibitors and GLP-1 mimetics when used in
combination with sulphonylureas. Therefore,
in this combination, these are felt to be a Answer & Comments
potentially higher risk treatment for drivers
2- Commence on tramadol 50mg qds and
holding Group 2 licences and as such requiring
discuss referral to a specialist pain
individual assessment.
service
Further reading According to recent NICE guidelines, the first
Medical Rules for Drivers. [accessed 11.02.2011].
and second steps in the management of
DVLA. Information for Health Professionals. neuropathic pain are as below:
[accessed 11.02.2011].
o First line:
Q 14 o Offer oral duloxetine
o Offer oral amitriptyline if duloxetine is
A 65-year-old gentleman with a 22-year history contraindicated
of type 2 diabetes complicated by painful o Second line:
peripheral neuropathy is seen in the diabetes o Offer treatment with another drug
clinic. His HbA1c is 7.6% (60mmol/mol) and his instead of or in combination with the
is BMI is 26.7kg/m2. He is currently taking original drug, after informed discussion
metformin 1g bd, gliclazide 80mg bd, with the person
duloxetine 60mg bd and pregabalin 300mg bd. o If first line treatment was with
He is tolerating this combination of tablets well duloxetine, switch to amitriptyline or
and they have controlled his symptoms until pregabalin, or combine with
recently. However, when he attended clinic, he pregabalin
complained of a recurrence of neuropathic o If first line treatment was with
pain in both legs which is significantly affecting amitriptyline, switch to or combine
his sleeping. with pregabalin
What should be the next step in the
This gentleman requires third line treatment
management of this patient? for his neuropathic pain. Therefore he ought to
1- Commence on amitriptyline in be considered for referral to specialist pain
onjunction with his existing medication services. While waiting for assessment at a
at a starting dose of 10mg daily and specialist pain clinic, he can be given tramadol
increase to maximum of 75mg daily if he instead or in combination with second line
tolerates this treatment. He should not be started on long-
term strong opiates until after his specialist
2- Commence on tramadol 50mg qds and
pain clinic assessment.
discuss referral to a specialist pain
service
Further reading
3- Improve glycaemic control and continue National Institute for Health and Clinical
current therapy for another 6 months Excellence. Neuropathic pain – the
and reassess pharmacological management of neuropathic
pain in non-specialist settings (CG96). NICE,
4- Discontinue both duloxetine and March 2010.
pregabalin and commence on

11 | Dr. Khalid Yusuf (FB: Sohag Endocrine Group)


SCE Endocrinology (sce.practicaldiabetes.com), 2017

Q 15 Tuomilehto J, et al. Prevention of type 2 diabetes


mellitus by changes in lifestyle among subjects
with impaired glucose tolerance. N Engl J Med
A 63-year-old lady with type 1 diabetes attends
2001;344:1343–50.
clinic. At the end of the consultation she
mentions to you that her husband, who is the
same age, has a family history of type 2 Q 16
diabetes mellitus and recently had a fasting
A 45-year-old lady was referred to the
glucose level of 5.9mmol/L. He is a manager by
endocrine clinic with deepening of her voice,
profession and finds his job stressful. He has
erratic eating habits and eats a large amount of coarsening of her physical features, and an
junk food. He has been treated with increase in her shoe size. A glucose tolerance
bendroflumethiazide 2.5mg daily for the last 3 test showed failure of suppression of growth
years for hypertension. She understands that hormone and an MRI of her pituitary gland
people develop type 2 diabetes mellitus due to confirmed the presence of a pituitary mass.
genetic and environmental factors and wants She was diagnosed to have acromegaly.
to know what is the single greatest risk factor
for her husband to develop diabetes.
In the treatment of acromegaly, which of the
Which of the following answers is correct? following does not cause tumour shrinkage?
1- High fasting glucose of 5.9mmol/L 1- Lanreotide
2- Lack of fruit and vegetables in the diet 2- Cabergoline
3- Family history of diabetes 3- Octreotide
4- Bendroflumethiazide treatment for 4- Pegvisomant
hypertension
5- Radiotherapy
5- Sedentary lifestyle

Answer & Comments


Answer & Comments
4- Pegvisomant
5- Sedentary lifestyle
The different treatment modalities have
Obesity and large waist circumference are the different effects on tumour mass. Surgery
single greatest risk factors for pre-diabetes and achieves immediate and substantial debulking,
type 2 diabetes. Men are at high risk if they but radiotherapy can take years to reduce
have a waist circumference of 94–102cm (37– tumour mass. Dopamine agonist therapy
40ins). They are at very high risk if it is more (cabergoline or bromocriptine) only reduces
than 102cm. Women are at high risk if they tumour mass in approximately 5% of patients,
have a waist circumference of 80–88cm (31.5– while somatostatin receptor analogues
35ins). They are at very high risk if it is more (octreotide or lanreotide) reduce tumour mass
than 88cm. A sedentary lifestyle not only by over 20% (on average approximately by
contributes to obesity and high waist about 50%) in 75% of patients. However,
circumference but is also an independent risk therapy with the growth hormone receptor
factor for the development of type 2 diabetes. antagonist, pegvisomant, lowers IGF1 but does
not induce tumour shrinkage, and in a small
Further reading proportion of cases may cause an increase in
Hu G, et al. Physical activity, body mass index, and
tumour growth.
risk of type 2 diabetes in patients with normal
or impaired glucose regulation. Arch Intern
Med 2004;164:892–6. Further reading

12 | Dr. Khalid Yusuf (FB: Sohag Endocrine Group)


SCE Endocrinology (sce.practicaldiabetes.com), 2017

Melmed S, et al. Guidelines for acromegaly antibodies, pre-existing autoimmune disease,


management: an update. J Cl in Endocrinol vitiligo or a strong family history of thyroid
Metab 2009;94:1509—17. disease, the decision as to whether this would
be autoimmune thyrotoxicosis or hyperemesis
Q 17 would be more difficult – but there is no
mention of any of these factors in the history.
A 26-year-old primagravida at 12 weeks’ It could be argued that, as this lady is
gestation was referred to the endocrine clinic symptomatic, it might be best to control
with vomiting, tremors and palpitations. symptoms with propranolol. However,
Uterine ultrasound done 2 weeks earlier although the use of propranolol during
confirmed a viable twin pregnancy. Thyroid pregnancy has not been conclusively
function tests (TFTs) had been arranged by her associated with an increased risk of structural
general practitioner and showed: Free fetal malformations to date, some recent
thyroxine (T4) – 35.2pmol/L (normal range studies have suggested a possible increased
11.5–22.7pmol/L) Thyroid stimulating risk of congenital heart defects associated with
hormone (TSH) – 0.1mU/L (normal range 0.35– antihypertensive therapy including beta
5.5mU/L) On examination, she was anxious and adrenoceptor blocking drugs; nevertheless, it is
tachycardic. There was a minimally enlarged unclear whether this risk is due to the
goitre. underlying maternal condition or to the
medications. Its use is therefore not without a
What would be the best next step in small risk, and best treatment would be to
management of this lady? observe and repeat TFTs in a couple of weeks.
1- Observe and recheck TFTs two weeks
later Further reading
Hershmann J. Physiological and pathological
2- Start propylthiouracil aspects of the effect of human chorionic
gonadotropin on the thyroid. Best Pract Res Cl
3- Start carbimazole
in Endocrinol Metab 2004;18:249–65.
4- Start propranolol and repeat TFTs 2 National electronic Library for Medicines. Use of
weeks later propranolol in pregnancy. [accessed
5- Start propylthiouracil and refer for 11.02.2011].
thyroidectomy in the 2nd trimester
Q 18
Answer & Comments
A 32-year-old woman was referred to the
1- Observe and recheck TFTs two weeks endocrinology clinic with intermittent
later symptoms of sweating, shaking and dizziness.
These mostly occurred when she had not
This lady has got hyperemesis gravidarum. eaten. Her husband noted that her symptoms
Women with hyperemesis gravidarum often are relieved by sweets. During one episode the
have high levels of human chorionic paramedics were called and he recalls that they
gonadotropin (hCG) which has structural mentioned that her blood glucose was
similarity to thyroid stimulating hormone. 2.8mmol/L. Her U&Es and LFTs were normal.
Stimulation of the thyroid gland by hCG causes The 30min cortisol level after a short syncathen
a transient thyrotoxicosis type effect in some test was 678nmol/L. She undergoes a 72-hour
pregnant women. In the vast majority of such supervised fast. After 36 hours she complains
patients, there will be spontaneous remission of feeling shaky and sweaty. Blood glucose was
of the increased thyroid function when the noted to be 2.1mmol/L. Results from
hyperemesis resolves (usually after 12 weeks’ investigations sent out at that time are as
gestation). If this lady had positive thyroid follows: Glucose – 2.0mmol/L Plasma insulin-

13 | Dr. Khalid Yusuf (FB: Sohag Endocrine Group)


SCE Endocrinology (sce.practicaldiabetes.com), 2017

250pmol/L (range 21.5–115) C-peptide – plasma insulin concentration is inappropriately


0.02pmol/L (range 0.17–0.5) high with a relatively low C-peptide suggesting
an exogenous source of insulin. In such cases, a
The next step should be:
family history should be sought as it may clarify
1- Inquire about a family history of diabetes the source of the exogenous insulin (the
mellitus patient may have obtained the insulin from a
family member). If these symptoms were
2- Refer to a dietitian for advice regarding
caused by sulphonylurea ingestion, then one
low glycaemic index diet
would expect an increased C-peptide level.
3- MRI of the abdomen
4- Arrange urine sulphonylurea screen Further reading
Cryer P, et al. Evaluation and management of
5- Arrange a prolonged glucose tolerance adult hypoglycemic disorders: an Endocrine
test Society Clinical Practice Guideline. J Cl in
Endocrinol Metab 2009;94:709–28.
Answer & Comments
Q 19
1- Inquire about a family history of iabetes
mellitus A 28-year-old woman of South Asian origin was
diagnosed with gestational diabetes mellitus at
Causes of hypoglycaemia in patients in
28 weeks gestation. She was screened because
apparent good health include endogenous
of her ethnicity and high BMI (which was
hyperinsulinism and accidental or surreptitious
34kg/m2). Initially, she was provided with diet
hypoglycaemia. Endogenous hypoglycaemia
and lifestyle advice together with a blood
can be caused by an insulinoma, antibodies to
glucose meter and was advised to monitor her
insulin or insulin receptors and nesidioblastosis
blood sugars. She returns to clinic 2 weeks
due to gastric bypass surgery or non-
later. Her blood glucose monitoring is very
insulinoma pancreatogenous hypoglycaemia.
intermittent but 1-hour post-meal readings
The key pathophysiological feature of
average between 8–9mmol/L, and her pre-
endogenous hyperinsulinism is the failure of
meal readings are between 5–6mmol/L. Her
insulin secretion to fall to very low rates as
fetal abdominal circumference was around the
plasma glucose concentrations fall to
95th centile for the gestational age. She was
hypoglycaemic level. Plasma insulin, C-peptide,
commenced on metformin but had to
and proinsulin concentrations are
discontinue due to severe gastrointestinal side
inappropriately high in the setting of low
effects. Although she was previously
fasting plasma glucose concentrations. Critical
counselled that she may require insulin
diagnostic findings are plasma insulin
treatment in the future, she tells you that she
concentrations of at least 18pmol/L, plasma C-
is needle-phobic and is absolutely unwilling to
peptide concentrations of at least 0.2nmol/L
consider insulin treatment.
and plasma pro-insulin concentrations of at
least 5.0pmol/L when the fasting plasma Which of the management options below
glucose is 3.0mmol/L. In patients who are ill or would be most suitable for her next?
on medication, consider drugs such as insulin
1- Continue to monitor with no change of
or insulin secretagogues, hepatic, renal or
treatment
cardiac failure and cortisol deficiency. In
addition, alcohol and numerous drugs, 2- Offer sitagliptin
including quinine and indomethacin, have
3- Give advice to check her blood glucose
been implicated. A prolonged glucose
level 2 hours after meals
tolerance test or, alternatively, a mixed meal
test is indicated when postprandial
hypoglycaemia is suspected. In this case, the

14 | Dr. Khalid Yusuf (FB: Sohag Endocrine Group)


SCE Endocrinology (sce.practicaldiabetes.com), 2017

4- Advise her only to check pre-meal foot over the past 2 days. Physical examination
readings as they are more predictive of a revealed a swollen, warm and erythematous
better outcome right mid-foot with no signs of ulceration. The
temperature difference between right and left
5- Offer glibenclamide
feet was 2.6°C. A plain radiograph of her right
foot was reported as normal. Her CRP was
Answer & Comments 16mg/L (NR 0–10mg/L).
5- Offer glibenclamide What is the next best step in the management
of this patient?
The specified glycaemic targets for pregnancy
are 3.5–5.9mmol/L pre-meal and <7.8mmol/L 1- Immobilisation in a total contact plaster
1 hour post-meal. NICE guidance states that in cast
gestational diabetes mellitus hypoglycaemic 2- Administration of intravenous
treatment should be considered if these pamidronate 90mg and review patient in
targets cannot be achieved within 1–2 weeks clinic in 1 week
with diet and exercise or if fetal ultrasound
3- Commence antibiotic therapy
scan shows incipient macrosomia (fetal
abdominal circumference >70th centile at 4- Arrange MRI scanning of the foot
diagonosis). The summary of product
5- Urgently refer to orthopaedic surgeons
characteristics (SPC) for metformin states that
a limited amount of data from its use in
pregnant women does not indicate an Answer & Comments
increased risk of congenital abnormalities, and
1- Immobilisation in a total contact plaster
there is evidence for its use in gestational
cast
diabetes. Although the SPC for glibenclamide
does not comment on its possible use in This patient has Charcot neuro-
pregnancy, animal studies do not indicate osteoarthropathy (CN). Predisposing factors
harmful effects with respect to pregnancy, for CN include somatic and autonomic
embryonic or fetal development, parturition or neuropathy, renal impairment and osteopenia.
postnatal development. NICE guidance The most common presentation is pain and
suggests that informed consent should be discomfort of the foot, commonly mid-foot.
obtained and glibenclamide use in pregnancy Differential diagnoses include cellulitis, gout
should be documented. Sitagliptin has no and deep vein thrombosis. The acute phase of
safety data in pregnancy and should not be CN is characterised by unilateral erythema and
used. oedema. The foot is at least 2°C hotter than the
contralateral foot. X-rays in the early acute
Further reading phase can be normal. MRI abnormalities at this
National Institute for Health and Clinical
early stage of CN include sub-chondral bone
Excellence. Diabetes in Pregnancy:
management of diabetes and its complications
marrow oedema with or without
from preconception to the postnatal period microfractures, but an MRI is not always
(CG63). 2008. needed for diagnosis. The primary treatment is
offloading by immobilisation in a plaster cast
until there is no longer evidence on X-ray of
Q 20
continuing bone destruction, and the foot
temperature is within 2°C of the contralateral
A 61-year-old woman with type 2 diabetes, foot. An alternative is a prefabricated walking
hypertension, peripheral neuropathy and cast, such as the Aircast. A single infusion of
chronic renal impairment was referred pamidronate 90mg has been shown to cause a
urgently to the diabetic foot clinic. She had significant reduction in markers of bone
developed pain and swelling in the right mid- turnover; however, its efficacy in a randomised

15 | Dr. Khalid Yusuf (FB: Sohag Endocrine Group)


SCE Endocrinology (sce.practicaldiabetes.com), 2017

controlled trial has not yet been demonstrated Ryan EH, et al. Diabetic macular oedema
and immobilisation is crucial in the associated with glitazone use. Retina
management of acute CN. Surgery has no role 2006;26:562–70.
in the management of acute CN.
Q 22
Further reading
Petrova NL, Edmonds ME. Charcot neuro- A 50-year-old man who has recently moved to
osteoarthropathy – current standards. the area is referred by his GP to the endocrine
Diabetes Metab Res Rev 2008;24(Suppl 1):
clinic for continuing follow up of his thyroid
S58–S61.
problems. He underwent a total thyroidectomy
6 years ago for a follicular neoplasm of the
Q 21 thyroid. Following this, he had radioiodine
ablation treatment. Since then, he has been
A 52-year-old man with a 7-year history of type free of any evidence of recurrence and remains
2 diabetes is seen in the diabetes clinic. He has in good health. He is currently taking 200µg of
hypertension, ischaemic heart disease and thyroxine, and is not on any other medication.
renal impairment (CKD stage 3). His oral His serum TSH is 0.09mU/L (0.3–5) and free T4
hypoglycaemic treatment consists of is 26pmol/L (9–25).
metformin 1g bd and gliclazide 160mg bd and
Which of the following is true regarding his
a recent HbA1c was 8.7% (72mmol/mol). He
management?
works as a telecoms engineer, which often
involves working from heights, and he is 1- He should reduce the dose of hyroxine to
unwilling to start on insulin at this time due to 175µg od
the risk of hypoglycaemia. The possibility of
2- He should increase the dose of thyroxine
starting pioglitazone is discussed with him;
to 225µg od
however, the patient expresses a degree of
concern that he has read about some potential 3- A rising serum thyroid peroxidase level
eye problems which may occur with indicates a tumour recurrence
pioglitazone.
4- I-131 scanning has limited value as a
What can pioglitazone cause? first-line investigation in detecting a
recurrence in his case
1- Pre-proliferative diabetic retinopathy
5- He can be safely discharged from follow
2- Proliferative retinopathy
up with no further monitoring after a
3- Retinal haemorrhage further 4 years if he remains disease free
4- Background diabetic retinopathy
Answer & Comments
5- Maculopathy
4- I-131 scanning has limited value as a
Answer & Comments first-line investigation in detecting a
recurrence in his case
5- Maculopathy
This patient has a previous history of follicular
Fluid retention can occur in 5–15% of patients thyroid cancer. Follow up should be lifelong,
commenced on thiazolidinediones. Rarely, in with suppression of serum TSH level
some of these patients, macular oedema can (<0.1mU/L) being one of the main components
occur. Cessation of medication appears to of treatment in high-risk cases. Surveillance for
result in rapid resolution of both peripheral recurrence of disease is essential and is based
and macular oedema. on annual clinical examination, measurement
of serum thyroglobulin and TSH. Detectable
Further reading

16 | Dr. Khalid Yusuf (FB: Sohag Endocrine Group)


SCE Endocrinology (sce.practicaldiabetes.com), 2017

serum thyroglobulin is highly suggestive of 4- Urine sodium excretion may be low


thyroid remnant, residual or recurrent tumour,
5- Urine cortisol–cortisone ratio is likely to
and a progressively increasing thyroglobulin
be raised
while on suppressive thyroxine therapy is
highly suggestive of tumour recurrence or
progression. After total thyroidectomy and Answer & Comments
postoperative I-131 ablation, diagnostic whole-
4- Urine sodium excretion may be low
body scanning has a relatively low sensitivity in
detecting residual or recurrent disease Although ACE inhibitors can cause false
compared with measurement of serum negative results on screening test for primary
thyroglobulin. hyperaldosteronism, stopping the drug is not
practical in many cases and may not always be
Further reading necessary. Beta-blockers are perhaps the most
British Thyroid Association & Royal College of
interfering agents in this context. A low normal
Physicians. Guidelines for the management of
thyroid cancer, 2nd edn. Report of the Thyroid
aldosterone–renin ratio, in this case, should
Cancer Guidelines Update Group. London: prompt further investigations into genetic
2007. causes of apparent mineralocorticoid excess
and questioning about possible large intake of
liquorice. Liddle’s syndrome is a valid
Q 23
differential diagnosis and can present in adult
life in this way. A low sodium and high
A 59-year-old woman presented to her GP with potassium excretion are expected in this
fatigue and muscle weakness. She was found to condition. A cortisol–cortisone ratio can be
have raised blood pressure of 200/110mmHg, used as a surrogate to diagnose 11-beta-
but no other findings on general or systemic hydroxysteroid dehydrogenase deficiency,
examination. She had had normal vaginal which can also give rise to a similar clinical
deliveries in the past with no history of pre- picture (glucocorticoid remediable
eclampsia. Lisinopril (5mg od) was commenced hypertension). However, patients present
by the GP but the blood pressure failed to much earlier and often have associated adrenal
respond adequately. Subsequent biochemistry hyperplasia. 17-hydroxylase deficiency
revealed serum potassium 3.0mmol/L (3.5–5), presents much earlier, with sexual infantilism
Na 141mmol/L (135–145), and creatinine and hypertension. There is nothing suggestive
74µmol/L (20–120). She was referred to of this in the patient’s past medical and
secondary care with suspected Conn’s obstetric history. Renal artery stenosis is an
syndrome. She did not have any features of important differential diagnosis and must be
cortisol excess and her aldosterone–renin ratio considered here too.
was found to be 7.1ng/mU (0–25) [renin:
ambulant 9mU/L; aldosterone: ambulant Further reading
64ng/L]. MRI scanning showed anatomically Funder JW, et al. Case detection, diagnosis, and
normal adrenal glands. treatment of patients with primary
Which of the following statements is most aldosteronism: an Endocrine Society clinical
practice guideline. J Clin Endocrinol Metab
valid? 2008;93:3266–81.
1- Being on an ACE inhibitor invalidates the
aldosterone–renin ratio Q 24
2- Adrenal venous sampling should be the
next step A 65-year-old man is on testosterone
supplementation and attends the endocrine
3- 17-hydroxylase deficiency should be
clinic for a follow-up visit. He was found to have
considered as a differential diagnosis
primary hypogonadism of uncertain aetiology

17 | Dr. Khalid Yusuf (FB: Sohag Endocrine Group)


SCE Endocrinology (sce.practicaldiabetes.com), 2017

when he presented with a low libido 3 years Q 25


ago. He has been on testosterone enanthate
250mg intramuscular injections every 3 weeks A 15-year-old girl presents with hirsutism and
and had a reasonable response after acne. She has also had irregular periods over
commencing this. At this consultation he the past 6 months. Physical examination shows
complains of continued tiredness, sweating, evidence of excess hair growth on her chin,
erectile dysfunction and a low libido. His BMI is arms and chest. She has acanthosis nigricans in
30kg/m2, and other than this physical both axillae and on her neck. Her BMI is
examination was unremarkable. His blood 28kg/m2. A glucose tolerance test shows a 2-
results are: Testosterone 7nmol/L (11–36); PSA hour plasma glucose of 8.9mmol/l and pelvic
0.55ng/L (<4); AST 25U/L; Total cholesterol ultrasound shows evidence of multiple small
4.2mmol/L; Hb 17.4g/dl; Haematocrit 0.58; ovarian cysts bilaterally.
MCV 86fl
What is the most effective treatment for her
Which would be the most appropriate in his condition?
management?
1- Dianette
1- Change his testosterone preparation to
testosterone gel 2- Oral contraceptive pill

2- Stop testosterone treatment 3- Metformin

3- Advise him to recheck his testosterone 4- Diet and lifestyle changes


level at 9am 5- Topical eflornithine cream (Vaniqua)
4- Change him to a depot testosterone
preparation such as testosterone Answer & Comments
undecanoate (Nebido)
4- Diet and lifestyle changes
5- Increase the frequency of injections to
every 2 weeks This girl has polycystic ovarian syndrome and
impaired glucose tolerance. In general, the
Answer & Comments management of polycystic ovarian syndrome
should be tailored individually to the
2- Stop testosterone treatment presenting complaint. However, weight loss as
a result of diet and lifestyle changes has been
If the haematocrit is 54% or over, then shown to improve all manifestations of the
testosterone treatment should be stopped syndrome, including resolution of hirsuitism
until the haematocrit decreases to a safe level. and acne, resumption of ovulation,
The patient should also be evaluated for improvement in fertility and normalisation of
hypoxia and sleep apnoea syndrome. When, or glucose tolerance. Metformin treatment may
if, treatment is reinitiated, it should be at a also be indicated in this girl but only after diet
reduced dose. and lifestyle measures have been put in place.
Topical eflornithine may improve hirsuitism,
Further reading but will not have any effect on her other
Bhasin S, et al. Testosterone therapy in men with symptoms. Cyclical oestrogen will result in
androgen deficiency syndromes: an Endocrine regular withdrawal bleeds and can be used if
Society clinical practice guideline. J Clin irregular periods are a major issue, but may
Endocrinol Metab 2010;95: 2536–59.
make weight loss more difficult.

Further reading
Royal College of Obstetricians and Gynaecologists.
Long term consequences of Polycystic Ovarian
Syndrome. Green-top guideline No. 33. 2007.

18 | Dr. Khalid Yusuf (FB: Sohag Endocrine Group)


SCE Endocrinology (sce.practicaldiabetes.com), 2017

Q 26 Deafness – due to mutation of the


mitochondrial MT-TL1 gene). Mitochondrial
An 18-year-old male is seen in the diabetes DNA is inherited exclusively from the mother
clinic. He was originally diagnosed with type 1 and therefore these conditions are maternally
diabetes at the age of 5 years old. He has a inherited.
strong family history of diabetes and
consequently underwent genetic testing at the Further reading
age of 12 which showed he had a form of Website of the Diabetes Research Department
monogenic diabetes. During the consultation and the Centre for Molecular Genetics at the
he asks you about the possibility that he could Peninsula Medical School and Royal Devon
and Exeter Hospital. www.diabetesgenes.org
pass this condition onto any future children
[accessed 15.02.2011]. Kadowaki T, et al. A
that he might have. subtype of diabetes mellitus associated with a
Which of the following gene mutations is not mutation of mitochondrial DNA. N Engl J Med
autosomal dominant? 1994;330:962–8.

1- Hepatic nuclear factor 1 alpha (HNF1?)


Q 27
mutation
2- Glucokinase mutation A 30-year-old woman with a 14-year history of
3- Mitochrondial DNA mutation type 1 diabetes was referred for preconception
counselling. She was taking insulin aspart three
4- Permanent neonatal diabetes caused by times daily and insulin glargine at night. Her
insulin gene mutation home blood glucose monitoring was
5- Hepatic nuclear factor 1 beta (HNF1?) haphazard but the few readings provided
mutation ranged from 14–22 and her HbA1c level was
found to be 13.5% (124mmol/mol). She was
reviewed on a weekly basis by the diabetes
Answer & Comments
specialist nurses who supervised upward
3- Mitochrondial DNA mutation titration of her insulin doses and more
frequent home blood glucose monitoring. Her
HNF1α, HNF1β, glucokinase and insulin gene blood glucose readings improved dramatically
mutations are all autosomal dominant over the next 6 weeks with most readings
inherited forms of monogenic diabetes, eventually ranging from 5–8. She presents
although penetrance differs between the again urgently to the diabetes clinic with a 3-
types. Diabetes caused by HNF1α mutations week history of severe burning pain in both
can be very successfully treated with feet together with marked postural
sulphonylureas. HNF1β mutation induced hypotension, nausea and palpitations. Physical
diabetes is associated with renal cysts and examination reveals marked peripheral
often requires insulin treatment, although it neuropathy in a stocking distribution, together
may initially be possible to use diet and oral with a supine blood pressure of 105/72mmHg
hypoglycaemic medication. Glucokinase and standing blood pressure of 89/60mmHg.
mutations result in mildly elevated blood How would you treat her?
glucose levels which often do not require
treatment except in specific situations such as 1- Reduce insulin doses
pregnancy. Insulin gene mutations are an 2- Duloxetine
important cause of permanent neonatal
diabetes and this can often also be treated 3- Hydrocortisone
successfully with sulphonylureas. Mutations of 4- Fludrocortisone
mitochondrial DNA can lead to a number of
illnesses such as DAD (Diabetes Mellitus and 5- Domperidone

19 | Dr. Khalid Yusuf (FB: Sohag Endocrine Group)


SCE Endocrinology (sce.practicaldiabetes.com), 2017

Answer & Comments 1- There is a 75% chance that this lesion is


malignant
1- Reduce insulin doses
2- The most appropriate management
This lady has insulin neuritis. This was would be annual observation with
precipitated by her rapid correction of her annual CT scanning
chronic hyperglycaemia with insulin. This can 3- She is likely to need perioperative
occur 4–8 weeks after tightening of glycaemic glucocorticoid replacement if she
control with insulin and symptoms are as a undergoes surgical removal
result of epineurial arteriovenous shunting,
endoneurial hypoxia and nerve ischaemia, 4- She requires a fludrocortisone
resulting in a severe acute painful sensory suppression test
neuropathy or more rarely an acute autonomic 5- Serum DHEAS and androstenedione
neuropathy. Treatment is by relaxation of levels ought to be checked
glycaemic control which can cause rapid
resolution of symptoms.
Answer & Comments
Further reading 3- She is likely to need perioperative
Tesfaye S, et al. Arterio-veneous shunting and glucocorticoid replacement if she
proliferating new vessels in acute painful undergoes surgical removal
neuropathy of rapid glycaemic control (insulin
neuritis). Diabetologia 1996;39:329–35.
Adrenal lesions such as these are likely to be
Holt IG, et al. (eds). Textbook of diabetes, 4th edn. benign in the majority of cases. Evaluation of
Wiley-Blackwell, 2010; 618. these lesions should include multiple
collections of urinary metanephrines to
Q 28 exclude a phaeochromocytoma and an
overnight dexamethasone suppression test in
A 65-year-old woman was noted to have right- all cases. A plasma renin/aldosterone ratio
sided 6.1cm adrenal adenoma on a CT scan should be checked in hypertensive patients or
which was requested by the surgical team if serum potassium is low or low/normal.
following a bout of acute abdominal pain. She Androgen concentrations are not routinely
is normally fit and well and is not on any measured unless there is evidence of
medication. No cause was found for her hyperandrogenism. Cortisol should suppress to
abdominal pain which resolved spontaneously. <50nmol/L after overnight dexamethasone
She was referred to the endocrinology clinic for testing; in this case, the inadequate
further evaluation. She had no residual suppression of cortisol (with a result
symptoms and felt well in herself. Her blood >140nmol/L) indicates that she may have
pressure was 124/78mmHg. Clinical subclinical hypercortisolism. These patients
examination was essentially normal with no may benefit from perioperatative cortisol
evidence of hirsuitism or virilisation. replacement. If there is unequivicol hormone
Investigation results are as follows: Urine hypersecretion, then adrenalectomy should be
metanephrine excretion x 3 were all within considered. However, the question of surgical
normal ranges Urine free cortisol excretion removal is more difficult in cases of subclinical
205, 210 and 208 (0–280nmol/L/24hrs) Cushing’s syndrome. Nevertheless, in this case,
Cortisol after overnight dexamethasone as the lesion is >4cm diameter, the patient
suppression 152nmol/L. should be referred for consideration of surgery
regardless.
Which of the following statements is true
regarding this patient? Further reading
AACE/AAES Guidelines. American Association of
Clinical Endocrinologists and American

20 | Dr. Khalid Yusuf (FB: Sohag Endocrine Group)


SCE Endocrinology (sce.practicaldiabetes.com), 2017

Association of Endocrine Surgeons Medical Answer & Comments


Guidelines for the management of adrenal
incidentalomas. 2009. 4- Cerebral salt wasting

The criteria for diagnosis of SIADH are:


Q 29
(1) hyponatraemia (with serum sodium
A 69-year-old man is referred by the <130mmol/L);
neurosurgical team. He was originally admitted
with an extradural haematoma following a (2) hypotonic plasma (with serum
traumatic head injury as the result of a road osmolarity <270mOsm/kg);
traffic accident. He suffered numerous other
injuries including multiple rib and sternal (3) inappropriately elevated urine
fractures. The neurosurgeons had performed a osmolarity (>100mOsm/kg) in the
surgical decompression 5 days previously; presence of dilute serum;
however, despite being extubated, he has been
slow to recover. Physical examination reveals (4) excessive renal sodium loss
he is drowsy and clinically dehydrated. (>20mmol/L);
Laboratory tests (performed day 5
postoperatively) demonstrated abnormal (5) absence of hypovolaemia; and
biochemistry:
(6) normal thyroid and adrenal function.
Sodium 119mmol/L
Therefore, although this patient’s biochemistry
Potassium 3.6mmol/L results are consistent with SIADH, he is
clinically dehydrated and also by inference
Urea 8.7mmol/L hypovolaemic, excluding this diagnosis.
Cerebral salt wasting is a rare condition which
Creatinine 151µmol/L causes hyponatraemia and dehydration after
cerebral injury/lesion/trauma/surgery by
Random cortisol 702nmol/L causing excessive sodium excretion via a
centrally mediated process which is not yet
Spot urinary sodium 52mmol/L fully understood. The biochemistry can be
virtually indistinguishable from SIADH with the
Urine osmolarity 290mOsm/kg (normal range only difference often being the total fluid
350–1000) status of the patient. The distinction is
important as the management of cerebral salt
Plasma osmolarity 231mOsm/kg (normal range
wasting depends on rehydration and salt
278–305)
replacement as opposed to SIADH, when the
The most likely diagnosis is: first-line treatment is fluid restriction. Diabetes
insipidus also causes dehydration and high
1- Excess intravenous fluid administration
urine output but is usually accompanied by
2- Syndrome of inappropriate antidiuretic hypernatraemia. Severe traumatic head injury
hormone secretion (SIADH) has been associated with pituitary dysfunction
3- Secondary adrenocortical insufficiency and ACTH deficiency leading to secondary
adrenocortical failure, but the cortisol level of
4- Cerebral salt wasting 702nmol/L would make this unlikely.
5- Diabetes insipidus
Further reading
Verbalis JG, et al. Hyponatremia treatment
guidelines 2007: expert panel
recommendations. Am J Med 2007;120(11

21 | Dr. Khalid Yusuf (FB: Sohag Endocrine Group)


SCE Endocrinology (sce.practicaldiabetes.com), 2017

Suppl 1):S1–21. Betjes M. Hyponatremia in Q 31


acute brain disease: the cerebral salt wasting
syndrome. Eur J Intern Med 2002;13:9–14.
A 45-year-old man was reviewed due to poor
glycaemic control. He was diagnosed with type
Q 30 2 diabetes about 3 years ago. He was initially
treated with diet alone and then subsequently
A 36-year-old woman is referred to the started on metformin which was eventually
endocrine clinic with abnormal thyroid titrated up to a dose of 1g bd. His latest HbA1c
function tests (TFTs). She gives a 3-year history is 8.4% (68mmol/mol).
of increased sweating and anxiety following an
assault, and initially her symptoms had been He is a non-smoker, drinks moderate amounts
attributed to post-traumatic stress disorder. of alcohol and has a BMI of 31kg/m2. He is also
Her TFTs are: an HGV driver and eats irregularly. The decision
was made to add nateglinide (a meglitinide) to
Free T4 30 (normal range 9–22) his treatment.
What is the mechanism of action of
Free T3 2.5 (normal range 1.1–3.2)
meglitinides?
TSH 0.7 (normal range 0.5–5.7) 1- Stimulation of a membrane receptor,
The most likely diagnosis is: SUR-1

1- TSH secreting pituitary adenoma 2- Insulin-mediated muscle glucose uptake

2- Resistance to thyroid hormone 3- Suppression of hepatic glucose output

3- Graves disease 4- Stimulation of a nuclear receptor, PPAR?

4- Pregnancy 5- Decreased fatty acid oxidation

5- Familial dysalbuminaemic hyperthyroxinaemia


Answer & Comments

1- Stimulation of a membrane receptor,


Answer & Comments SUR-1

5- Familial dysalbuminaemic Meglitinides bind to the benzamido site of the


hyperthyroxinaemia sulphonylurea receptor (SUR-1) on pancreatic
beta-cells, which closes ATP-sensitive Kir6.2
Familial dysalbuminaemic hyperthyroxinaemia potassium channels and thus stimulates
(FDH) is caused by a mutation in albumin which release of pre-formed insulin. The main mode
then binds T4 with higher affinity. The free T4 of action of metformin is suppression of
assay does not distinguish this condition from hepatic glucose output; however, it also
normal. In this case, the raised T4 in the increases insulin-mediated muscle glucose
absence of suppressed TSH suggests either an uptake and decreases fatty acid oxidation.
assay artefact, e.g. heterophile antibodies, or a Thiazolidinediones increase tissue sensitivity to
binding protein problem, e.g. FDH. Resistance insulin mainly by activating a nuclear receptor
to thyroid hormone is less likely, as the free T3 called peroxisome proliferator-activated
is not elevated. receptor gamma (PPARγ) which forms a
complex with the retinoid X receptor (RXR) and
Further reading is mostly found in adipose tissue.
Ruiz M, et al. Familial Dysalbuminemic
Hyperthyroxinemia – a syndrome that can be
Further reading
confused with thyrotoxicosis. N Engl J Med
Holt RIG, et al. (eds). Textbook of diabetes. 4th
1982;306:635–9.
edn. Wiley-Blackwell, 2010; 455.

22 | Dr. Khalid Yusuf (FB: Sohag Endocrine Group)


SCE Endocrinology (sce.practicaldiabetes.com), 2017

Q 32 Q 33

A 62-year-old woman with newly diagnosed A 57-year-old woman with a 10-year history of
type 2 diabetes attends for an education type 2 diabetes and hypertension attends
session. She is on metformin treatment clinic. Recent blood tests show a raised serum
prescribed by her GP. She is a smoker and creatinine.
currently smokes 10 cigarettes per day. Her
According to NICE guidance, which one of the
blood pressure is hypertensive at
following would not be an indication to initiate
142/77mmHg and she is overweight with a BMI
of 32kg/m2. Recent blood tests have revealed investigations to look for a cause of renal
an LDL-cholesterol of 2.6mmol/L, an disease other than diabetes?
HDLcholesterol of 0.8mmol/L and an HbA1c of 1- Absence of significant or progressive
7.7% (61mmol/mol). She has been doing some retinopathy
research on the internet about the risks of
diabetes and heart disease as a close friend of 2- Absence of microalbuminuria or
hers has recently died from a myocardial proteinuria
infarction. 3- Hypertension is resistant to treatment
She asks you what is her most important 4- Presence of haematuria
coronary heart disease risk factor:
5- Presence of systemic illness
1- Systolic blood pressure
2- Smoking Answer & Comments
3- HDL-cholesterol 2- Absence of microalbuminuria or
4- HbA1c proteinuria

5- LDL-cholesterol Recommendation 97 in the NICE guidance for


type 2 diabetes (CG66) lists the indications for
Answer & Comments investigation for non-diabetic causes of renal
disease in a diabetic patient with renal
5- LDL-cholesterol impairment. It states that non-diabetic renal
disease should be suspected if patients first
The UKPDS risk engine is a type 2 diabetes have a documented normal ACR and then
specific risk calculator based on 53 000 patient develop heavy proteinuria. Other criteria
years of data from the UKPDS. It provides risk include no significant or progressive
estimates for individuals with type 2 diabetes retinopathy, resistant hypertension,
not known to have heart disease. This risk haematuria, rapid worsening of glomerular
engine stratifies LDL-cholesterol as the most filtration rate and systemic illness.
important independent determinant of
coronary heart disease (CHD), followed in Further reading
order by HDL-cholesterol, HbA1c, systolic National Collaborating Centre for Chronic
blood pressure and smoking. Of course, it Conditions. Type 2 diabetes: national clinical
should be explained to the patient that guideline for management in primary and
effective CHD risk reduction is multifactorial. secondary care (update). London: Royal
College of Physicians, 2008
Further reading
Website of Oxford Diabetes Trials Unit. Q 34
[accessed08.05.2011
Stevens RJ, et al. The UKPDS risk engine: a model A 51-year-old man is seen in the endocrinology
for the risk of coronary heart disease in type 2 clinic for a follow-up appointment. He was
diabetes. Clin Sci (Lond) 2001;101:671–9.

23 | Dr. Khalid Yusuf (FB: Sohag Endocrine Group)


SCE Endocrinology (sce.practicaldiabetes.com), 2017

originally referred to the clinic by his GP that he had difficulty adducting his right eye.
following an incidental finding of a 5.1cm His visual fields were normal to confrontation
adrenal adenoma on MRI scanning. The MRI and his blood pressure was 102/58mmHg.
was performed privately by an independent Further history from the patient revealed that
provider as part of a screening ‘health check he had recently been investigated for a
package’, which was purchased by the patient. ‘pituitary lump’ at a nearby hospital. This had
He has no symptoms, no past medical history been found incidentally when he underwent CT
of note and works as an investment banker. scanning for a sinus problem. He was unsure of
Urine catecholamines, renin/aldosterone ratio the nature of this lesion. He recalls attending
and an overnight dexamethasone suppression for a synacthen test and thinks that this was
test were arranged when he was originally normal. His only other medical problem was
assessed in clinic – these are all normal. irritable bowel syndrome. He has had a CT scan
of his head, which was reported as normal by
What would be the most appropriate next
the on-call radiology registrar. The on-call
step?
medical team enquire whether he needs any
1- Repeat MRI scan in 3 months endocrine input in view of the previous history
of the pituitary lesion.
2- Repeat MRI scan in 6 months
What is the most appropriate next step?
3- Repeat MRI scan in 12 months
4- Refer for surgical removal 1- Obtain an urgent MRI scan of the brain
and pituitary
5- Reassure and discharge from clinic
2- Urgent referral and transfer to the
regional neurosurgical centre for
Answer & Comments consideration of surgery
4- Refer for surgical removal 3- Urgent formal perimetry
4- Lumbar puncture for xanthochromia
Although the patient has no symptoms, the
scan was done incidentally and the endocrine 5- Intravenous hydrocortisone
investigations suggest that the adenoma is not
functional, there is a risk of adrenal carcinoma. Answer & Comments
The criteria for surgical removal of an adrenal
mass is a diameter of 4cm or more as the risk 5- Intravenous hydrocortisone
of primary carcinoma in such a lesion is 3–4.
This patient may have pituitary apoplexy. This
Further reading can occur in 2–7% of pituitary
AACE/AAES Guidelines. American Association of macroadenomas, most often in non-
Clinical Endocrinologists and American functioning lesions. Hypertension is the most
Association of Endocrine Surgeons Medical common precipitating factor. Clinical features
Guidelines for the management of adrenal are sudden onset headache, ocular palsies, and
incidentalomas. 2009. reduction in visual acuity and visual field
defects. Current UK guidelines suggest that in
Q 35 haemodynamically unstable patients with
suspected pituitary apoplexy, IV
You are referred a 58-year-old man who was hydrocortisone should be administered after
admitted a few hours ago by the on-call drawing blood samples for baseline endocrine
medical team, with a sudden onset headache function tests including random serum cortisol.
and vomiting. At the time of admission his GCS In addition, other indications for steroids in
was 15/15. Neurological examination revealed pituitary apoplexy are altered consciousness
that he had a diplopia looking upwards and also level, reduced visual acuity, severe visual field
on looking to the left. In addition, it was noted defects and a 9am cortisol level of <550nmol/L.

24 | Dr. Khalid Yusuf (FB: Sohag Endocrine Group)


SCE Endocrinology (sce.practicaldiabetes.com), 2017

CT scans may not always be diagnostic in Random GH 13ng/ml (normal range 0.05–10.0
pituitary apoplexy and MRI scanning is the
investigation of choice. Formal visual field TSH 0.2mU/L (normal range 0.5–5.5
assessment should be done within 24 hours
provided the patient is stable. When the Free T4 9pmol/L (normal range 9–25)
diagnosis is confirmed, all patients should be What is the most likely explanation of these
transferred to a specialist blood results?
endocrine/neurosurgical team once stable.
The decision to initially manage the patient 1- Growth hormone secreting adenoma
conservatively or surgically should be taken 2- Anorexia nervosa
after a multidisciplinary team discussion.
Patients with pituitary apoplexy who are 3- Pregnancy
without any neuroophthalmic signs or have 4- Alcohol abuse
mild and stable signs can be considered for
conservative management. They should be 5- Surreptitious thyroxine self-
carefully monitored for neuro-ophthalmic administration
deterioration. Surgery should be considered if
the neuro-ophthalmic signs fail to improve or Answer & Comments
deteriorate. All patients require long-term
follow up for tumour recurrence and endocrine 2- Anorexia nervosa
function.
This is the classical biochemical picture seen in
anorexia nervosa, with functional
Further reading
Rajasekaran S, et al. UK guidelines for the
gonadotrophin deficiency, a ‘sick-euthyroid’
management of pituitary apoplexy. Clin picture in the TFTs, elevated baseline GH levels
Endocrinol (Oxf) 2011;74:9–20 and a low IGF-1. If tested, random cortisol
levels would also be elevated. Acromegaly
wouldbe associated with an elevated IGF-1 and
Q 36
none of the other causes explains all the
biochemical features.
A 21-year-old female university student is
referred with a 1-year history of fatigue and Further reading
amenorrhoea. Prior to these symptoms she Warren MP. Endocrine manifestations of eating
was generally well and has no past medical disorders J Clin Endocrinol Metab
history of note. She went through menarche at 2011;96:333–43.
the age of 13 years and her periods were
regular up to 2 years ago when they became Q 37
infrequent and irregular. They stopped
completely 12 months ago. This was associated
A 58-year-old man was referred to the diabetes
with an increased general feeling of weakness
clinic by his GP because of painful feet. He gives
and exhaustion even when carrying out the
a 12-year history of sub-optimally controlled
slightest tasks. Her blood results show:
type 2 diabetes mellitus which has been
LH 1U/L (normal range 5–25 – depending on complicated by chronic kidney disease and
phase in cycle) proteinuria over the last few years. Pre-clinic
blood test reveals an HbA1c of 8.9%
FSH 2mU/L (normal range 4.7–21.5 – (74mmol/mol) and a creatinine of 230µmol/L
depending on phase in cycle) (eGFR 27ml/min/1.73m2). He also has
ischaemic heart disease and has had a
Oestradiol <50pg/ml myocardial infarction 2 months ago, and a
subsequent echocardiogram demonstrated
IGF-1 10ng/ml (normal range 18–35) the presence of moderate left ventricular

25 | Dr. Khalid Yusuf (FB: Sohag Endocrine Group)


SCE Endocrinology (sce.practicaldiabetes.com), 2017

impairment. His main complaint is severe renally; therefore, the dose should be reduced
burning pain in his feet, which was worse at in renal impairment.
night. On examination, his feet are warm with
good peripheral pulses and mild pitting Fluoxetine has not been shown to be
oedema. Neurological examination confirms efficacious in diabetic peripheral neuropathy.
the presence of impaired vibration and pain
sensation extending up to his knees. Further reading
Cardiovascular examination reveals fine National Institute for Health and Clinical
inspiratory crackles over his lung bases and an Excellence. Neuropathic pain: the
elevated jugular venous pulse (JVP). His GP has pharmacological management of neuropathic
pain in adults in non-specialist settings. Clinical
enclosed some recent investigations in the
guideline 96. NICE, 2010.
referral letter including an ESR of 5mm/hr, and
B12 levels of 468pmol/L (normal range 140– Ziegler D. Painful diabetic neuropathy: advantage
of novel drugs over old drugs? Diabetes Care
780). His current treatment includes insulin
2009;32(Suppl 2):S414–9.
glargine 92 units at night, insulin aspart 56
units with meals, aspirin 75mg od, clopidogrel
75mg od, ramipril 10mg od, atorvastatin 80mg Q 38
od, frusemide 80mg od, bisoprolol 1.25mg od
and spironolactone 25mg od. A 19-year-old man was assessed in the
diabetes clinic as a new patient about 8 weeks
Which is the most appropriate treatment
previously with newly diagnosed diabetes. At
option for this patient?
that time he had severe osmotic symptoms of
1- Duloxetine thirst and polyuria, but was unsure as to
whether he had lost any weight in the
2- Amitriptyline
preceding weeks. He also had recurrent thrush
3- Pregabalin and blurred vision. There was no family history
4- Imipramine of diabetes. He was also found to be morbidly
obese with a weight of 120kg and a BMI of
5- Fluoxetine 41.5kg/m2. His urine tested negative for
ketones, and a fasting blood glucose done by
Answer & Comments his GP was 16mmol/L. His GP stated in the
referral letter to the clinic that he was
3- Pregabalin uncertain as to whether this patient had type 1
or type 2 diabetes mellitus. His autoantibodies
Duloxetine is contraindicated in renal were checked to try and help diagnose the type
impairment where the eGFR is of diabetes. The patient attends for a review
2
<30ml/min/1.73m . Amitriptyline is appointment and it is noted that his
contraindicated in the immediate recovery cytoplasmic islet cell antibody (ICA) serology
period following a myocardial infarction (<6 was negative.
months). In addition, the summary of product
characteristics for amitriptyline states that it What proportion of patients with type 1
should be avoided in congestive cardiac failure. diabetes will exhibit high titres of cytoplasmic
Similar contraindications exist for imipramine. ICA?
1- 5–10%
Current NICE guidance on neuropathic pain
recommends pregabalin over gabapentin, as 2- 30–40%
the former has a simpler dosing schedule and 3- 50–60%
was considered more cost effective (although
it should be noted that at the time of writing 4- 70–90%
this is under review). Pregabalin is excreted 5- 100%

26 | Dr. Khalid Yusuf (FB: Sohag Endocrine Group)


SCE Endocrinology (sce.practicaldiabetes.com), 2017

Answer & Comments 5- Systematic digital retinal screening


programme
4- 70–90%

There are 4 autoantibodies which are markers Answer & Comments


of beta-cell autoimmunity in type 1 diabetes.
4- Patient satisfaction surveys
These are islet cell antibodies (ICA, against
cytoplasmic proteins in the beta cell), The NSF for Diabetes in England contains 12
antibodies to glutamic acid decarboxylase standards and was published in the form of
(GAD-65), insulin autoantibodies (IAA), and IA- separate standards and delivery documents.
2A to protein tyrosine phosphatase. The provision of patient-held records
Autoantibodies against GAD 65 are found in andpersonal care plans are part of standard 3.
80% of people with type 1 diabetes at clinical (‘All children, young people and adults with
presentation. High titres of ICA and IA-2A at diabetes will receive a service which
diagnosis for type 1 diabetes range from 69– encourages partnership indecision-making,
90% and 54–75% in affected patients, supports them in managing their diabetes and
respectively. Cytoplasmic ICA are IgG helps them to adopt and maintain a healthy
antibodies which can react on frozen sections lifestyle. This will be reflected in an agreed and
of human, rat or mouse pancreas. High titres of shared care plan in an appropriate form at and
ICA are also found in approximately 3% of first language. Where appropriate, parents and
degree relatives of patients with type 1 carers should be fully engaged in this process.’)
diabetes and in 0.2% of the general population. The requirement for systematic digital
IAA prevalence correlates inversely with age at retinopathy screening is enshrined in standard
onset of diabetes; it is usually the first marker 10. (‘All young people and adults with diabetes
in young children at risk for diabetes and is will receive regular surveillance for the long-
found in approximately 70% of young children term complications of diabetes.’) The delivery
at the time of diagnosis. strategy of the NSF clearly states the need for
practice-based diabetes registers in order to
Further reading plan services, meet standards and identify
Taplin CE, Barker JM. Autoantibodies in type 1 patients who require intervention. Although
diabetes. Autoimmunity 2008;41:11–8. Winter
patient satisfaction surveys could be
WE, et al. Type 1 diabetes islet autoantibody
markers. Diabetes Technol Ther 2002;4:817– considered part of good medical practice and
39. may well be a current appraisal and a future
revalidation requirement, they are not an
explicitly stated NSF requirement.
Q 39
Further reading
The National Service Framework (NSF) for Department of Health. National Service
Diabetes in England defines clear quality Framework for Diabetes: Standards, 2001.
requirements for diabetes care, and provides [accessed 9.6.2011]. Department of Health.
strategies to achieve these standards. National Service Framework for Diabetes:
Delivery Strategy, 2003. [accessed 9.6.2011].
Which of the following is not an explicitly stated
Diabetes NSF requirement?
Q 40
1- Patient-held records
2- Personal care plans A 27-year-old woman was diagnosed with
Cushing’s syndrome during pregnancy at 26
3- Practice-based diabetes registers weeks’ gestation. She gave a 2-year history of
4- Patient satisfaction surveys hypertension and had gestational diabetes
mellitus in a previous pregnancy. Her

27 | Dr. Khalid Yusuf (FB: Sohag Endocrine Group)


SCE Endocrinology (sce.practicaldiabetes.com), 2017

symptoms included hirsutism, weight gain and contraindicated because it crosses the
bruising. Investigations confirmed a diagnosis placenta and is teratogenic.
of Cushing’s syndrome, but subsequently she
was found to be pregnant. Physical Further reading
examination revealed a BMI of 35kg/m2, blood Lindsay JR, et al. Cushing's syndrome during
pressure of 141/87mmHg, multiple pigmented pregnancy: personal experience and review of
striae, and a gravid uterus which was the literature. J Clin Endocrinol Metab
consistent with gestational age. A decision is 2005;90:3077–83. Biller BMK, et al. Treatment
of adrenocorticotropin-dependant Cushing’s
made to start her initially on primary medical
syndrome: A Consensus Statement. J Clin
therapy, pending definitive treatment.
Endocrinol Metab 2008;93:2454–62.
What would be the initial drug of choice?
1- Metyrapone Q 41

2- Ketoconazole
A 65-year-old lady attends the endocrine clinic
3- Cyproheptidine for annual follow up. She had a follicular
4- Aminoglutethimide thyroid carcinoma diagnosed 5 years ago and
underwent total thyroidectomy followed by
5- Mitotane 131I ablation therapy. She is currently taking
levothyroxine 200µg daily and 1-alfacalcidol.
Answer & Comments She feels well in herself. Physical examination
does not show any evidence of further nodules
1- Metyrapone in her neck. Pre-clinic blood results show: TSH
<0.1mU/L (normal range 0.5–5.5) Free T4 22
Management of Cushing’s syndrome in (normal range 9–21) Serum thyroglobulin –
pregnancy represents a significant challenge undetectable Serum adjusted calcium
and untreated is associated with significant 3mmol/L (normal range 2.2–2.8)
maternal morbidity including diabetes,
hypertension, heart failure, and pre-eclampsia. What should be the next correct course of
Detection of Cushing’s syndrome usually management?
occurs late in gestation, and the diagnosis can 1- Reassurance and discharge
be complicated by the signs of normal
pregnancy, such as central weight gain, facial 2- Reduce levothyroxine dose to achieve
plethora, and pigmentation. Surgery is the normal level of TSH
preferred treatment (except late in the 3- Discontinue levothyroxine and repeat
thirdtrimester), but primary medical therapy thyroid function test in 6 weeks
can be used as an interim measure. There is
most experience with metyrapone, which 4- Continue on same dose of levothyroxine
seems generally well tolerated. Although 5- Arrange whole body scan (WBS) after
ketoconazole has been used successfully in stopping levothyroxine for 4 weeks
some pregnancies without adverse event, in
rat models ketoconazole has been shown to Answer & Comments
cross the placenta and be teratogenic and
abortifacient, so that the drug is FDA category 4- Continue on same dose of levothyroxine
C. Therefore it should be reserved for
individuals who need emergent medical The British Thyroid Association guidelines for
therapy but cannot tolerate metyrapone. the management of thyroid cancer
Cyproheptadine is not recommended due to recommend lifelong follow up for patients with
lack of efficacy. Fetal masculinisation precludes differentiated thyroid cancer, as the disease
the use of aminoglutethimide. Mitotane is has a long natural history and late recurrences
are not rare and can be treated. Lifelong

28 | Dr. Khalid Yusuf (FB: Sohag Endocrine Group)


SCE Endocrinology (sce.practicaldiabetes.com), 2017

suppression of TSH below normal (<0.1mU/L) is syndrome, which would be the main
recommended. In patients confirmed to be low differential diagnosis.
risk, a serum TSH <0.5mU/L is probably
acceptable. Regular follow up is also necessary Q 43
to assess the consequences of supra-
physiological doses of levothyroxine and hypo
A 17-year-old boy is referred to the transitional
- calcaemia treatment. Thyroglobulin is
diabetes service by the paediatric diabetes
secreted by both normal and cancerous thyroid
service. He was diagnosed with diabetes at the
cells. If this patient had detectable
age of 12, when it was found that he had a
thyroglobulin, it is suggestive of recurrence of
raised blood glucose level after minor surgery.
thyroid carcinoma as she had total
He was asymptomatic at the time and has
thyroidectomy and131I ablation previously.
never been hospitalised with diabetic
ketoacidosis. He is an only child. On further
Further reading
questioning, his mother mentions that she was
British Thyroid Association & Royal College of
Physicians. Guidelines for the management of
diagnosed with gestational diabetes, but that
thyroid cancer, 2nd edn. Report of the Thyroid she had defaulted on post-natal clinic visits. His
Cancer Guidelines Update Group. London: maternal grandfather, who is now deceased,
2007. had diabetes for many years. His mother
mentions that the patient’s uncle from the
maternal side was diagnosed with diabetes in
Q 42
his 30s. The patient is currently on insulin
aspart 2 units three times a day and insulin
A 16-year-old girl is referred to clinic with a glargine 2 units at night. On referral, his HbA1c
secondary amenorrhoea and a prolactin of is 5.6% (IFCC 38mmol/ml) and his BMI is
900mU/L. She is obese and hirsute. She went 21kg/m2.
through menarche at the age of 12, but since
this time has gained considerable weight. Her What is the next most appropriate step?
periods were initially regular, but have become 1- Stop insulin
increasing irregular since then.
2- Stop insulin and start the patient on a
What should be the next choice of trial of gliclazide
investigation?
3- Test for glucokinase mutations
1- Serum 17-hydroxyprogesterone
4- Test for mitochondrial diabetes
2- MRI of the pituitary
5- Test for HNF1b mutations
3- Ultrasound of the ovaries
4- Serum FSH, LH and testosterone levels Answer & Comments
5- Beta human chorionic gonadotropin 3- Test for glucokinase mutations
levels
This patient has a 3-generation history of
Answer & Comments diabetes with evidence of non-insulin
dependence (small insulin requirement and no
5- Beta human chorionic gonadotropin history of ketoacidosis). Monogenic diabetes
levels or maturity-onset diabetes of the young
(MODY) should beconsidered in such cases.
The first investigation in any case of secondary These conditions are associated with variable
amenorrhoea should be to exclude pregnancy. defects in beta-cell function with minimal
Pregnancy would be consistent with a high insulin resistance. They are inherited in an
prolactin level, as would polycystic ovarian autosomal dominant manner. Several subtypes

29 | Dr. Khalid Yusuf (FB: Sohag Endocrine Group)


SCE Endocrinology (sce.practicaldiabetes.com), 2017

are described. The more common ones are medication includes Humulin M3 26?units am
listed below. and 22?units pm. He also takes metformin 1g
bd, ramipril 10mg od and simvastatin 40mg od.
MODY 1. Caused by mutations of hepatocyte He conducts frequent home blood glucose
nuclear factor 4a (HNF4a). This accounts for monitoring and has recently heard about the
about 10% of MODY cases. They can be treated changes in the way in which HbA1c will be
with sulphonylurea initially but may need reported. His pre-clinic HbA1c is 9% (DCCT
insulin with disease progression. aligned). He asks you what this value would be
in the new units.
MODY 2. Caused by glucokinase mutations.
This accounts for about 32% of cases. Patients What would be his HbA1c using the IFCC
have a slightly elevated fasting blood glucose reference method?
(5.5–8mmol/L) and this is present from birth. 1- 55mmol/mol
This remains stable throughout life. Typically,
the rise in blood glucose during an oral glucose 2- 60mmol/mol
tolerance test is small (less than 3mmol/L). This 3- 65mmol/mol
is often identified during routine screening and
rarely requires treatment except during 4- 70mmol/mol
pregnancy. 5- 75mmol/mol

MODY 3. Caused by mutations of HNF1a and is


Answer & Comments
associated with chromosome 12. Thisaccounts
for 52% of MODY cases. They develop diabetes 5- 75mmol/mol
mainly in their late teens or early 20s. They
respond well to sulphonylurea. A new standard specific for HbA1c has been
prepared by the International Federation of
MODY 5. Caused by mutations of HNF1b. The Clinical Chemistry and Laboratory Medicine
most common phenotype is renal cysts and (IFCC) and HbA1c results will be expressed as
diabetes (RCAD syndrome). mmol/mol of unglycated haemoglobin.
Manufacturers will supply IFCC standardised
Confirmation of the diagnosis by genetic values for their calibrators as well as DCCT-
testing is considered prudent prior to aligned values. The HbA1c value using the new
withdrawal of treatment or stopping insulin. IFCC reference method can be calculated from
the existing DCCT aligned units using the
Mitochondrial diabetes is maternally inherited
equation: IFCC-HbA1c (mmol/mol) = [DCCT-
and is unlikely as the maternal grandfather was
HbA1c (%) -2.15] x 10.929.
affected.
However, this may be too complex to use in an
Further reading outpatient clinic setting and a much simpler
Shields BM, et al. Maturity-onset diabetes of the way of converting DCCT to IFCC exists called
young (MODY): how many cases are we
‘Kilpatrick’s Kludge’ or the ‘minus 2, minus 2’
missing? Diabetologia 2010;53:2504–8.
method. By means of example, in this case as
Ellard S, et al. Best practice guidelines for the the DCCT-aligned HbA1c is 9%, the IFCC HbA1c
molecular genetic diagnosis of maturity-onset
is 9 minus 2 (7), minus 2 (5) = 75mmol/mol. This
diabetes of the young. Diabetologia
works for other whole number percentages of
2008;51:546–53.
HbA1c, and also holds true for all percentages
between 4% and 13% inclusive.
Q 44
For DCCT numbers that are not whole
A 61-year-old man with an 8-year history of numbers, the slightly more complicated
type 2 diabetes attends clinic. His current ‘Middle’s Manipulation’ can be used. To do this

30 | Dr. Khalid Yusuf (FB: Sohag Endocrine Group)


SCE Endocrinology (sce.practicaldiabetes.com), 2017

the DCCT value should be multiplied by 11, 4- People with diabetes who have
then 24 should be subtracted from the experienced hypoglycaemia requiring
resulting number to get the IFCC value. For medical attention are referred to a
example, for a DCCT result of 7.5%, the IFCC specialist diabetes team
value is 7.5 x 11 = 82.5; minus 24 =
5- People with diabetes are assessed for
58.5mmol/mol. A reversal of this can be used
psychological problems, which are then
to estimate a DCCT-aligned value from an IFCC
managed appropriately
value, i.e. add 24 to the IFCC value and divide
by 11. So an IFCC HbA1c of 80mmol/mol would
be 104/11, which is 9.5% as a DCCT value. Answer & Comments

2- People with type 1 diabetes receive


Further reading
access to continuous subcutaneous
Kilpatrick ES. Consensus meeting onreporting
glycated haemoglobin and estimated average
insulin infusion (CSII) therapy and
glucose in the UK: time for ‘Kilpatrick’s accompanying education, when clinically
Kludge’? Ann Clin Biochem 2009;46(Pt 1):84– appropriate to do so
5.
National Institute for Health and Clinical
www.diabetes.org.uk/Professionals/Publications-
reports-and-resources/Tools/Changes-to- Excellence (NICE) quality standards for
HbA1c-values [accessed 9 June 2011]. diabetes are a set of 13 specific evidence-based
statements that aim to act as markers of high
quality and cost-effective diabetes patient care
Q 45
against which the effectiveness of diabetes
service commissioning can be measured.
You are invited to deliver a presentation about
the National Institute for Health and Clinical They have been developed from existing NICE
Excellence (NICE) quality standards for guidance and other evidence sources
diabetes at a meeting attended by the local accredited by NHS Evidence. They address 3
Clinical Commissioning Group leads for dimensions of quality: namely, clinical
diabetes and the local Diabetes UK patient effectiveness, patient safety and patient
representative group. experience.
Which of the following is not a NICE quality
The need for care planning (quality standard 3),
standard for diabetes?
structured education (quality standards 1 and
1- People with diabetes participate in 2), referral to specialist team in cases where
annual care planning which leads to hypoglycaemia requires medical attention
documented agreed goals and an action (quality standard 13) and
plan assessment/management of psychological
problems (quality standard 9) are all separate
2- People with type 1 diabetes receive
NICE diabetes quality standards. Although
access to continuous subcutaneous
adequate access to insulin pump therapy is
insulin infusion (CSII) therapy and
mandated by a NICE technology appraisal, it is
accompanying education, when clinically
not stated specifically as a NICE diabetes
appropriate to do so
quality standard.
3- People with diabetes and/or their carers
receive a structured educational Further reading
programme that fulfils the nationally National Institute for Health and Clinical
agreed criteria from the time of Excellence. Quality Standard for Diabetes in
diagnosis, with annual review and access Adults. March 2011.
to ongoing education http://www.nice.org.uk/guidance/qualitystan
dards/diabetesinadults/diabetesinadultsquali

31 | Dr. Khalid Yusuf (FB: Sohag Endocrine Group)


SCE Endocrinology (sce.practicaldiabetes.com), 2017

tystandard.jsp Standard.pdf [accessed 18 thyroid cancer, 2nd edn. London: Royal College
November 2011]. of Physicians, 2007)

Q 46 Classification: THY1

Result: Non-diagnostic Inadequate


A 42-year-old woman is reviewed to the
endocrine clinic. She was originally referred Recommended action: FNAC (fine needle
due to a lump in the right side of her neck, aspiration for cytology) should be repeated
which was longstanding and caused no
symptoms. One of her friends had recently Classification: THY2
been diagnosed with lung cancer and this had
caused some anxiety prompting her to seek Result: Non-neoplastic
specialist referral and investigation. On
physical examination she was clinically Recommended action: 2 non-neoplastic
euthryoid and had a 1.5cm firm thyroid nodule results 3–6 months apart are generally
on the right lobe. At referral, her TFTs showed advisable to exclude neoplasia.
a TSH of 2.2mU/L (0.5–5.5) and a free T4 of
15.1pmol/L (9–21). A fine needle aspiration for In high clinical risk group cases, diagnostic
cytology (FNAC) of the nodule was arranged lobectomy may be indicated
and the thyroid aspiration cytology is reported
as THY1. Classification: THY3
What would be the next most appropriate Result: (1) Follicular lesion
action?
(2) Other suspicious findings
1- Reassure and discharge patient
2- Repeat FNAC, using ultrasound guidance Recommended action: Lobectomy with
if necessary completion thyroidectomy if histology proves
malignant.
3- Repeat FNAC in 6 months’ time
4- Offer lobectomy Discussion in the multidisciplinary team to
decide appropriate course of action
5- Offer lobectomy
Classification: THY4
Answer & Comments
Result: Suspicious of malignancy, but not
2- Repeat FNAC, using ultrasound guidance diagnostic of papillary, medullary or anaplastic
if necessary carcinoma or lymphoma
THY1 indicates that the sample was not Recommended action: Surgical intervention is
diagnostic and that the FNAC should be usually indicated for suspected cancer
repeated. Current guidelines for the
management of thyroid cancer give clear Classification: THY5
recommendations on the management and
classification of thyroid aspiration cytology Result: Diagnostic of malignancy
results.
Recommended action: Surgical excision for
Management and classification of thyroid differentiated thyroid cancer. Radiotherapy or
aspiration cytology results (adapted from: chemotherapy for anaplastic thyroid
British Thyroid Association & Royal College of carcinoma, lymphoma or metastatic tumour
Physicians. Guidelines for the management of

32 | Dr. Khalid Yusuf (FB: Sohag Endocrine Group)


SCE Endocrinology (sce.practicaldiabetes.com), 2017

Further reading rifampicin increase the metabolism of


British Thyroid Association & Royal College of dexamethasone giving rise to false negative
Physicians. Guidelines for the management of results.
thyroid cancer, 2ndedn. Report of the Thyroid
Cancer Guidelines Update Group. London: Urinary free cortisol excretion is not affected
Royal College of Physicians, 2007. by conditions or drugs which affect cortisol
binding globulin levels. False positive urinary
Q 47 free cortisol levels can be seen in high fluid
intake over 5L/day. In addition, carbamazepine
A 30-year-old woman was referred to the and fenofibrate may increase urinary cortisol
endocrinology clinic after she was found to excretion. False negative levels may be present
have left-sided 1.2cm adrenal adenoma on a CT if eGFR is less than 60ml/min/1.73m2.
scan carried out following an episode of right
upper quadrant pain. She has been previously Recommended initial tests of Cushing’s
fit and well and the only medication she was syndrome include x2 measurements of urinary
taking was the oral contraceptive pill. Clinical free cortisol excretion, x2 measurements of
examination was unremarkable except for a late night salivary cortisol levels, a 1mg
BMI of 31kg/m2. Her blood pressure was overnight dexamethasone suppression test or
122/81mmHg. She underwent an overnight a low dose (2mg) dexamethasone suppression
dexamethasone suppression test as part of the test.
work up. Her serum cortisol level the following
morning was 86nmol/L. Urine catecholamine Tests to determine the cause of Cushing’s
excretion was normal on two occasions. syndrome such as a high dose (8mg)
dexamethasone suppression testing, pituitary
Which of the following would be the most or adrenal imaging are not recommended as
appropriate next step? first line.
1- Measurement x2 of urinary free cortisol
levels Further reading
Nieman LK, et al. The diagnosis of Cushing’s
2- Adrenal vein sampling syndrome: an Endocrine Society Clinical
Practice Guideline. J Clin Endocrinol Metab
3- High dose dexamethasone suppression 2008;93:1526–40.
test
4- Repeat the overnight dexamethasone Q 48
suppression test using 2mg of
dexamethasone
An 18-year-old female presents to the
5- MRI scan of the adrenal gland endocrine clinic with hirsuitism, acne, mood
swings and oligomenorrhoea. She was
Answer & Comments diagnosed as having polycystic ovarian
syndrome in primary care, but requested a
1- Measurement x2 of urinary free cortisol second opinion when her symptoms
levels progressively worsened. After clinical
assessment and investigation, she is found to
False positive dexamethasone suppression have non-classical congenital adrenal
tests are seen in up to 50% of women on the hyperplasia (CAH). She would like to have a
oral contraceptive pill. This is due to oestrogen family and asks you about the risk of any of
increasing the cortisol binding globulin levels. If her future children suffering the disorder.
possible, then oestrogen containing drugs
What is the risk of this happening?
should be withdrawn for 6 weeks before the
test, but this may not always be practical. In 1- Virtually 0%
addition, drugs such as phenytoin and

33 |
2- Approximately 1% Q 49
3- Approximately 5%
A 31-year-old man presents to the acute
4- Approximately 25% medical assessment unit following
5- Approximately 50% deterioration of a chronic left foot ulcer. He
was diagnosed with type 1 diabetes at the age
of 4, and his diabetes is now complicated by
Answer & Comments
retinopathy and severe peripheral neuropathy.
2- Approximately 1% His glycaemic control is poor with an HbA1c of
10.6% (92mmol/mol). On physical examination
Non-classical congenital adrenal hyperplasia his temperature is 38.3°C. He has a deep ulcer
(CAH) is an autosomal recessive condition over his 1st metatarsal head, which is covered
which is due to mild deficiency of the 21- in greenish slough with a malodorous
hydroxylase enzyme. It may present at any age discharge. Blood tests show that his ESR is
with a variety of hyperandrogenic symptoms. 101mm/hr and white cell count is 16x109/L
Features of this condition may include (with a neutrophil count of 13x109).
premature development of pubic hair, severe Which investigation is best performed to
acne, advanced bone age and accelerated exclude osteomyelitis?
growth. Women may present with symptoms
of androgen excess, and secondary 1- Magnetic resonance imaging of his left
amenorrhoea is a frequent occurrence. foot
Polycystic ovarian syndrome may co-exist in 2- Plain radiograph left foot
these patients.
3- Probe to bone
A CAH carrier frequency of 1:55 is estimated 4- Bone biopsy from left foot
applying the Hardy-Weinberg equilibrium to
data on disease incidence obtained from 5- Indium white cell scan
newborn screening programmes. Thus, the
chances of this girl meeting a partner Answer & Comments
heterozygous for CAH is approximately 1 in 55
yet she is homozygous for the condition. 1- Magnetic resonance imaging of his left
Therefore, the chances of her having a child foot
with CAH is 1 in 2 with this person (with a 50%
chance that any children will be homozygous Conventional radiography has poor sensitivity
and a 50% chance that they will be unaffected). in early osteomyelitis and the changes seen on
Thus, the overall chance of having a child with the X-ray of patients with neuropathy might be
CAH would be 1 in 55 x 1 in 2 = 1 in 110 or indistinguishable from those of Charcot
roughly 1%. osteoarthropathy. Hence osteomyelitis cannot
be excluded by plain X-rays.
Further reading
Pang SY, et al. Worldwide experience in newborn If the clinical suspicion is sufficient, then MRI
screening for classical congenital adrenal scanning is the best and most cost-effective
hyperplasia due to 21-hydroxylase deficiency. option. However, if MRI scanning were to be
Pediatrics 1988;81:866–74. contraindicated then radio-labelled white cell
White PC, Speiser PW. Congenital adrenal scanning would be the next investigation of
hyperplasia due to 21-hydroxylase deficiency. choice.
Endocrine Rev 2000;21:245–91.
Use of bone scans such as 99mTc-MDP-labelled
99m
scintigraphy, Tc-HMPAO-labelled
scintigraphy, antigranulocyte Fab' fragment
antibody scintigraphy and 99mTc-labelled

34 | Dr. Khalid Yusuf (FB: Sohag Endocrine Group)


SCE Endocrinology (sce.practicaldiabetes.com), 2017

monoclonal antigranulocyte antibody Kaprio J, et al. Concordance for type 1 (insulin-


scintigraphy is not recommended. dependent) and type 2 (non-insulin-
dependent) diabetes mellitus in a population-
based cohort of twins in Finland. Diabetologia
Further reading
1992;35:1060–7.
National Institute for Health and Clinical
Excellence. Diabetic foot problems: inpatient Newman B, et al. Concordance for type 2 (non-
management. Clinical Guideline 119. London: insulin-dependent) diabetes mellitus in male
National Institute for Health and Clinical twins. Diabetologia 1987;30:763–8.
Excellence, 2011.
Q 51
Q 50
A man has a 7-year history of hypertension and
A 52-year-old woman was diagnosed with type poorly controlled type 2 diabetes. He has
2 diabetes approximately 6 months ago. She peripheral neuropathy and has had a previous
has received dietary advice and education, but foot ulcer, which has now healed. He is
is currently receiving no medication and has an diagnosed with significant diabetic retinopathy
HbA1c of 6.7% (50mmol/mol). She has an following digital eye screening.
identical twin sister and they are both
Which one of the following is not an indication
registered with the same GP. You receive a
for emergency referral to an ophthalmologist?
letter from her GP asking you whether he
ought to screen this patient’s identical twin for 1- Sudden loss of vision
diabetes.
2- Rubeosis iridis
What is the lifetime risk of this patient’s twin
3- Pre-retinal or vitreous haemorrhage
sister developing type 2 diabetes?
4- Retinal detachment
1- <5%
5- New vessels on the optic disc
2- 5–10%
3- 20–30% Answer & Comments
4- 40–50%
5- New vessels on the optic disc
5- >60%
Recommendation 110 in the National Institute
for Health and Clinical Excellence guidance for
Answer & Comments
type 2 diabetes (CG66) lists the indications for
5- >60% emergency referral to an ophthalmologist.
These are a sudden loss of vision, rubeosis
Since dizygotic twins share the environment iridis, pre-retinal or vitreous haemorrhage or
(both intrauterine and extrauterine) but only retinal detachment.
50% of their genes, concordance rates in
monozygotic twins in excess of those in In cases of new vessel formation on the optic
dizygotic twins have been used todistinguish disc, then the guidance recommends a rapid
genetic from non-genetic contributions. (rather than emergency) review by an
ophthalmologist (recommendation 111).
Most studies show monozygotic twins having a
higher concordance rate (60–95%) when Further reading
compared with dizygotic twins (3–37%), National Collaborating Centre for Chronic
providing evidence of a significant genetic Conditions. Type 2 diabetes: national clinical
component in type 2 diabetes. guideline for management in primary and
secondary care (update). London: Royal
College of Physicians, 2008.
Further reading

35 | Dr. Khalid Yusuf (FB: Sohag Endocrine Group)


SCE Endocrinology (sce.practicaldiabetes.com), 2017

Q 52 childhood have normal GH secretion during


late adolescence or young adulthood,
An 18-year-old male was seen in the presumably because of the stimulatory effects
transitional endocrinology clinic. He has been of gonadal steroid hormones on the
on growth hormone (GH) treatment since hypothalamic-pituitary axis for GH secretion.
childhood for isolated GH deficiency. He has 1
brother and 1 sister, both of whom have no As per NICE guidance, at completion of linear
health problems. He is currently applying to go growth (that is, growth rate <2cm/year), GH
to university. He had a normal puberty, and on treatment should be stopped for 2–3 months,
examination he is noted to have achieved his and then GH status should be re-assessed. GH
final predicted height with normal secondary treatment at adult doses should be restarted
sexual characteristics. He is keen to continue only in those satisfying the biochemical criteria
with GH therapy but is open to stopping it if it for severe GH deficiency (defined as a peak GH
is no longer clinically indicated. Investigations response of <9mU/L [3ng/ml] during an insulin
reveal: tolerance test or a cross-validated GH
threshold in an equivalent test), and continued
IGF-1 40.2nmol/L (normal range 28–50) until adult peak bone mass has been achieved
(normally around 25 years of age).
Testosterone 18nmol/L (9–30)
After adult peak bone mass has been achieved,
LH 1.6mU/L (2–10) the decision to continue GH treatment should
be based on the following criteria:
FSH 2.1mU/L (2–10)
The patient has severe GH deficiency, defined
Free T4 15.2pmol/L (9–22) as a peak GH response of <9mU/L (3ng/ml)
during an insulin tolerance test or a cross-
TSH 1.8mU/L (0.5–5) validated GH threshold in an equivalent test.

Prolactin 300mU/L (50–450) The patient has a perceived impairment of


Which one of the following would be the most quality of life (QoL), as demonstrated by a
appropriate management? reported score of at least 11 in the disease-
specific Quality of Life Assessment of Growth
1- Continue GH treatment indefinitely Hormone Deficiency in Adults (QoL-AGHDA)
2- Continue GH treatment only if receiving questionnaire.
treatment for other pituitary
The patient is already receiving treatment for
deficiencies
any other pituitary hormone deficiencies as
3- Stop GH for 2–3 months and reassess GH required.
axis via dynamic endocrine testing
4- Reduce dose of the GH Further reading
National Institute for Health and Clinical
5- Decision to continue GH left to patient’s Excellence. Human growth hormone
own choice (somatropin) in adults with growth
hormonedeficiency. Technology Appraisal 64.
London: National Institute for Health and
Answer & Comments Clinical Excellence, 2003.
3- Stop GH for 2–3 months and reassess GH
axis via dynamic endocrine testing Q 53

A majority of individuals who have idiopathic A 26-year-old woman is brought to the medical
isolated growth hormone (GH) deficiency in assessment unit with a 24-hour history of

36 | Dr. Khalid Yusuf (FB: Sohag Endocrine Group)


SCE Endocrinology (sce.practicaldiabetes.com), 2017

diarrhoea, vomiting and abdominal pain. She 2- Diabetic ketoacidosis


has an 8-year history of type 1 diabetes, which
3- Thyroid storm?
is usually well controlled. She also has
pernicious anaemia for which she is on lifelong 4- Hypertensive crisis caused by
B12 replacement. She is accompanied by her phaeochromocytoma
mother who tells you that her GP has been 5- Hypercalcaemic crisis
treating her with antibiotics for a presumed
chest infection, and that she has lost about
10kg in weight over the last few months. She Answer & Comments
does not smoke or drink alcohol. On 3- Thyroid storm?
examination, she is confused, disorientated,
flushed, agitated and jaundiced. She has a This patient is only mildly acidotic in the
temperature of 42°C, heart rate of 180 beats presence of hyperglycaemia. This would go
per minute irregularly irregular and blood against a diagnosis of diabetic ketoacidosis.
pressure of 160/70mmHg. Cardiovascular Hypertension with hypokalaemia, in contrast
examination reveals a third heart sound and to hypotension and hyperkalaemia, goes
pitting oedema of her lower limbs. Her chest is against significant adrenocortical insufficiency.
clear to auscultation and her abdomen Her calcium is not high enough to cause a
generally tender to palpation. It is also noted hypercalcaemic crisis.
that she has proximal muscle wasting and
weakness and all of her reflexes are brisk. Her past history of autoimmune disease makes
Urgent blood tests reveal: it more likely that she has a further
autoimmune disorder, rather than
Sodium 130mmol/L (normal range 132–145) phaeochromocyotoma. The history of weight
loss also suggests an underlying thyroid
Potassium 2.7mmol/L (3.5–5.5) disorder, with the thyroid storm possibly being
precipitated by a chest infection.
Bicarbonate 19mmol/L (20–26)
Thyroid storm is a life-threatening
Urea 18mmol/L (3.5–7)
exacerbation of thyrotoxicosis. Some series
Creatinine 155mmol/L (60–110) suggest a mortality rate of between 30 and
75%. Hyperpyrexia is a characteristic feature.
Corrected calcium 2.81mmol/L (2.1–2.78) The clinical picture is frequently clouded by a
secondary infection such as pneumonia or a
Bilirubin 62mmol/L (<25) viral infection. Death may be caused by cardiac
arrhythmia, congestive heart failure,
ALT 136U/L (7–55) hyperthermia or other unidentified factors.
The diagnosis of thyroid storm is made entirely
Alkaline phosphatase 205U/L (44–147) on clinical grounds. The results of thyroid

Glucose15mmol/L function tests will rarely be available soon


enough to make the diagnosis.
WCC 19x109/L
Propylthiouracil followed by a stable iodine
Haemoglobin 10.3d/dl preparation (e.g. Lugol’s iodine) is usually
given. Propranolol, intravenous fluids, steroids
MCV 102fl (80–100) and cooling are also often required. Treatment
What is the mostly likely diagnosis? of the precipitating cause is also essential.

1- Addisonian crisis Further reading

37 | Dr. Khalid Yusuf (FB: Sohag Endocrine Group)


SCE Endocrinology (sce.practicaldiabetes.com), 2017

Carroll R, Matfin G. Review. Endocrine and should always be withdrawn for at least 4–6
metabolic emergencies: thyroid storm. Ther weeks (6–8 weeks for spironolactone);
Adv Endocrinol Metab 2010;1:139–45. dihydropyridine calcium channel blockers, ACE
inhibitors, and angiotensin II receptor
Q 54 antagonists can potentially, but infrequently,
lead to false-negative results; in contrast, beta-
A 38-year-old man is referred to the endocrine blockers and central α2-agonists can cause
clinic for further investigation of hypertension false-positives. The direct renin inhibitor
that remains uncontrolled despite treatment aliskiren lowers plasma renin activity (PRA) but
with 3 different anti-hypertensive medications. raises direct renin concentration (DRC),
He is found to have a high aldosterone:renin resulting in false-positive ARR for renin
ratio. measured as PRA and false-negatives for renin
measured as DRC.
Which one of the following drugs is most likely
to have caused this? Further reading
1- Ramipril Funder JW, et al. Case detection, diagnosis, and
treatment of patients with primary
2- Atenolol aldosteronism: an Endocrine Society clinical
practice guideline. J Clin Endocrinol Metab
3- Nifedipine
2008;93:3266–81.
4- Amlodipine Mulatero P, et al. Diagnosis and treatment of
5- Doxazosin primary aldosteronism. Rev Endocrinol Metab
Disord 2011;12:3–9.

Answer & Comments


Q 55
2- Atenolol
A 38-year-old woman of Indian origin, who has
An aldosterone:renin ratio (ARR) is widely used a family history of diabetes (her sister has type
in screening for primary aldosteronism. 2 diabetes mellitus), books into the antenatal
clinic when 8 weeks’ pregnant. This is her
This screening test should be performed in all second pregnancy, and her previous child is a
patients with: healthy baby girl who had a birth weight of
4.6kg. Her BMI is 33kg/m2.
Resistance to conventional antihypertensives
(3 agents or more). Which one of the following is not an indication
for her to be considered for screening for
Hypertension associated with hypokalaemia. gestational diabetes mellitus by oral glucose
tolerance test?
Hypertension developing before the age of
40 years. 1- Her previous baby’s birth weight
2- Her ethnicity
Patients with adrenal masses.
3- Her BMI
False-positive and false-negative results can 4- Her age
occur if the test is not performed in controlled
circumstances. 5- Her positive family history of type 2
diabetes
Several factors affect ARR, the most important
being antihypertensive therapy; Answer & Comments
mineralocorticoid receptor antagonists and
diuretics lead to false-negative results and thus 4- Her age

38 | Dr. Khalid Yusuf (FB: Sohag Endocrine Group)


SCE Endocrinology (sce.practicaldiabetes.com), 2017

Indications for screening for gestational 2- Discontinue driving pending satisfactory


diabetes mellitus are a BMI >30kg/m2, a medical review
previous macrosomic baby (>4.5kg), previous
3- Discontinue driving for 6 months
gestational diabetes, first degree relative with
diabetes, and ethnic origin with a high 4- Discontinue driving for 1 year
prevalence of diabetes mellitus (e.g. South 5- Discontinue driving permanently
Asian). Screening should be performed using
the 2-hour 75g oral glucose tolerance test
(OGTT). Answer & Comments

2- Discontinue driving pending satisfactory


Women who have had gestational diabetes in
medical review
a previous pregnancy should be offered early
self-monitoring of blood glucose or OGTT at This man is experiencing frequent disabling
16–18 weeks, and a further OGTT at 28 weeks hypoglycaemia and has also developed
if the results are normal. Women with any of impaired awareness of hypoglycaemia. He
the other risk factors for gestational diabetes should be advised to cease driving immediately
should be offered an OGTT at 24–28 weeks. and be told that he has a legal obligation to
inform the DVLA of the episodes of
Further reading hypoglycaemia and lack of warning signs.
National Institute for Health and Clinical
Excellence. Diabetes in pregnancy:
Relaxing glycaemic control, carbohydrate
management of diabetes and its complications
counting and use of a basal bolus regimen are
from preconception to the postnatal period.
Clinical Guideline 63. NICE, 2008. all manoeuvres that could be considered to
help reduce hypoglycaemia and restore
warning signs.
Q 56
Most primary care trusts would not fund pump
A 42-year-old man with a 19-year history of therapy until a basal bolus regimen has been
type 1 diabetes is reviewed in the diabetes tried and the patient has undergone DAFNE or
clinic. He works as a marketing representative. equivalent training in insulin pump therapy.
He has background retinopathy found on eye Screening for Addison’s disease and coeliac
screening, but no other complications. His disease should also be considered in these
current medication includes Humalog Mix 25 cases if it has not already been done.
46 units am and 38 units pm. On physical
examination, his BMI is 26kg/m2, blood Driving should only be resumed when an
pressure is 125/75mmHg, and his visual acuity appropriate and satisfactory medical review
is 6/9 in the right eye and 6/6 in the left eye. has been conducted demonstrating that
His most recent HbA1c is 7.9% (63mmol/mol). adequate resolution of hypoglycaemia and
During the consultation he reveals that he has hypoglycaemia awareness has been achieved
been troubled with recurrent hypoglycaemia, according to DVLA guidelines.
and is concerned as there has been a reduction
in his hypoglycaemia warning symptoms. He Further reading
has needed third-party assistance to treat his DVLA. Information for Health Professionals.
hypoglycaemia at times, and has had www.dft.gov.uk/dvla/medical/medical_profe
paramedic assistance twice in the last 3 ssionals.aspx.
months.
What advice should be given to this man as Q 57
regards driving?
A 48-year-old man with insulin-treated type 2
1- Discontinue driving for 1 month diabetes is admitted electively under the care

39 | Dr. Khalid Yusuf (FB: Sohag Endocrine Group)


SCE Endocrinology (sce.practicaldiabetes.com), 2017

of the surgical team for a hernia repair. He is procedures: improving standards. April 2011.
currently taking Novorapid 20 units three times www.diabetes.nhs.uk/publications_and_reso
a day and insulin glargine 36 units daily. His urces/reports_and_guidance [accessed
HbA1c is 8.0% (64mmol/mol) and a capillary 8.6.2011].
blood glucose value is 11mmol/L.
Q 58
Which one of the following would be an
indication to start him on a variable rate
An 18-year-old male student was referred for
intravenous insulin infusion?
investigation of hypertension. He lived with his
1- The fact that he is taking insulin mother who had been divorced for many years
treatment and as such there was no information about his
father. He was initially noted to have high
2- HbA1c of 8.0%
blood pressure at the age of 15 years old, but
3- Capillary blood glucose value of was not investigated at this time as he was lost
11mmol/L to follow up after a house move and
4- If he is anticipated to have a long consequent change in address. Initial
starvation period (i.e. 2 or more missed biochemistry results show:
meals)
Sodium 142mmol/L (normal range 135–145)
5- Hypoglycaemia during the previous
week Potassium 4.0mmol/L (3.5–5.5)

Urea 6.5mmol/L (2.5–6.7)


Answer & Comments

4- If he is anticipated to have a long Creatinine 90µmol/L (90–118)


starvation period (i.e. 2 or more missed
meals) Plasma renin activity 0.3nmol/L (0.5–2.2)

NHS Diabetes guidance on the management of Aldosterone 782pmol/L (100–500)


diabetes in surgery and elective procedures
His father is contacted (with the patient’s
recommends that the term ‘variable rate
permission) to seek a further family history and
intravenous insulin infusion’ (VRIII) should
he reveals that he has had a 20-year history of
replace the term ‘sliding scale’. A VRIII would
hypertension complicated by a haemorrhagic
typically consist of 50 units of soluble insulin
stroke. Further investigations confirm a
such as Actrapid in 50ml 0.9% saline which is
diagnosis of gluco-corticoid-remediable
infused alongside the substrate recommended
aldosteronism (GRA).
in the guidance of 0.45% saline, 5% dextrose
and 0.15% or 0.3% potassium chloride. The mode of inheritance of this condition is:

The guidance suggests that patients with 1- Autosomal dominant


decompensated diabetes or those expected to 2- Autosomal recessive
miss more than 1 meal should have a VRIII,
3- X-linked
whereas patients with a planned short
starvation period (expected not to 4- Co-dominant
miss more than 1 missed meal in total) should 5- Maternal mitochondrial inheritance
be managed by modification of their usual
diabetes medication, avoiding a VRIII
wherever possible. Answer & Comments
Further reading
1- Autosomal dominant
NHS Diabetes. Management of adults with
diabetes undergoing surgery and elective

40 |
SCE Endocrinology (sce.practicaldiabetes.com), 2017

Glucocorticoid-remediable aldosteronism 5- Repeat ultrasound in 6 months ± repeat


(GRA) is inherited as an autosomal dominant FNAC
trait. A single copy of the abnormal gene is
sufficient to cause the disease. Answer & Comments
The mutation in patients with GRA is fusion of 5- Repeat ultrasound in 6 months ± repeat
the promoter region of the gene for CYP11B1 FNAC
and the coding sequences of CYP11B2. These
patients are biochemically unique with This girl has benign fine needle aspiration for
markedly increased levels of 18-oxocortisol cytology (FNAC). Thyroid nodules diagnosed as
and 18-hydroxycortisol. The plasma potassium benign require follow up because of a low, but
concentration is normal in more than one-half not negligible, false-negative rate of up to 5%
of cases of GRA in contrast to the hypokalaemia with FNAC. A repeat ultrasound and thyroid
frequently seen in primary aldosteronism. stimulating hormone (TSH) measurement in 6–
12 months’ time is recommended, with some
Treatment with physiologic doses of a authorities recommending routine repeat
glucocorticoid will correct the overproduction aspiration. Repeat aspiration under ultrasound
of aldosterone by sup- pressing ACTH. guidance is recommended if a nodule
significantly enlarges, if a cyst reappears, or in
Further reading case of suspicious clinical or ultrasound
Funder JW, et al. Case detection, diagnosis, and changes.
treatment of patients with primary
aldosteronism: an Endocrine Society clinical Ultrasound features suggestive of malignancy
practice guideline. J Clin Endocrinol Metab include nodule hypoechogenicity compared to
2008;93:3266–81.
the normal thyroid parenchyma, increased
intranodular vascularity, irregular infiltrative
Q 59 margins, and the presence of
microcalcifications, an absent halo, and a
An 18-year-old girl is referred by her GP when shape taller than the width measured in the
she was found to have a thyroid nodule. The transverse dimension. No single sonographic
nodule was spotted incidentally by her mother. feature or combinations of features is
The girl is otherwise fit and well. She has no adequately sensitive or specific to identify all
family history of thyroid disorders. On malignant nodules. A pure cystic nodule,
examination, she has a small nodule on the left although rare (<2% of all nodules), is highly
side of her thyroid. No obvious cervical unlikely to be malignant.
lymphadenopathy was noted. Her serum TSH
was 1.2mU/L. Ultrasound scanning reveals a In non-iodine deficient areas, 123I scanning for
predominantly cystic nodule, which is 0.8cm in thyroid nodules is indicated only if TSH is
length and 1cm in width. She undergoes fine suppressed to detect hot nodules, which are
needle aspiration for cytology (FNAC) and the unlikely to be malignant, and cytological
cytology is reported as THY2. examination is not necessary. Otherwise,
nodules appearing in patients with Graves’
What is the next most appropriate step in the disease or Hashimoto’s thyroiditis should be
management of this patient? managed in the same way as that in any other
1- Reassure and discharge patients.
2- Refer for thyroid surgery CT and MRI scanning are not routinely used in
3- Arrange for 123I scanning and refer for the initial evaluation of nodular thyroid disease
surgery if the nodule is cold as they are rarely diagnostic for malignant
disease except in advanced cases.
4- Arrange MRI scan of the neck

41 | Dr. Khalid Yusuf (FB: Sohag Endocrine Group)


SCE Endocrinology (sce.practicaldiabetes.com), 2017

Further reading Q 61
British Thyroid Association & Royal College of
Physicians. Guidelines for the management of A 62-year-old woman with a 2-year history of
thyroid cancer, 2nd edn. Report of the Thyroid
type 2 diabetes attends clinic. Her body mass
Cancer Guidelines Update Group. London:
index is 32kg/m2, and she is currently treated
Royal College of Physicians, 2007.
with metformin 500mg tds. She has previously
been unable to tolerate higher doses of
Q 60 metformin (including modified release). She
has been attending weight watchers for 6
A 34-year-old woman presented to the months and has lost around 1kg in weight. Her
endocrine clinic after gaining 11kg in weight HbA1c is 6.7% (50mmol/mol).
over 6 months. She also had severe hirsutism,
According to NICE guidance, what would be the
acne, ankle oedema, polydipsia, nocturia, back
pain, pigmentation, poor libido and correct next step in her management?
oligomenorrhoea. She had stopped the 1- Continue lifestyle measures
combined oral contraceptive pill 2 months
previously. She did not smoke and only drank 2- Start sulphonylurea
alcohol socially. On physical examination, she 3- Start sitagliptin
was grossly Cushingoid with florid clinical
4- Start pioglitazone
manifestations. Subsequent investigations
confirm the diagnosis of pituitary-driven 5- Start orlistat
Cushing’s disease. She is pretreated with
adrenalytic therapy using metyrapone prior to Answer & Comments
surgery.
2- Start sulphonylurea
Which enzyme is inhibited bymetyrapone?
1- 11 beta-hydroxylase In practice, many clinicians would not alter this
woman’s treatment. However, the question
2- 3 beta-hydroxysteroid dehydrogenase
tests knowledge of established guidance.
3- 21 hydroxylase Sulphonylureas are second line in NICE
4- 17 alpha-hydroxylase guidance (CG66 and CG87) unless they have a
history of hypoglycaemia, have a risk of
5- 17, 20-desmolase hypoglycaemia, or the drugs are
contraindicated or not tolerated. NICE
Answer & Comments guidance has differential thresholds for
titration. A unique feature of the NICE guidance
1- 11 beta-hydroxylase are the differential thresholds for addition and
titration of glucose lowering agents, i.e. HbA1c
Metyrapone can be used as short-term ≥6.5% (48mmol/mol) for first or second line
treatment for Cushing’s syndrome pending agents and HbA1c ≥7.5% (58mmol/mol) for
definitive treatment. Metyrapone blocks third line agents. This is based on cost
cortisol synthesis by inhibiting 11 beta- effectiveness – glucose lowering with
hydroxylase. This blockade can be measured by inexpensive older agents (such as metformin
the urinary increase of the metabolites of and sulphonylureas) being cost effective to
cortisol precursors in the urine (17- initiate and titrate at lower HbA1c levels.
hydroxycorticosteroids [17-OHCS] and 17-
ketogenic steroids [17-KGS]). Further reading
National Institute for Health and Clinical
Further reading Excellence. Type 2 diabetes: newer agents.
Tritos NA, et al. Management of Cushing disease. NICE short clinical guideline 87. NICE, May
Nat Rev Endocrinol 2011;7:279–89. 2009.

42 | Dr. Khalid Yusuf (FB: Sohag Endocrine Group)


SCE Endocrinology (sce.practicaldiabetes.com), 2017

Q 62 requirement for insulin among LADA patients


when compared to other patients who have
A 48-year-old South Asian man was diagnosed type 2 diabetes. There are some clues that can
to have diabetes during a routine health check- give rise to a clinical suspicion of LADA. These
up done at his general practitioner’s surgery. include an absence of metabolic syndrome
He did not complain of any symptoms at the features, uncontrolled hyperglycaemia despite
time of diagnosis and his HbA1c at diagnosis using oral agents, and evidence of autoimmune
was 7.9% (63mmol/mol). His mother had diseases (such as Graves’ disease or pernicious
developed type 2 diabetes at the age of 60. He anaemia). Determining the presence of LADA is
was referred to a structured diabetes achieved by examining the presence of
education programme, which he attended, and elevated levels of pancreatic autoantibodies
was also commenced on metformin therapy. among patients who have recently been
There was progressive intensification of oral diagnosed with diabetes but do not require
therapy and within 6 months he was taking insulin. A GAD antibody test can measure the
maximum doses of metformin and sulphonyl presence of these autoantibodies. These
urea. Despite this, his HbA1c at 6 months was antibodies can identify LADA, and can also
11.2% (99mmol/mol). predict the rate of progression towards insulin
dependency.
The most likely diagnosis in this case would be:
1- HNF1? mutation Further reading
Naik RG, et al. Latent autoimmune diabetes in
2- HNF1? mutation adults. JCEM 2009;94:4635–44.
3- Latent autoimmune diabetes in adults
(LADA) Q 63
4- Type 2 diabetes mellitus
An 84-year-old woman with type 2 diabetes
5- Glucokinase deficiency
mellitus for 10 years attends clinic. She has
poor glycaemic control despite taking maximal
Answer & Comments doses of metformin and gliclazide. She lives
alone and is frail. She has some osmotic
3- Latent autoimmune diabetes in adults symptoms. She is commenced on insulin
(LADA) glargine to be administered daily by district
nurses.
Latent autoimmune diabetes of adulthood
(LADA) is often mistaken for type 2 diabetes Which of the following options best describes
despite not exhibiting all the classic symptoms the amino acid modifications seen in insulin
associated with this condition. Patients with glargine?
LADA may lack some of the typical features of
type 2 diabetes. These could include age, 1- Glycine is at position A21, 2 arginines are
obesity and difficulty in achieving glycaemic added to the B chain
control using standard oral hypoglycaemic 2- Aspartic acid is at position B28
agents. If these are lacking, then it is quite
3- Glycine is at position B28
possible that the patient has LADA. The
presence of some of the usual immune markers 4- Aspartic acid is at position A21
common to type 1 diabetes are present in
5- Proline is at position A21
LADA, yet in its early stages it does not require
insulin.
Answer & Comments
Patients in the early stages of LADA may also
1- Glycine is at position A21, 2 arginines are
lack the presence of ketonaemia. However,
added to the B chain
there may be a more rapid progression to

43 | Dr. Khalid Yusuf (FB: Sohag Endocrine Group)


SCE Endocrinology (sce.practicaldiabetes.com), 2017

The clue is in the name! ‘Glargine’ is an 2- Vitamin B1, B6, B12


amalgam of glycine and arginine. Insulin
3- Vitamin C
glargine is a long-acting insulin analogue.
Insulin aspart is a shortacting insulin analogue 4- Vitamin D
which has an aspartic acid residue at position 5- Copper and zinc
B28. Studies of insulin glargine in type 2
diabetes have shown that the only benefit is a
small reduction in non-severe nocturnal Answer & Comments
hypoglycaemia. It is for this reason that NICE 1- Vitamin A
guidance recommends human isophane insulin
(NPH) as first line treatment, and that long- Biliopancreatic diversion (BPD) is a
acting analogue insulin use should be restricted malabsorptive bariatric procedure that limits
to those who: carbohydrate, protein and fat absorption to a
relatively short segment of small intestine as
• Need assistance from a carer or health well as decreasing the gastric reservoir size.
care professional to inject insulin (as in This procedure can be associated with
this question). longterm serious side effects including
significant protein calorie malnutrition and
• Have recurrent symptomatic
deficiencies of fat-soluble vitamins. Vitamin A
hypoglycaemic episodes.
being a fat soluble vitamin could be poorly
• Cannot use the device to inject NPH absorbed following the procedure and its
insulin. deficiency is associated with night blindness.

Further reading Further reading


Chea T, et al. Vitamin A deficiency in patients with
Evans M, et al. A review of modern insulin
a remote history of intestinal surgery. Br J
analogue pharmacokinetic and
Ophthalmol 2006;90:955–6.
pharmacodynamic profiles in type 2 diabetes:
improvements and limitations. Diabetes Obes
Metab 2011;13:677–84. Q 65
National Institute for Health and Clinical
Excellence. Type 2 diabetes: newer agents. A 32-year-old woman presents with tiredness,
NICE short clinical guideline 87. NICE, May headache and lethargy, 6 weeks post-partum.
2009. She had an uncomplicated delivery and
delivered a healthy baby. She had no significant
Q 64 past medical history and 2 previous
pregnancies were both uneventful. She has
A 50-year-old man is admitted following a road continued to breast-feed her baby since birth.
traffic accident. He has a past medical history In view of tiredness and headaches she had
of morbid obesity for which he underwent a further investigations requested. These
biliopancreatic diversion and duodenal switch revealed:
procedure 5 years ago. He is not on any regular
medications or vitamin supplements as he did WBC – 8.5 x 109/L
not feel the necessity to continue taking them.
He admits to poor vision, especially in poorly lit Hb – 11.4g/dl
conditions and thinks that this could have
Sodium – 132mmol/L
contributed to the accident.
Which is the likely vitamin or micronutrient that Potassium – 5.0mmol/L
could be contributing to his poor vision?
C-reactive protein – 40mg/L (normal range<5)
1- Vitamin A

44 | Dr. Khalid Yusuf (FB: Sohag Endocrine Group)


SCE Endocrinology (sce.practicaldiabetes.com), 2017

ESR – 60mm/hr described, leading to adrenal insufficiency and


hypothyroidism. Posterior pituitary
Urea – 6.5mmol/L involvement is less common. MRI shows an
enhancing mass diffuse and homogeneous
Creatinine – 82µmol/L contrast enhancement of the anterior
pituitary, but enhancement may be delayed or
TSH – 0.86mU/L (normal range 0.5–5) even absent in the posterior pituitary area.
Free thyroxine – 6.2pmol/L (9–22)
Further reading
Laws ER, et al. Hypophysitis. Pituitary 2006;9:
Random cortisol – 125nmol/L
331–3. Bellastella A, et al. Lymphocytic
hypophysitis: a rare or underestimated
Prolactin – 3000mU/L (45–375) disease? Eur J Endocrinol 2003;149:363–76.

FSH – 1.2IU/L (2–10)


Q 66
LH – 1.6IU/L (2–10)
A 22-year-old woman presents with irregular
Oestradiol – 180pmol/L (130–800) periods, weight gain and problematic facial
hirsutism. She has a raised LH:FSH ratio, low
30-min cortisol post synacthen – 201nmol/L SHBG and normal testosterone levels. The rest
(>550) of her biochemistry, including 17-OHP,
androstenedione, DHEAS, thyroid functions,
An urgent MRI of the pituitary was done which prolactin and fasting blood glucose levels, are
revealed diffuse and homogenous contrast within normal limits. Based on these, she is
enhancement of the anterior pituitary with a diagnosed with polycystic ovarian syndrome.
convex upper border and a thickened stalk. She is keen to try eflornithine for hirsutism as
What is the likely diagnosis, given her clinical one of her colleagues has had good effect with
and biochemical picture? it for facial hair.

1- Macroprolactinoma What is the mechanism of action of


eflornithine?
2- Pituitary apoplexy
1- Ornithine decarboxylase inhibitor
3- Lymphocytic hypophysitis
2- GnRH analogue
4- Dysgerminoma
3- 5 alpha-reductase inhibitor
5- Pituitary incidentaloma
4- Third generation topical retinoid
Answer & Comments 5- Androgen receptor antagonist

3- Lymphocytic hypophysitis
Answer & Comments
This woman has secondary hypothyroidism
1- Ornithine decarboxylase inhibitor
and inadequate cortisol response to short
synacthen test. Her raised prolactin levels can Eflornithine is a topical cream that irreversibly
be attributed to breastfeeding. Lymphocytic inhibits the enzyme ornithine decarboxylase,
hypophysitis is an uncommon disorder that is which is involved in the cell division and
initially characterised by lymphocytic proliferation in the hair follicle. It is
infiltration and enlargement of the pituitary. It administered as a topical cream on the face
most often occurs in late pregnancy or the and its long-term use reduces new hair growth.
post-partum period. Preferential hypofunction It can be used as an adjunct to laser therapy for
of ACTH and TSH secreting cells has been

45 | Dr. Khalid Yusuf (FB: Sohag Endocrine Group)


SCE Endocrinology (sce.practicaldiabetes.com), 2017

facial hirsutism in women in the management recommended first-line drug therapy for
of mild facial hirsutism. Treatment should be hypertension in type 2 diabetes and, as he is of
discontinued if there is no benefit at 4 months. Afro-Caribbean origin, addition of a diuretic or
calcium channel antagonist is recommended
Further reading due to the prevalence of low renin
Balfour JA, McClennan K. Topical eflornithine. Am hypertension in this population.
J Clin Dermatol 2001;2:197–201.
NICE guidance suggests that, in patients with
Q 67 kidney, eye or cerebrovascular damage whose
blood pressure is above the target of
130/80mmHg, a re-evaluation of blood
A 59-year-old man, of Afro-Caribbean descent,
pressure ought to be performed in 2 months’
with type 2 diabetes attends clinic. Recent
time.
digital eye screening has found evidence of
microaneurysms and he has previously been
Further reading
noted to have a mildly raised ACR but this has
National Collaborating Centre for Chronic
never been repeated. He is taking Humulin M3
Conditions. Type 2 diabetes: national clinical
twice daily and has an HbA1c of 7.9% guideline for management in primary and
(63mmol/mol). He is taking no other secondary care (update). London: Royal
medication. His blood pressure is measured College of Physicians, 2008.
and found to be 144/82mmHg.
Which of the following is the correct treatment Q 68
according to NICE guidelines ?
Which one of the following statements
1- Start ACE inhibitor and recheck blood regarding the insulin receptor is true?
pressure within 1 month
1- It is a 600kDa glycoprotein
2- Start ACE inhibitor and recheck blood
pressure within 2 months 2- It is coded for on the long arm of
chromosome 19
3- Start ACE inhibitor plus calcium channel
antagonist and recheck blood pressure 3- It comprises 2 alpha- and 2 beta-
in 1 month subunits linked by disulphide bonds

4- Start ACE inhibitor plus calcium channel 4- The alpha-subunit is intracellular


antagonist and recheck blood pressure 5- The beta-subunit is extracellular
in 2 months
5- Start ACE inhibitor plus thiazide diuretic Answer & Comments
and recheck blood pressure at next
annual review appointment 3- It comprises 2 alpha- and 2 beta-
subunits linked by disulphide bonds
Answer & Comments The insulin receptor is a 400kDa glycoprotein
4- Start ACE inhibitor plus calcium channel and belongs to the class of tyrosine kinase
antagonist and recheck blood pressure receptors. It is coded for on the short arm of
in 2 months chromosome 19 and consists of 2 alpha- and 2
beta-subunits linked by disulphide bonds. It is
This man has clear evidence of retinopathy, a transmembrane protein, the alpha-subunits
together with some suggestion of being mainly extracellular while the beta-
microalbuminuria (although not confirmed). subunits straddle the cell membrane.
Therefore his target blood pressure is a value
below 130/80mmHg. ACE inhibitors are the Further reading

46 | Dr. Khalid Yusuf (FB: Sohag Endocrine Group)


SCE Endocrinology (sce.practicaldiabetes.com), 2017

Holt RIG, et al. (eds). Textbook of diabetes, 4th acute rise in glucagon concentrations, with
edn. Chapter 7. Wiley-Blackwell, 2010. other counter-regulatory hormones being little
affected initially. Basal hepatic glucose
Q 69 production in DKA is approximately twice that
in stable diabetic patients, with a much smaller
A 20-year-old woman, with a 7-year history of decrease in peripheral glucose utilisation. Lack
type 1 diabetes, is admitted to A&E with of insulin causes direct stimulation of carnitine
abdominal pain, nausea and vomiting. Her acyl-CoA transferase 1 and high glucagon levels
current insulin regimen comprises Levemir and inhibit the formation of malonyl-CoA, further
NovoRapid via a basal bolus regimen. On stimulating transferase activity. Consequently,
examination she is dehydrated and breathing there is a switch from non-esterified fatty acid
deeply, her blood pressure is 98/66mmHg and (NEFA) re-esterification to oxidation. Beta-
her pulse is 105bpm. Investigations reveal a hydroxybutyrate is produced in excess and is
plasma glucose of 21.2mmol/L, haemoglobin subsequently metabolised to acetoacetate
of 15.9g/dl (12–16), and white cell count of which is detected in urine ketone analysis by
21.1x109 (4–11x109). Arterial blood gas dip stick testing. Glucagon levels also increase
analysis reveals pH of 6.9 (7.35–7.45), pO2 of levels of carnitine, further enhancing fatty acid
12.1kPa (9.5–13) and a pCO2 of 3.2kPa (4.7–6). oxidation. Cortisol and catecholamines directly
She is noted to be markedly ketonuric. stimulate lipolysis. Glucagon is of particular
importance as ketone body levels rise early,
Which of the following is a feature of the with a continuous increase over 10–12 hours.
pathophysiology of diabetic ketoacidosis? Acidaemia combined with hyperglycaemia and
1- Acetoacetate is metabolised to beta- hyperglucagonaemia promote intracellular loss
hydroxybutyrate of potassium. The final serum potassium
concentration largely depends on the rate of
2- Hyperglucagonaemia is the primary urinary potassium loss. Platelet secretory
cause of intracellular potassium loss activity is often increased in DKA, but
3- Hepatic glucose production reaches a aggregation decreased. White cell count and
plateau within 4 hours of insulin neutrophil count are also commonly raised and
withdrawal correlate with ketone body levels, so do not
necessarily imply underlying infection.
4- Hepatic glucose production rises
gradually over the first 6–8 hours Further reading
following insulin withdrawal Joint British Diabetes Societies Inpatient Care
5- Hyperglycaemia together with cortisol Group. The Management of Diabetic
and catecholamine excess is the main Ketoacidosis in Adults. March 2010.
cause of ketone body formation Holt RIG, et al. (eds). Textbook of diabetes, 4th
edn. Chapter 34. Wiley-Blackwell, 2010.

Answer & Comments


Q 70
3- Hepatic glucose production reaches a
plateau within 4 hours of insulin Which among the following individuals has
withdrawal definitive familial hypercholesterolaemia as per
the Simon Broome criteria?
Diabetic ketacidosis (DKA) is a state of acute
1- A man with a total cholesterol of
metabolic disturbance. Hepatic glucose
8.0mmol/L, but no evidence of tendon
production rises rapidly over the first 2–4 hours
xanthomata. However, his grandfather
following absolute or severe relative insulin
had tendon xanthomata
deficiency, reaching a plateau after around 4
hours. These changes correlate with the initial

47 | Dr. Khalid Yusuf (FB: Sohag Endocrine Group)


SCE Endocrinology (sce.practicaldiabetes.com), 2017

2- A woman with a total cholesterol of A diagnosis of possible familial


7.5mmol/L and a family history of hypercholesterolaemia requires:
myocardial infarction in a second-degree
relative aged 55 years (a) A total cholesterol above 7.5mmol/L or LDL
cholesterol above 4.9mmol/L in an adult, or
3- A man with a total cholesterol of
total cholesterol above 6.7mmol/L or LDL
7.5mmol/L and a family history of
cholesterol above 4mmol/L in a child aged
myocardial infarction in a first-degree
under 16.
relative aged 65 years
4- A woman with total cholesterol of PLUS ONE OF:
7.5mmol/L, and who has a 17-year-old
child with a total cholesterol of (b) Family history of myocardial infarction:
6.5mmol/L before 50 years in a second-degree relative or
below age 60 in a first-degree relative.
5- A man with a total cholesterol of
8mmol/L with a family history of OR:
myocardial infarction in a first-degree
relative aged 55 years, and who has a 17- (c) Family history of raised total cholesterol:
year-old child with a total cholesterol of above 7.5mmol/L in an adult first- or second-
6.5mmol/L degree relative or above 6.7mmol/L in a child
or sibling aged under 16 years.
Answer & Comments
Further reading
1- A man with a total cholesterol of DeMott K, et al. Clinical guidelines and evidence
8.0mmol/L, but no evidence of tendon review for familial hypercholesterolaemia: the
xanthomata. However, his grandfather identification and management of adults and
had tendon xanthomata children with familial hypercholesterolaemia.
London: National Collaborating Centre for
A diagnosis of definite familial Primary Care and Royal College of General
Practitioners, 2008.
hypercholesterolaemia using the Simon
Broome criteria is defined as:
Q 71
(a) A total cholesterol >6.7mmol/L or LDL
cholesterol above 4.0mmol/L in a child <16 A 26-year-old man with learning difficulties has
years of age, or total cholesterol >7.5mmol/L been found to have abnormal thyroid function
or LDL cholesterol above 4.9mmol/L in an adult tests. He is noted to have a strong family
(levels should be either pre-treatment or history of thyroid problems. On examination he
highest on treatment). was found to have a goitre, and audiological
assessment demonstrated some hearing
PLUS: impairment. He was clinically euthyroid. His
thyroid function tests showed a TSH of
(b) Tendon xanthomata in the patient, or in a 8.3mU/L (0.1–5.0), free T4 of 32pmol/L (11–23)
first-degree relative (parent, sibling, child), or and a free T3 of 7.2pmol/L (3.5–6.5).
in a second-degree relative (grandparent,
uncle, aunt). Which of the following would be the best test
to help establish the diagnosis in this case?
OR:
1- Measurement of alpha-subunit
(c) DNA-based evidence of an LDL receptor 2- Measurement of sex-hormone binding
mutation or familial defective apo B-100, or a globulin
PCSK9 mutation.
3- TSH receptor mutation analysis

48 | Dr. Khalid Yusuf (FB: Sohag Endocrine Group)


SCE Endocrinology (sce.practicaldiabetes.com), 2017

4- Thyroid hormone receptor mutation tissue transglutaminase antibodies for which


analysis she takes calcium and vitamin D supplements
and folic acid. She reports alopecia and pica in
5- MRI scan of pituitary gland
childhood, now resolved. Her identical twin has
a similar medical history and also has
Answer & Comments hypothyroidism. The patient states she is
intolerant of levothyroxine therapy and has
4- Thyroid hormone receptor mutation
taken various combinations of thyroxine and
analysis
liothyronine in the past. She has also tried
The most likely diagnosis in this scenario is that thyroid Armour extract.
of generalised resistance to thyroid hormone
Currently, she is prescribed liothyronine only,
(GRTH). The main differential diagnosis would
the dose of which was increased, one week
be a thyrotrophin secreting pituitary adenoma
prior to clinic, to 25mg twice daily. She appears
(TSH-oma). In selective pituitary resistance to
clinically euthyroid although you note she is
thyroid hormones, the patient exhibits definite
hirsute with excess facial hair. She is obese
features of thyrotoxicosis. About 90% of
(body mass index 34kg/m2) although reports
thyroid hormone resistance syndromes result
her weight is stable, and has no features of
from mutations in the gene coding TRb. The
cortisol excess – notably no myopathy, bruising
diagnosis can involve identifying such a
or livid striae.
mutation of the thyroid receptor, which is
present in approximately 85% of cases; Her investigations are as follows: TSH 9.0mU/L
however, lack of a mutation in a patient does (0.1–5.0) Free T4 8pmol/L (11–23); Free T3
not rule out this diagnosis. The condition is rare 6.5pmol/L (3.5–6.5) Prolactin 500mU/L (<450)
and occurs in 1 in 50?000 births. Common U&Es normal FSH 3U/L (normal ranges:
features of patients include goitre (most follicular 0.5–5; mid-cycle 8–15; luteal 2.8) LH
common), emotional disturbances, ear, nose 6.2U/L (normal ranges: follicular 3–12; mid-
and throat infections, language disabilities, cycle 20–80; luteal 3–16) Oestradiol 30pmol/L
auditory disorders, low body weight, cardiac (normal ranges: follicular 17–260; mid-cycle
abnormalities and subnormal intelligence 370–1470; luteal 180–1100) SHBG 22nmol/L
quotients. (22–126) MRI pituitary: enlarged 11x10mm
pituitary gland, no discrete adenoma.
Further reading
Refetoff S, Dumitrescu AM. Syndromes of reduced Which diagnosis is most consistent with these
sensitivity to thyroid hormone: genetic defects findings?
in hormone receptors, cell transporters and
deiodination. Best Pract Res Clin Endocrinol 1- Coeliac disease with malabsorption
Metab 2007;21:277–305. 2- Early pregnancy
Bottcher Y, et al. Thyroid hormone resistance
without mutations in thyroid hormone
3- Thyroid hormone resistance
receptor beta. Med Sci Monit 2007;13:CS67– 4- Pituitary hyperplasia secondary to long-
70. term hypothyroidism and poor
compliance
Q 72 5- Thyrotrophin secreting pituitary
adenoma (TSH-oma)
A 33-year-old woman of Afro-Caribbean origin
is referred to the endocrine clinic for further
Answer & Comments
investigation of hypothyroidism. She has a
previous medical history of iron deficiency 4- Pituitary hyperplasia secondary to long-
anaemia secondary to menorrhagia, dietary term hypothyroidism and poor
folate and vitamin D deficiency with negative compliance

49 | Dr. Khalid Yusuf (FB: Sohag Endocrine Group)


SCE Endocrinology (sce.practicaldiabetes.com), 2017

Pituitary hyperplasia may be associated with 3- Removable cast walker


hormonal excess but may also be seen in
4- Half shoes
patients with undiagnosed primary
hypothyroidism or with long-term non- 5- Any of the above
compliance with therapy. The elevated TSH
indicates that the patient is largely non- Answer & Comments
compliant but has taken her treatment prior to
testing, resulting in a normal free T3 5- Any of the above
measurement. The low normal free T4 reflects
the fact that she takes L-triiodothyronine (T3). Pressure off-loading is a standard part of
Pituitary hyperplasia is also a physiological diabetic foot ulcer care. A systematic evidence
response to pregnancy and lactation due to review of the various off-loading techniques
lactotroph hyperplasia. In this case, the was carried out as part of the development of
patient’s presentation is more consistent with the NICE ‘Inpatient management of diabetic
polycystic ovary syndrome (PCOS) than foot problems’ (CG119) clinical guideline and
pregnancy in view of her phenotypic concluded that no one technique is superior to
appearance, follicular range any other. The emphasis ought to be to ensure
gonadotrophins/oestrogen, and borderline off-loading is offered and also to encourage
low SHBG. A slight elevation of prolactin patient compliance. The guideline
(<2000mU/L) may be seen in PCOS. recommended that the clinical scenario, type
of wound and acquisition cost of the technique
TSH-omas are extremely rare pituitary should determine the choice of off-loading
macroadenomas and usually present with a technique.
diffuse goitre, typical signs and symptoms of
thyrotoxicosis and mass effects. Further reading
National Institute for Health and Clinical
Further reading Excellence. Diabetic foot problems: inpatient
Howarth E, et al. Pituitary hyperplasia. Pituitary management of diabetic foot problems. Short
1999;1:169–79. Clinical Guideline 119. NICE, May 2011.

Q 73 Q 74

A 67-year-old man with long-standing type 2 A 51-year-old man with recently identified type
diabetes and severe peripheral neuropathy is 2 diabetes, currently managed with lifestyle
admitted with deteriorating renal function modification alone, undergoes a mixed meal
following a bout of presumed gastroenteritis. tolerance test (ingestion of a liquid meal such
While in hospital it is noted that he has a new as Sustacal or Boost) with measurement of
left plantar ulcer. His renal function has glucose, insulin and glucagon profiles over a 4-
improved with rehydration and temporary hour period.
withdrawal of his nephrotoxic medication. Which of the following glucose, insulin and
Eventually he is ready to mobilise. He is glucagon profiles following a mixed meal
reviewed by the inpatient diabetic foot team tolerance test would you expect to occur in this
who feel that pressure off-loading should be
man compared with an age matched healthy
offered to aid healing of his plantar ulcer.
volunteer?
What is the preferred technique for off-
1- Increased plasma glucose, reduced early
loading?
phase insulin secretion, reduced
1- Total contact foot casting glucagon secretion
2- Custom made temporary footwear

50 | Dr. Khalid Yusuf (FB: Sohag Endocrine Group)


SCE Endocrinology (sce.practicaldiabetes.com), 2017

2- Increased plasma glucose, reduced early 4- GLUT-3 is expressed mostly in neurons


phase insulin secretion, elevated and placenta
glucagon secretion
5- GLUT-4 is stored as intracellular vesicles
3- Increased plasma glucose, exaggerated in the pancreatic beta cells
early phase insulin secretion, reduced
glucagon secretion Answer & Comments
4- Increased plasma glucose, delayed early
phase insulin secretion, no difference in 5- GLUT-4 is stored as intracellular vesicles
glucagon secretion in the pancreatic beta cells

5- Increased plasma glucose, no difference Glucose transport across plasma membranes


in insulin secretion, elevated glucagon occurs via glucose transporter proteins (GLUT).
secretion Several isoforms have been identified, each
with a specific role in glucose metabolism. The
Answer & Comments major transporter isoforms are GLUT-1 and
GLUT-4. GLUT-4 is the major transporter
2- Increased plasma glucose, reduced early facilitating insulin mediated glucose uptake in
phase insulin secretion, elevated tissues such as skeletal muscle and fat. GLUT-4
glucagon secretion is stored as intracellular vesicles in skeletal
muscle and adipose tissue. GLUT-1 is expressed
Beta and alpha cell dysfunction are at highest levels in erythrocytes and also in the
characteristic features of the pathophysiology endothelial cells of barrier tissues such as the
of type 2 diabetes. The earliest manifestation blood-brain barrier. GLUT-2 is the principal
of beta cell dysfunction occurs in the form of transporter for transfer of glucose between
reduced and delayed postprandial early phase liver and blood, and for renal glucose
insulin secretion. Alpha cell dysfunction is reabsorption.
manifest as elevated plasma glucagon levels,
compared to healthy subjects, which remain Further reading
inappropriately elevated following meal Holt RIG, et al. (eds). Textbook of diabetes. 4th
ingestion. edn. Chapter 7. Wiley-Blackwell, 2010.

Further reading Q 76
Holt RIG, et al. (eds). Textbook of diabetes. 4th
edn. Chapter 10. Wiley-Blackwell, 2010.
A 37-year-old female with morbid obesity
Del Prato S, et al. Phasic insulin release and
attends clinic. She has been non-compliant
metabolic regulation in type 2 diabetes.
with dietary therapy and as such has a history
Diabetes 2002;51(Suppl 1):S109–16. of multiple failed dietary attempts. She is
intolerant to orlistat. She enquires as to
Q 75 whether she might be considered a candidate
for bariatric surgery.
Which of the following statements is not true of
glucose transporter (GLUT) proteins? NICE criteria for bariatric surgery include all of
the following except:
1- GLUT-1 enables glucose uptake, which is
largely non-insulin mediated 1- BMI >40kg/m2 or >35kg/m2 plus other
significant disease that could be
2- GLUT-2 is expressed by pancreatic beta- improved by weight loss
cells, renal cells, small intestinal
epithelial cells and liver cells 2- As a first-line option if BMI >45kg/m2
and surgical intervention is deemed
3- GLUT-2 is a bidirectional transporter appropriate

51 | Dr. Khalid Yusuf (FB: Sohag Endocrine Group)


SCE Endocrinology (sce.practicaldiabetes.com), 2017

3- All appropriate non-surgical measures to National Institute for Health and Clinical
achieve or maintain adequate clinically Excellence. Obesity: guidance on the
beneficial weight loss for at least 6 prevention, identification, assessment and
months should have failed management of overweight and obesity in
adults and children. Clinical Guideline 43.
4- In the case of young people, they should NICE, December 2006.
have achieved or nearly achieved
physiological maturity Q 77
5- The patient should be generally fit for
anaesthesia and surgery A 66-year-old female with a long-standing
history of epilepsy, hypothyroidism, bipolar
Answer & Comments disorder and gastrooesophageal reflux disease
was referred to the endocrine clinic with
2- As a first-line option if BMI >45kg/m2 abnormal laboratory results which were
and surgical intervention is deemed discovered incidentally. She is asymptomatic.
appropriate Both her epilepsy and bipolar disorder are well
controlled on medication. She is currently
Current NICE obesity guidance (CG43) taking lithium, topiramate, omeprazole,
recommends bariatric surgery as a treatment thyroxine, strontium ranelate and furosemide.
option in adults who fulfil all of the following Laboratory investigations revealed: Full blood
criteria: count normal Sodium 142mmol/L (134–144)
Potassium 4.2mmol/L (3.5–5.2) Urea
• Have a BMI of 40kg/m2 or more, or between 11.5mmol/L (2.5–10.7) Creatinine 134?mol/L
35kg/m2 and 40kg/m2 and other significant (62–106) Calcium 2.90mmol/L (2.2–2.6)
disease (for example, type 2 diabetes or high Phosphate 0.7mmol/L (0.7–1.4) Parathormone
blood pressure) that could be improved if they (PTH) 8.2nmol/L (3–6)
lost weight.
Which one of her medications could cause
• All appropriate non-surgical measures have hyper-parathyroidism and hypercalcaemia?
been tried but have failed to achieve or 1- Lithium
maintain adequate, clinically beneficial
weight loss for at least 6 months. 2- Topiramate
3- Strontium
• The person has been receiving or will receive
intensive management in a specialist 4- Furosemide
obesity service. 5- Omeprazole
• The person is generally fit for anaesthesia and
surgery. Answer & Comments

1- Lithium
• The person commits to the need for long-
term follow up.
Patients receiving chronic lithium therapy may
develop mild hypercalcaemia. Lithium
Bariatric surgery is also recommended as a
decreases parathyroid gland sensitivity to
first-line option (instead of lifestyle
calcium, shifting the set point of the calcium-
interventions or drug treatment) for adults
PTH curve to the right. The hypercalcaemia
with a BMI of more than 50kg/m2 inwhom
usually, but not always, subsides when the
surgical intervention is considered
lithium is stopped. Lithium can also unmask
appropriate.
previously unrecognised mild
hyperparathyroidism.
Further reading

52 | Dr. Khalid Yusuf (FB: Sohag Endocrine Group)


SCE Endocrinology (sce.practicaldiabetes.com), 2017

Further reading Answer & Comments


Haden ST, et al. Alterations in parathyroid
dynamics in lithium treated subjects. J Clin 5- Trial of dopamine agonist therapy to
Endocrinol Metab 1997;82:2844–8. suppress lactation

Indications for treatment of hyperprolactin –


Q 78
aemia include restoration of gonadal function
and fertility or to induce tumour shrinkage
A 32-year-old woman is referred to the when associated with macroadenoma.
endocrine clinic by her GP. She is very
distressed by spontaneous galactorrhoea for Treatment may also be considered to alleviate
over 7 years, persistent since the birth of her distressing symptoms or prevent deleterious
first child. Triggers such as warm baths, crying effects of sex hormone deficiency such as
babies and cuddling her children stimulate a osteoporosis, although oral
let-down reflex. Despite this she has regular contraceptives/testosterone replacement may
menses, and has conceived twice without be more appropriate in this situation.
problems, breastfeeding both children. She
denies headache or visual field disturbance. However, there is some evidence to guide
She has a previous medical history of asthma management of galactorrhoea associated with
and chronic lower back pain, for which she normal serum prolactin levels, sometimes
takes no regular medication. She underwent referred to as ‘normoprolactinaemic
laparoscopic sterilisation 2 years ago and is a galactorrhoea’.
smoker of 20 cigarettes a day. On examination,
visual fields are full to confrontation and she is In this case, the patient is distressed by her
freely able to express milky fluid from both symptoms, and it would be appropriate to offer
breasts. She has some pale abdominal striae a trial of dopamine agonist therapy. It has been
consistent with previous pregnancy but there suggested that galactorrhoea in this situation
are no additional features on physical may be a consequence of intermittent
examination. Her investigations are as follows: prolactin hypersecretion.
Prolactin 71mU/L (45–375) FSH 11.4U/L (2–10)
LH 8.9U/L (2–10) Oestradiol 121pmol/L (130– The ‘high dose hook effect’ should be excluded
800) TSH 0.79mU/L (0.5–5) Free T4 11.4pmol/L when a pituitary macroadenoma is associated
(9–22) with a moderate rise in prolactin, to aid
differentiation between macroprolactinoma or
What is the next most appropriate step in her
other pituitary macroadenoma with stalk
management?
compression.
1- Exclusion of ‘high dose hook effect’ by
serial dilutions of serum sample Further reading
Melmed S, et al. Diagnosis and treatment of
2- PEG precipitation to assess for the hyperprolactinemia: an Endocrine Society
presence of macroprolactin clinical practice guideline. J Clin Endocrinol
3- MRI pituitary to exclude lesion Metab 2011;96:273–88.
compressing pituitary stalk Kapcala LP, et al. Twenty-four-hour prolactin
secretory patterns in women with
4- Treatment is not necessary as she does galactorrhea, normal menses, normal random
not desire further pregnancy prolacting levels and abnormal sellar
5- Trial of dopamine agonist therapy to tomograms. J Endocrinol Invest 1984;7:455–
60.
suppress lactation

53 | Dr. Khalid Yusuf (FB: Sohag Endocrine Group)


SCE Endocrinology (sce.practicaldiabetes.com), 2017

Q 79 Further reading
NHS Diabetes. The Hospital Management of
A 31-year-old man with type 1 diabetes is Hypoglycaemia in Adults with Diabetes
Mellitus, March 2010.
found collapsed after having a seizure outside
the hospital and is brought into the emergency
department. He is known to have a history of Q 80
alcohol excess with several presentations of
diabetic ketoacidosis. He is thin (BMI 18kg/m2) Women with type 2 diabetes in pregnancy are
and has multiple spider naevi across his chest. less likely than women with type 1 diabetes in
He is unrousable and his blood glucose is found pregnancy to have all of the following except
to be 1.5mmol/L on capillary glucose testing. for:
What should be given for the immediate 1- Contraceptive use in the 12 months prior
management of his hypoglycaemia? to pregnancy

1- 25ml of 50% dextrose intravenously 2- Retinal assessments or test for


albuminuria in the 12 months prior to
2- Glucagon 1mg intramuscularly pregnancy
3- 150ml of 20% dextrose intravenously 3- Increased antenatal evidence of
4- 150ml of 10% dextrose intravenously macrosomia
5- 75ml of 10% dextrose intravenously 4- Received postnatal contraception advice
5- Received antenatal assessment in the
Answer & Comments first trimester of pregnancy

4- 150ml of 10% dextrose intravenously


Answer & Comments
The NHS Diabetes guidance on the treatment
3- Increased antenatal evidence of
of hypoglycaemia in hospital settings would
macrosomia
usually recommend the use of glucagon 1mg
administered intramuscularly in a patient with The 2007 Confidential Enquiry into Maternal
type 1 diabetes who is admitted unconscious and Child Health (CEMACH) report found that
with hypoglycaemia. However, in this case it there was no difference in the proportion of
should be noted the patient has a history of women with type 1 or type 2 diabetes to have
chronic heavy alcohol intake and evidence of antenatal evidence of macrosomia, despite the
chronic liver disease. In such a scenario, it is fact that the greater proportion of women with
possible that glucagon would be less effective type 2 diabetes had a BMI >30kg/m2.
due to impaired hepatic gluconeogenesis.
Therefore, the best immediate treatment Further reading
according to the NHS Diabetes guidance would Confidential Enquiry into Maternal and Child
be 150ml of 10% dextrose. After this is Health (CEMACH). Diabetes in pregnancy: are
administered, capillary blood glucose we providing the best care? Findings of a
monitoring should be repeated 10–15 minutes national enquiry. February 2007. England,
later, and again 10–15 minutes after that; once Wales and Northern Ireland. London:
the blood glucose is CEMACH, 2007.

above 4.0mmol/L, a long-acting carbohydrate Q 81


should be given, e.g. two biscuits, one slice of
bread/toast,
A 52-year-old woman with a past medical
history of type 2 diabetes diagnosed in 2005,
200–300ml glass of milk (not soya) or a normal
hypertension and asthma was seen in the
meal if due (must contain carbohydrate).

54 | Dr. Khalid Yusuf (FB: Sohag Endocrine Group)


SCE Endocrinology (sce.practicaldiabetes.com), 2017

endocrine clinic. She has a 2-month history of Q 82


worsening polyuria and polydipsia, drinking an
average of 5–6L/day and passing similar A 69-year-old Caucasian woman with a history
amounts of urine. There was no history of of rheumatoid arthritis is diagnosed with
trauma, weight loss or systemic illness. Her osteoporosis. She is teetotal. Her T-scores on
medication history included Humulin M3 Dexa scan are -3.6 at the hip and -3.8 at the
twice-daily, ramipril, simvastatin and inhalers. spine. She is unable to tolerate alendronic acid.
Investigations included: Hb 10.2g/dl, platelets She is already on calcium and vitamin D
160x109/L, WCC 7.7x109/L, sodium supplements. She has not had any previous
140mmol/L, potassium 4.8mmol/L, urea fractures and she is not aware of any family
5mmol/L, creatinine 140?mol/L, HbA1c 8% history of osteoporosis.
(64mmol/mol) and cholesterol 5.3mmol/mol.
Ward test of urine revealed 2+ glucose, 1+ What is the next treatment option according to
protein, 1+ leukocytes, no ketones, specific NICE guidelines for primary prevention of
gravity 1.005. A 24-hour urine collection osteoporotic fragility fractures in
revealed a urine specific gravity of 1.004, postmenopausal women?
protein 0.10g/24 hours. 1- Oral etidronate or risedronate
What is the most likely cause of her symptoms? 2- Strontium ranelate
1- Poorly-controlled diabetes mellitus 3- Raloxifene
2- Worsening renal failure 4- Denosumab
3- Diabetes insipidus 5- Calcitonin
4- Urinary tract infection
5- Medication related Answer & Comments

1- Oral etidronate or risedronate


Answer & Comments
The NICE guidance for primary prevention of
3- Diabetes insipidus osteoporotic fragility fractures in
postmenopausal women recommends
This patient’s poor glycaemic control could risedronate or etidronate in women intolerant
certainly be contributing to her symptoms; to alendronate, provided they have a
however, the abnormal urine specific gravity combination of T-score, age and number of
should not be overlooked. independent clinical risk factors for fracture, as
indicated in Table 1.
In poorly-controlled diabetes mellitus, the
urine specific gravity should exhibit a mild or Independent risk factors for fragility fractures
moderate increase. However, in cases where are parental history of hip fracture, alcohol
there is a low or low normal urinary specific intake of 4 units or more per day and
gravity, a diagnosis of diabetes insipidus must rheumatoid arthritis.
be considered.
Age No. of independent clinical
Further reading risk factors
Akarsu E, et al. The Value of Urine Specific Gravity
in Detecting Diabetes Insipidus in a Patient 0 1 2
with Uncontrolled Diabetes Mellitus: Urine
Specific Gravity in Differential Diagnosis. J Gen 65-69 NR* -3.0
Intern Med 2006;21:C1–C2. -3.5
70-74 -3.5 -3.0 -2.5
75 or older -3.0 -3.0 -2.5

55 | Dr. Khalid Yusuf (FB: Sohag Endocrine Group)


SCE Endocrinology (sce.practicaldiabetes.com), 2017

*Treatment with risedronate or etidronate is Answer & Comments


not recommended.
3- Mutation of the GNAS1 gene causes
Table 1. T-scores (SD) at (or below) which decreased activation of adenylate
risedronate or etidronate is recommended cyclase
when alendronate cannot be taken. (Source:
NICE Technology Appraisal Guidance 160 This patient has pseudohypoparathyroidism
[amended Jan 2010 and Jan 2011]. (PHP) as evidenced by the low calcium, high
phosphate and high PTH levels. This patient
Further reading also has Albright’s hereditary osteodystrophy,
National Institute for Health and Clinical thus making this condition PHP type 1a which
Excellence. Alendronate, etidronate, is maternally transmitted. Paternal
risedronate, raloxifene and strontium ranelate transmission causes only phenotypic features
for the primary prevention of osteoporotic of Albright hereditary osteodystrophy (AHO).
fragility fractures in postmenopausal women The condition occurs because of
(amended). October 2008 (amended January unresponsiveness to PTH in the target tissues
2010 and January 2011). Technology Appraisal (kidneys and bone) as a result of inactivating
Guidance 160. mutation of GNAS1. Because GNAS1 is
expressed dominantly from the maternal allele
Q 83 in the thyroid, pituitary and gonads, patients
with PHP 1a may show evidence of resistance
A 20-year-old man, with an adjusted calcium to TSH, LH, FSH and GnRH. Currently, this
level of 1.82mmol/L, was referred to the condition is treated with oral calcium salts and
endocrine clinic. He reported some episodes of vitamin D analogues.
muscle twitching and paraesthesia of his toes.
He was noted to be on the 60th centile for his Further reading
height and had a BMI of 33kg/m2. Further Spiegel AM, Weinstein LS. Inherited diseases
biochemistry tests revealed a phosphate of involving G proteins and G
1.8mmol/L and a parathyroid hormone (PTH) proteincoupledreceptors. Annu Rev Med
2004;55:27–39.
of 120pmol/L.
Which one of the following statements is true
Q 84
regarding this condition?
1- This condition is inherited by paternal A 28-year-old woman presented to the medical
transmission assessment unit with severe, sudden onset,
constant frontal headache associated with
2- The condition occurs as a result of a
photophobia and neck stiffness. She had not
defect in the PTH receptor
experienced any fever, rash, visual phenomena
3- Mutation of the GNAS1 gene causes or phonophobia, and had no previous history
decreased activation of adenylate of primary headache disorder. She was
cyclase nauseous, although not vomiting. On
4- There is usually resistance to thyroid examination she was alert and orientated with
hormones no localising neurological signs. She was noted
to have a prominent lower jaw, and on direct
5- PTH is a recognised treatment for this questioning reported that she had previously
condition been under the care of the oral surgeons for
neglected dentition, gross dental caries and
planned eventual correction of prognathism.
She also reported excessive sweating, an
increase in shoe size, and swelling of her hands
which prevented her from wearing her

56 | Dr. Khalid Yusuf (FB: Sohag Endocrine Group)


SCE Endocrinology (sce.practicaldiabetes.com), 2017

wedding and engagement rings. Baseline potentially lifesaving. Further management


investigations were as follows: Sodium should include pituitary imaging and liaison
134mmol/L Potassium 4.9mmol/L Urea with an experienced neurosurgical centre and,
2.8mmol/L Creatinine 73?mol/L Glucose although it is important to ascertain the nature
5.8mmol/L WCC 11.1x109/L Hb 13.2g/dl MCV of a pituitary adenoma, this may be delayed
96.6fL Platelets 172x109/L ECG: normal sinus until the acute presentation has been
rhythm CXR: no acute appropriately managed and the patient is
collapse/consolidation/pneumothorax CT stabilised.
brain: no acute haemorrhage, infarct or space
occupying lesion. No frontal sinus disease CSF: Further reading
glucose 3.2mmol/L, protein 0.26g/L, Gram Rajasekaran S, et al. UK guidelines for the
stain negative. management of pituitary apoplexy. Pituitary
Xanthochromia/oxyhaemoglobin not detected Apoplexy Guidelines Development Group:
BP 106/68mmHg May 2010. Clin Endocrinology 2011;74:9–20.

What is the most appropriate next step in her


Q 85
management?
1- Measurement of IGF-1 (insulin-like A 27-year-old man with a 5-year history of
growth factor 1) schizophrenia is admitted with recurrent
2- Oral glucose tolerance test with growth vomiting and upper abdominal pain. His mental
hormone illness was initially resistant to treatment, but
for the past 2 years has been stable on
3- MRI pituitary clozapine treatment. He is not known to have
4- Short synacthen test diabetes mellitus, but has been feeling thirsty
for several weeks and has lost about 10kg in
5- Consider IV hydrocortisone pending weight. He is drowsy but rousable, with a
assessment of hypothalamo-pituitary- Glasgow Coma Scale score (GCS) of 13/15,
adrenal axis pulse of 120bpm and blood pressure of
95/50mmHg. Initial laboratory testing shows a
Answer & Comments serum glucose of 61mmol/L, a serum
potassium of 6.8mmol/L, a serum bicarbonate
5- Consider IV hydrocortisone pending of 7mmol/L and a blood ketone level of
assessment of hypothalamo-pituitary- 7mmol/L.
adrenal axis
Which of the following is an indication for level
This patient has clinical features of acromegaly, 2 high dependency unit (HDU) admission?
although the diagnosis has not yet been
1- Bicarbonate level of 7mmol/L
confirmed. Headache may be the first
presentation of a pituitary tumour in up to 80% 2- Serum potassium level of 6.8mmol/L
of patients. In patients with known or 3- Blood ketone level of 7mmol/L
suspected pituitary disease presenting with
headache, with or without neuro-ophthalmic 4- GCS of 13/15
signs, a diagnosis of pituitary apoplexy should 5- Systolic blood pressure of 95mmHg
be considered. This may present in a subacute
fashion and mimic other neurological disorders
including subarachnoid haemorrhage, Answer & Comments
meningo-encephalitis and stroke. During the 3- Blood ketone level of 7mmol/L
acute presentation, administration of
intravenous hydrocortisone to a patient with The NHS Diabetes guidance on the treatment
features of adrenal insufficiency or of diabetic ketoacidosis recommends the
haemodynamic insufficiency may be

57 | Dr. Khalid Yusuf (FB: Sohag Endocrine Group)


SCE Endocrinology (sce.practicaldiabetes.com), 2017

following as indications for high dependency Answer & Comments


unit admission.
1- Topical steroids
• Blood ketones over 6mmol/L.
This is a classical description of necrobiosis
• Bicarbonate level below 5mmol/L. lipoidica which is seen in 0.3–1% of patients
with diabetes and is more common in females.
• Venous/arterial pH below 7.1. Around 40–60% of patients with necrobiosis
lipoidica have diabetes and the condition may
• Hypokalaemia on admission (under pre-date the development of abnormal blood
3.5mmol/L). glucose. Treatments with topical steroids,
injectable steroids, skin grafting and
• GCS less than 12 or abnormal AVPU scale. camouflage creams have been used as
therapies.
• Oxygen saturation below 92% on air
(assuming normal baseline respiratory Further reading
function). Holt RIG, et al. (eds). Textbook of diabetes. 4th
edn. Chapter 47. Wiley-Blackwell, 2010.
• Systolic BP below 90mmHg.

• Pulse over 100 or below 60bpm. Q 87

• Anion gap >16 (Anion gap = [Na+ + K+] – [Cl- A 67-year-old woman with type 2 diabetes for
+ HCO3-]). the past 15 years presents with painful feet,
worse at night time. She is currently taking
Further reading metformin 1g twice-daily and gliclazide 80mg
Joint British Diabetes Societies Inpatient Care twice-daily. Her HbA1c is 60mmol/mol (7.6%);
Group. The Management of Diabetic her BMI is 32kg/m2 and BP is 146/86mmHg. On
Ketoacidosis in Adults. March 2010. examination she has absent sensation to all
sensory modalities affecting her feet bilaterally
Q 86 extending to the mid-shins, associated with
absent ankle jerks and non-reactive plantars.
A 38-year-old man presents with ulcerated There is no motor deficit. She is prescribed
lesions over both shins. They are erythematous duloxetine to try and control the pain.
around the edges, but the centres are yellowed Regarding duloxetine which of the following is
and atrophic with signs of ulceration. He has a incorrect?
history of diabetes mellitus. He has recently
lost his job and has poor standards of personal 1- Uncontrolled hypertension is a
care. contraindication

Which of the following represents the 2- Creatinine clearance <50ml/min is a


treatment likely to be most beneficial for this contraindication
condition? 3- It is recommended as the first-line agent
for the treatment of painful diabetic
1- Topical steroids
neuropathy
2- Aspirin
4- It is metabolised by the cytochrome
3- Azathioprine P450
4- Flucloxacillin 5- Its mechanism of action involves
inhibition of both serotonin and
5- Compression bandaging
noradrenaline reuptake so that their

58 | Dr. Khalid Yusuf (FB: Sohag Endocrine Group)


SCE Endocrinology (sce.practicaldiabetes.com), 2017

concentration at the synapse is microprolactinoma compared to the scan she


increased had at diagnosis.
Which one of the following statements
Answer & Comments describes the best course for further
management?
2- Creatinine clearance <50ml/min is a
contraindication 1- Continue with the same dose of
cabergoline long term until the
Duloxetine is a serotonin and noradrenaline microprolactinoma disappears on
reuptake inhibitor, increasing the levels of imaging
these neurotransmitters at the synapse in the
CNS, which in turn function to modulate pain 2- Reduce the dose of cabergoline and
pathways. In the updated NICE guidelines on continue long-term treatment
painful diabetic neuropathy, duloxetine is the 3- Change cabergoline to quinagolide to
recommended first-line agent, with second- achieve further shrinkage of the
line combination therapy with duloxetine and microprolactinoma
pregabalin, or otherwise amitriptyline (with or
4- Withdraw cabergoline treatment and
without pregabalin). Duloxetine undergoes
monitor symptoms and prolactin level
metabolism in the liver by the cytochrome
regularly
P450 enzymes. It is contraindicated in
uncontrolled hypertension, creatinine 5- Start hormone replacement therapy to
clearance <30ml/min, uncontrolled reduce her risk of osteoporosis
angleclosure
Answer & Comments
glaucoma and should not be used in
combination with tricyclic antidepressants (e.g. 4- Withdraw cabergoline treatment and
amitriptyline) and monoamine oxidase monitor symptoms and prolactin level
inhibitors. regularly

Further reading Up to 60% of patients with microprolactinoma


National Institute for Health and Clinical can achieve long-term remission upon
Excellence. Neuropathic pain: the withdrawal of cabergoline treatment. The
pharmacological management of neuropathic nadir prolactin level and the size of the
pain in adults in non-specialist settings. Clinical microprolactinoma at withdrawal of
guideline 96. March 2010.
cabergoline treatment are useful indicators of
Harkin G, Hopkinson H. Duloxetine. Pract Diabetes the risk of recurrence of hyperprolactinaemia.
Int 2011;28:81–2. The best option for further management of this
patient is therefore to withdraw cabergoline
Q 88 treatment and monitor closely for recurrence
of symptoms of hyperprolactinaemia together
A 28-year-old woman with a history of with a significant rise in the level of serum
microprolactinoma has been treated with prolactin. A change in the dose or type of
0.5mg of cabergoline twice-weekly for the last dopamine agonist treatment is unnecessary
5 years. Her menstrual cycle is regular and she and HRT is not indicated in this patient as she
does not complain of galactorrhoea. Her serum is having regular menstruation.
prolactin level has been less than 100mU/L for
the last 3 consecutive 6-monthly follow-up Further reading
visits. Her last MR pituitary scan 1 year ago Colao A, et al. Withdrawal of long-term
showed (>50%) shrinkage of the size of the cabergoline therapy for tumoral and non
tumoral hyperprolactinemia. N Engl J Med
2003;349:2023–33.

59 | Dr. Khalid Yusuf (FB: Sohag Endocrine Group)


SCE Endocrinology (sce.practicaldiabetes.com), 2017

Q 89 Q 90

A 29-year-old woman was found to have A 32-year-old woman, previously fit and well, is
elevated calcium levels of 2.9mmol/L with a 32 weeks’ pregnant with her second child. She
parathyroid hormone level of 27pmol/L has a family history of type 2 diabetes and
(normal range 1.1–6.9). Her grandmother had atopy. Her mother also has primary
endometrial cancer. Her mother also had hypothyroidism. The patient develops upper
hyperparathyroidism and had to have a respiratory tract symptoms with fever, lethargy
hysterectomy at the age of 54 for endometrial and poor appetite. After 48 hours the fever
cancer. resolves but she remains tired and listless. She
visits her general practitioner for advice who
Which of the following inherited conditions is
performs some blood tests which show the
being described in this scenario?
following results:
1- Multiple endocrine neoplasia type 1
Sodium 135mmol/L
2- Multiple endocrine neoplasia type 2a
3- Multiple endocrine neoplasia type 2b Potassium 4.6mmol/L
4- Hyperparathyroidism-jaw tumour Urea 1.4mmol/L
syndrome (HPTJT)
5- Familial isolated primary Hb 11.2g/dl
hyperparathyroidism (FIHP)
WCC 4.2x109/L

Answer & Comments Mild eosinophilia


4- Hyperparathyroidism-jaw tumour Platelets 250x109/L
syndrome (HPTJT)
Creatinine 68µmol/L
Up to 10% of patients with primary HPT will
have a hereditary form, such as one of the TSH 1.9mU/L (normal range 0.5–5.5)
Multiple Endocrine Neoplasia (MED)
syndromes or HPT-JT. HPT-JT is an autosomal FT4 8.1pmol/L (normal range 10–23)
dominant condition. Parathyroid tumours
show high penetrance. Up to 30% of HPT-JT What is the next most appropriate step in the
patients have ossifying fibromas which affect patient’s management?
the maxilla and/or mandible. Benign and 1- Commence on levothyroxine 25µg once-
malignant uterine pathology is seen in up to daily
75% of women affected. Other tumours that
have been reported in this condition include 2- Measure prolactin, FSH, LH, oestradiol,
pancreatic adenocarcinomas, testicular mixed IGF-1 and random cortisol
germ cell tumours and Hurthle cell thyroid 3- Perform short synacthen test with ACTH
adenomas. measurement
4- Reassure patient that she has a viral
Further reading
illness
Marx SJ, et al. Hyperparathyroidism in hereditary
syndromes: special expressions and special 5- Refer for urgent endocrinology opinion
managements. J Bone Miner Res
2002:17:N37–43.

60 | Dr. Khalid Yusuf (FB: Sohag Endocrine Group)


SCE Endocrinology (sce.practicaldiabetes.com), 2017

Answer & Comments sulphonamides (including co-trimoxazole),


monoamine oxidase inhibitors, non-steroidal
4- Reassure patient that she has a viral anti-inflammatories (NSAIDs), probenecid,
illness somatostatin analogues and serotinin selective
reuptake inhibitors (SSRIs).
The normal range for free thyroid hormone can
be lower during pregnancy than in non- Nicotinic acid improves lipid profile but can
pregnant subjects depending on the assay in cause minor elevations in blood glucose,
use. This patient is clinically euthyroid, as potentially precipitating impaired glucose
confirmed by a normal TSH level. Her tolerance in individuals without diabetes or
symptoms are consistent with a viral illness and overt diabetes in individuals with pre-existing
not apoplexy, although this should be impaired glucose tolerance.
considered in the differential diagnosis should
she present with headache, vomiting or visual Further reading
disturbance. NHS Diabetes. The hospital management of
hypoglycaemia in adults with diabetes
Further reading mellitus. March 2010.
The Endocrine Society. Management of thyroid Florentin M, et al. Pleiotropic effects of nicotinic
dysfunction during pregnancy and acid: beyond high density lipoprotein
postpartum: an Endocrine Society clinical cholesterol activation. Curr Vasc Pharmacol
practice guideline. J Clin Endocrinol Metab 2011;9:385–400.
2007;92:S1–47.

Q 92
Q 91
A 50-year-old man was diagnosed as HIV
Hypoglycaemia is a common side effect of positive in 2007. His past medical history
insulin and sulphonylurea use. The concurrent includes diet-treated type 2 diabetes mellitus
use of certain medications with hypoglycaemic and hypertension. He is treated with a highly
agents (sulphonylureas or insulin) can further active antiretroviral therapy (HAART) regimen
increase the risk of hypoglycaemia. which includes ritonavir. On a routine clinic
All of the following can increase the risk of visit, blood tests showed HbA1c of
hypoglycaemia when used with an 53mmol/mol (7.0%), total cholesterol
antihypoglycaemic agent except: 19mmol/L, triglycerides 20mmol/L, eGFR >90.
He was commenced on simvastatin 40mg daily.
1- Quinine sulphate
As regards the management of his lipid profile,
2- Aspirin all of the following are true except:
3- Warfarin
1- Ritonavir is known to increase fasting
4- Fibrates triglycerides
5- Nicotinic acid derivatives 2- Atorvastatin at a low dose is the first
choice of statin in patients taking
Answer & Comments protease inhibitors
3- Simvastatin is preferred to pravastatin in
5- Nicotinic acid derivatives patients taking protease inhibitors
Concurrent use of certain drugs with 4- Fibrates will cause a reduction of
hypoglycaemic agents (insulin or triglycerides by 40%
sulphonylureas) can increase the risk of
precipitating hypoglycaemia; these include
warfarin, quinine, salicylates, fibrates,

61 | Dr. Khalid Yusuf (FB: Sohag Endocrine Group)


SCE Endocrinology (sce.practicaldiabetes.com), 2017

5- There is a high risk of developing 5- Give 3 units of Novorapid


myopathy and rhabdomyolysis with the subcutaneously and re-check capillary
current lipid lowering regimen blood glucose 1 hour later

Answer & Comments Answer & Comments

3- Simvastatin is preferred to pravastatin in 5- Give 3 units of Novorapid


patients taking protease inhibitors subcutaneously and re-check capillary
blood glucose 1 hour later
Protease inhibitors inhibit the metabolism of
most statins via inhibition of cytochrome P450 NHS Diabetes has produced comprehensive
3A4, and can significantly increase serum statin guidelines for the management of diabetes
levels, thus increasing the risk of toxicity, during surgery and elective procedures. Use of
including myopathy and rhabdomyolysis. The an intravenous variable rate insulin infusion is
degree to which statin metabolism is affected only recommended if there is to be a prolonged
by protease inhibitors varies according to the starvation period (defined as 2 or more missed
statin as well as the specific protease inhibitor. meals) or uncontrolled diabetes mellitus. The
Unfortunately, many specific interactions have recommendations on the management of
not been studied; however, simvastatin is preoperative hyperglycaemia suggest that
contraindicated with HIV protease inhibitors. target blood glucose should range between 6–
10mmol/L. Surgery should be cancelled if there
Further reading is heavy ketonuria/ketonaemia. In patients
Coyne K, et al. Simvastatin co-prescribed with with type 1 diabetes, a rapid insulin analogue
protease inhibitors despite dangerous drug can be administered at a dose of 1 unit to drop
interaction. Sex Transm Infect 2007;83:498–9. glucose by 3mmol/L.

Q 93 Further reading
NHS Diabetes. Management of adults with
diabetes undergoing surgery and elective
A 35-year-old woman with type 1 diabetes is
procedures: improving standards. April 2011.
admitted as a day case for laparoscopy. Her
glycaemic control is usually good with a recent
HbA1c of 52mmol/mol (6.9%). She usually Q 94
takes 6 units of Novorapid three times daily
and 16 units of Levemir daily. Prior to the A 16-year-old male presents via the accident
procedure, her capillary blood glucose is found and emergency department with new-onset
to be 20mmol/L – this is confirmed by seizures. His only history is of severe
repeating it. She feels thirsty but is otherwise depression for which he takes olanzapine and
well and blood ketones are negative. fluoxetine. Initial blood tests reveal
hypocalcaemia (adjusted calcium 1.40mmol/L)
What would be the best course of action?
and hyperphosphataemia (2.9mmol/L). He is
1- No action needed – continue with commenced on an intravenous calcium
procedure infusion. Additional oral calcium and vitamin D
supplementation are commenced when his
2- Postpone procedure until another day
conscious level improves sufficiently. Despite
3- Start an intravenous variable rate several days of this regimen, his adjusted
intravenous insulin infusion calcium remains low at 1.6mmol/L and he is
4- Give 8 units of Novorapid referred for an endocrinology opinion. At this
subcutaneously and re-check capillary point he appears pale and thin with some
blood glucose 1 hour later pitting of his nails attributed to psoriasis, and
enamel veneers on his front teeth due to

62 | Dr. Khalid Yusuf (FB: Sohag Endocrine Group)


SCE Endocrinology (sce.practicaldiabetes.com), 2017

discolouration in childhood. His secondary 1- Alkaline phosphatase isoenzymes and


sexual characteristics are normal. His mother 24-hour urine collection for urinary
reports that his growth and development were calcium measurement
normal throughout childhood, and that mild
2- Measurement of vitamin D levels
hypocalcaemia had been noted by his
psychiatrist a number of years previously but 3- Overnight or 36-hour fast with
had not been investigated further. He has measurement of insulin and C-peptide
some further investigations the results of levels if hypoglycaemia occurs
which are as follows: 4- Short synacthen test
Sodium 134mmol/L 5- Urine sulphonylurea screen

Potassium 5.0mmol/L Answer & Comments


Urea 0.8mmol/L 4- Short synacthen test

Creatinine 47µmol/L This patient has primary hypoparathyroidism


as evidenced by a normal PTH despite
Random glucose 3.9mmol/L significantly low adjusted calcium.
Hypoparathyroidism in the presence of clinical
GGT 56U/L features such as enamel hypoplasia and nail
abnormalities is consistent with a diagnosis of
ALP 222U/L
auto - immune polyendocrinopathy-
candidiasis-ectodermal dystrophy syndrome
ALT 34U/L
(APECED). This is an autosomal recessive
Bilirubin 16µmol/L condition mapped to 21q22.3 (the
autoimmune regulator gene) and is defined by
Albumin 36g/L the presence of 2 of the 3 following conditions:
chronic mucocutaneous candidiasis,
Amylase 45U/L (30–100) hypoparathyroidism or Addison’s disease.
Other mucocutaneous manifestations include
Magnesium 0.7mmol/L alopecia areata and vitiligo.

Adjusted calcium (on treatment) 1.82mmol/L There is an association with other autoimmune
(2.1–2.7) conditions including hypothyroidism, type 1
diabetes, hypogonadism, chronic active
Phosphate 2.4mmol/L (0.7–1.0) hepatitis, primary biliary cirrhosis and
pernicious anaemia.
PTH 4.5ng/L (10–55)
In the context of positive adrenal antibodies,
Adrenal autoantibodies: positive minor electrolyte disturbance consistent with
adrenal insufficiency and a relatively low
Random cortisol 400nmol/L cortisol level considering the physiological
insult undergone by this patient, it is
Renal ultrasound: normal sized kidneys with
imperative firstly to exclude adrenal
normal cortical depth and no hydronephrosis
insufficiency which is potentially life
What is the most appropriate next step in his threatening if left untreated.
management?
Further reading
Buzi F, et al. Autoimmune polyendocrinopathy-
candidiasis-ectodermal dystrophy syndrome:

63 | Dr. Khalid Yusuf (FB: Sohag Endocrine Group)


SCE Endocrinology (sce.practicaldiabetes.com), 2017

time to review diagnostic criteria? J Clin Bartter’s and Gitelman’s syndrome would
Endocrinol Metab 2003;88:3146–8. present at an earlier age. Patients tend to be
normotensive or hypotensive, with activation
Q 95 of the renin-aldosterone system. There is an
excess of renal prostaglandin production in
A 27-year-old man presents with lethargy and Bartter’s syndrome and NSAIDs can be used for
muscle cramps. He was recently found to have treatment.
low potassium levels by his GP for which he
received a short course of potassium Further reading
supplements. There is a family history of Hassan-Smith Z, Stewart PM. Inherited forms of
mineralocorticoid hypertension. Curr Opin
hypertension. He has a BMI of 23kg/m2 and his
Endocrinol Obes 2011;18:177–85.
BP is recorded at 185/90mmHg. The rest of his
physical examination is normal.
Q 96
Which of the following statements is true for his
condition?
A 56-year-old woman is referred to the
1- Mode of inheritance is autosomal endocrinology clinic with weight gain, easy
recessive bruising, tiredness and hirsutism. On
examination, she was found to have features of
2- The renin-aldosterone system will be
proximal myopathy. A clinical diagnosis of
activated
Cushing’s syndrome was made.
3- Urinary calcium excretion will be
All of the following are suitable for initial
increased
testing of Cushing’s syndrome except:
4- His hypertension can be treated with
amiloride 1- 1mg overnight dexamethasone
suppression test
5- There is a role for NSAIDs in treatment of
this condition 2- 48-hour low dose (2mg/day)
dexamethasone suppression test

Answer & Comments 3- 9:00am cortisol level


4- Late-night salivary cortisol
4- His hypertension can be treated with
amiloride 5- 24-hour urinary free cortisol
measurement (at least 2 collections)
The case here is descriptive of Liddle’s
syndrome – the important findings here being Answer & Comments
hypertension and hypokalaemia. There is
suppression of the reninaldosterone system. 3- 9:00am cortisol level
The mode of inheritance is auto somal
dominant with variable penetrance, and the The Endocrine Society published an evidence-
defect is in the gene encoding the epithelial Na based clinical practice guideline on the
channel (ENaC) on chromosome 16. The diagnosis of Cushing’s syndrome in 2008. The
hypertension responds better to amiloride, guideline describes 4 screening tests for
which directly blocks ENaC. Cushing’s syndrome which include:

Spironolactone acts by regulating aldosterone • Urine free cortisol measurements (at least 2
and Liddle’s syndrome does not respond to this measurements).
regulation. Liddle’s syndrome can mimic
Conn’s syndrome and this is a differential • Late-night salivary cortisol (2
diagnosis. measurements).

64 | Dr. Khalid Yusuf (FB: Sohag Endocrine Group)


SCE Endocrinology (sce.practicaldiabetes.com), 2017

• 1mg overnight dexamethasone suppression 2- Low HDL cholesterol level


test.
3- High total cholesterol level
• Low-dose dexamethasone suppression test 4- Severe obesity
(2mg/day for 48 hours).
5- Presence of background retinopathy
The guideline specifically recommends against
the use of the following to test for Cushing’s Answer & Comments
syndrome.
5- Presence of background retinopathy
• Random serum cortisol or plasma ACTH
levels. Obesity, high total cholesterol, low HDL
cholesterol and hypertension are features of
• Urinary 17-ketosteroids. metabolic syndrome and thus would be in
keeping with a diagnosis of type 2 diabetes in
• Insulin tolerance test. patients in a younger age group, such as this
woman. Nevertheless, these features do not
• Loperamide test. provide definitive evidence of type 2 diabetes,
as it would always be possible for type 1
• Tests designed to determine the cause of diabetes to manifest in such patients.
Cushing’s syndrome (e.g. pituitary and adrenal However, retinopathy would not be seen at
imaging, 8mg high dose dexamethasone diagnosis of type 1 diabetes, whereas up to
suppression test). 20% of patients with type 2 diabetes will
demonstrate evidence of microvascular
A 9:00am cortisol is not a good screening test complications.
for Cushing’s syndrome.
Further reading
Further reading Turner RC. The UK Prospective Diabetes Study: A
Nieman LK, et al. The diagnosis of Cushing’s review. Diabetes Care 1998;21(Suppl 3):C35–
syndrome: an Endocrine Society Clinical 8.
Practice Guideline. J Clin Endocrinol Metab
2008;93:1526–40.
Q 98

Q 97 A 58-year-old man with a 10-year history of


type 2 diabetes is reviewed in clinic. He
A 29-year-old woman presents to her GP with currently takes metformin 2g/day and
recurrent vaginal thrush. She has a previous gliclazide 160mg/day and a recent HbA1c
history of hypothyroidism and is receiving confirmed poor diabetes control at
thyroxine replacement. On examination, her 79mmol/mol (9.4%). He complains of weight
body mass index is 36kg/m2 and her blood loss, burning proximal muscle pain and
pressure is 144/85mmHg. Dilated fundoscopy weakness. On examination there is leftsided
shows the presence of microaneurysms and quadriceps wasting.
blot haemorrhages. Laboratory investigations
reveal an HDL of 0.7mmol/L, total cholesterol Which of the following would be the most
of 4.5mmol/L, fasting plasma glucose of important initial investigation?
13mmol/L and an HbA1c of 75mmol/mol (9%). 1- Autoantibodies
Which of the following would most favour a 2- Nerve conduction studies
diagnosis of type 2 rather than type 1 diabetes
3- MRI of the spinal cord
for this woman?
4- Lumbar spine plain films
1- Presence of hypertension

65 | Dr. Khalid Yusuf (FB: Sohag Endocrine Group)


SCE Endocrinology (sce.practicaldiabetes.com), 2017

5- Muscle biopsy 4- Metformin increases hepatic and


peripheral fatty acid oxidation
Answer & Comments 5- Metformin undergoes significant drug
interactions with warfarin, cimetidine
3- MRI of the spinal cord and ACE inhibitors
This clinical presentation is suggestive of
diabetic amyotrophy, an uncommon and Answer & Comments
disabling condition which is said to usually
4- Metformin increases hepatic and
affect men during or after middle age. It can
peripheral fatty acid oxidation
occur in type 1 or type 2 diabetes, but more
commonly is seen in type 2 diabetes. The
Metformin improves insulin sensitivity, with
condition affects lumbar sacral plexus lower
data suggesting an increase in AMP kinase
motor neurones. Patients develop severe
activity mediating an increase in glucose
aching or burning and lancinating pain in the
utilisation and fatty acid oxidation both in the
hip and thigh. This is followed by weakness and
liver and peripheral tissues. The primary site of
wasting of the thigh muscles, which often occur
action of metformin appears to be the liver
asymmetrically. Concomitant distal,
with suppression of hepatic glycogenolysis,
predominantly sensory neuropathy may exist.
gluconeogenesis and an increase in splanchnic
The results of most electrodiagnostic studies
glucose turnover all being involved in the
are consistent with the presence of a
glucose lowering effects of metformin. It is
neurogenic lesion that could be associated
excreted almost unchanged in the urine, more
with lumbo sacral plexopathy, radiculopathy,
by tubular secretion than by glomerular
or proximal crural neuropathy. However, prior
filtration. Hence, metformin is contraindicated
to making the diagnosis, an MRI of the spine is
in significant renal impairment due to an
important to exclude other causes such as disc
increased risk of metabolic acidosis,
disease or malignancy.
particularly in the presence of an inter-current
illness or in the context of other conditions
Further reading associated with tissue hypoxia. Metformin may
Tracy JA, Dyck PJ. The spectrum of diabetic
reduce intestinal vitamin B12 absorption and
neuropathies. Phys Med Rehabil Clin N Am
2008;19:1–26. only undergoes drug interactions with
cimetidine.

Q 99 Further reading
Holt RIG, et al. (eds). Textbook of diabetes, 4th
A 65-year-old man with a 2-year history of diet- edn. Chapter 29. Wiley-Blackwell, 2010.
controlled type 2 diabetes is found to have an
HbA1c of 57mmol/mol (7.4%). He is Q 100
commenced on metformin 500mg twice daily.
Which of the following is a recognised feature A woman attends the antenatal clinic and is
of metformin therapy? noted to be taking 10?g of vitamin D daily to
maintain adequate vitamin D stores during
1- Metformin improves hepatic glucagon
pregnancy.
sensitivity
All of the following groups have a particularly
2- Metformin suppresses AMP-activated
high risk of antenatal vitamin D deficiency,
protein kinase
except:
3- Metformin decreases splanchnic glucose
turnover 1- Women of South Asian, African,
Caribbean or Middle Eastern family
origin

66 | Dr. Khalid Yusuf (FB: Sohag Endocrine Group)


SCE Endocrinology (sce.practicaldiabetes.com), 2017

2- Women who have limited exposure to the healthy pregnant woman. CG62. NICE,
sunlight March 2008.

3- Women who eat a diet particularly low


in vitamin D Q 101

4- Women with 3 or more previous


A 54-year-old man is referred to the endocrine
pregnancies
clinic with abnormal thyroid function tests. He
5- Women with a pre-pregnancy body mass was commenced on amiodarone 6 months
index above 30kg/m2 previously for an episode of ventricular
tachycardia following an acute ST elevation
Answer & Comments myocardial infarction. On examination he is in
normal sinus rhythm and appears clinically
4- Women with 3 or more previous euthyroid with no evidence of goitre. His
pregnancies thyroid function tests are as follows: TSH
1.2mU/L (0.2–6.0) Free T4 32pmol/L (10.0–
There is some evidence of benefit from vitamin 20.0) Free T3 2.9pmol/L (3.0–6.5)
D supplementation for pregnant women at
What is the next most appropriate
high risk of vitamin D deficiency, but there is
management step for this patient?
less evidence in the case of pregnant women
currently regarded as being at low risk of 1- Arrange a thyroid technetium uptake
deficiency. NICE guidelines (CG62) recommend scan or colour Doppler ultrasound
that women should be informed at the booking
2- Commence on carbimazole 20mg once
appointment about the importance, for their
daily and recheck thyroid function tests
own and their baby’s health, of maintaining
in 6–8 weeks
adequate vitamin D stores during pregnancy
and while breastfeeding. Particular care should 3- Commence on prednisolone 40mg and
be taken to enquire as to whether women at carbimazole 40mg once daily and
greatest risk are following advice to take this recheck thyroid function tests in 2 weeks
daily supplement. These include:
4- Discharge him to primary care and
advise his GP to check thyroid function
• Women of South Asian, African, Caribbean or
tests 6 monthly or if symptomatic
Middle Eastern family origin.
5- Withdraw amiodarone therapy after
• Women who have limited exposure to discussion with his cardiologist
sunlight, such as those who are predominantly
housebound, or usually remain covered when
Answer & Comments
outdoors.
4- Discharge him to primary care and
• Women who eat a diet particularly low in advise his GP to check thyroid function
vitamin D, such as those who consume no oily tests 6 monthly or if symptomatic
fish, eggs, meat, vitamin D-fortified margarine
or breakfast cereal. This patient is clinically and biochemically
euthyroid, as evidenced by the normal TSH
• Women with a pre-pregnancy BMI above level and free T3 level. Amiodarone inhibits the
30kg/m2. A previous history of pregnancy is metabolism of free T4 (FT4) to free T3 (FT3) and
not a risk factor for vitamin D deficiency. increases the proportion metabolised to
reverse T3 (rT3). In the first few months of
Further reading therapy this results in an initial rise in TSH and
National Institute for Health and Clinical a subsequent rise in FT4 with normalisation of
Excellence. Antenatal care: routine care for TSH and FT3 as a compensatory mechanism

67 | Dr. Khalid Yusuf (FB: Sohag Endocrine Group)


SCE Endocrinology (sce.practicaldiabetes.com), 2017

(the amiodarone effect). This does not Chapman MJ. Fibrates: therapeutic review. Br J
constitute thyrotoxicosis and does not require Diab Vasc Dis 2006;6:11–21.
treatment, although long-term monitoring of
thyroid function tests is advisable. Q 103

Further reading A 57-year-old Caucasian male is admitted to


Han TS, et al. Benzofuran derivatives and the the coronary care unit following an episode of
thyroid. Clin Endocrinol (Oxf) 2009;70:2–13.
chest pain with diaphoretic symptoms. His
blood investigations confirm a rise in serum
Q 102 troponin level consistent with an acute
coronary syndrome (ACS). He is an ex-smoker
A 46-year-old man with type 2 diabetes is with a previous medical history of
reviewed in clinic. He has a family history of hypertension, dyslipidaemia and obesity-
premature cardiovascular disease. Since his hypoventilation syndrome. He has no known
last review, his GP has commenced him on history of diabetes mellitus. On admission, he
fibrate therapy as his lipid profile revealed is found to have a random glucose level of
triglyceride levels of 4.2mmol/L and a high- 11.5mmol/L. Prior to his discharge from
density lipoprotein level of 0.7mmol/L. hospital 5 days later, his fasting glucose is
5.9mmol/L and HbA1c 48mmol/mol (6.5%).
Which of the following options best describes
the mode of action for fibrates? Which of the following statements is correct?

1- They competitively inhibit 1- Intensive insulin therapy comprising


hydroxymethylglutaryl-CoA (HMG-CoA) intravenous insulin and glucose infusion
activity with or without potassium should be
routinely offered to patients with
2- They are inhibitors of cholesterol ester
hyperglycaemia following ACS (defined
transferase protein (CETP) activity
as blood glucose >11.0mmol/L)
3- They inhibit adipocyte lipolysis
2- Intensive treatment to lower glucose in
4- They bind bile acids in the gut ACS is clearly associated with a reduction
in inpatient morbidity and mortality
5- They are activators of lipoprotein lipase
activity and increase HDL synthesis 3- An oral glucose tolerance test should be
routinely performed in all patients with
Answer & Comments hyperglycaemia after ACS
4- The patient is most likely to have
5- They are activators of lipoprotein lipase impaired glucose tolerance and should
activity and increase HDL synthesis be given lifestyle advice regarding
healthy eating, smoking cessation and
The mode of action of fibrates is to enhance
alcohol consumption
lipoprotein lipase activity, increase low-density
lipoprotein (LDL) receptor-mediated clearance 5- Patients with hyperglycaemia after ACS
of LDL and increase high-density lipoprotein are at increased risk of developing type
(HDL) synthesis. This leads to a corresponding 2 diabetes and should be offered
fall in triglycerides due to a fall in very low- monitoring of fasting blood glucose
density lipoprotein (VLDL), and a rise in HDL. In levels and HbA1c at least every 6 months
total, they reduce triglycerides by 20–60% and
LDL by 5–25%, and increase HDL by 15–30%. Answer & Comments

Further reading 4- The patient is most likely to have


impaired glucose tolerance and should

68 | Dr. Khalid Yusuf (FB: Sohag Endocrine Group)


SCE Endocrinology (sce.practicaldiabetes.com), 2017

be given lifestyle advice regarding blood glucose monitoring, which was


healthy eating, smoking cessation and confirmed on venous glucose sampling. The
alcohol consumption patient denied change in weight and had no
features of sepsis or excess catabolism. She
Up to 85% of patients with acute coronary had no known history of diabetes mellitus and
syndrome (ACS) will have some degree of did not use oral hypoglycaemic agents or
dysglycaemia at presentation, of whom 40% insulin therapy. Her investigations are as
will have impaired glucose tolerance, 25% follows:
undiagnosed type 2 diabetes and 20% known
diabetes mellitus. Lifestyle intervention is TSH 1.6mU/L (0.2–6.0)
recommended for patients without known
diabetes and hyperglycaemia after ACS. Free T4 14.4pmol/L (10.0–20.0)

The NICE clinical guideline for the management Short synacthen test: 9am cortisol 407nmol/L
of hyperglycaemia in ACS recommends a dose- (280–700); 30-min cortisol 609nmol/L
adjusted insulin infusion to keep blood glucose
below 11mmol/L, but does not support the Glucose 1.6mmol/L
routine use of intensive insulin therapy as
described in the question stem. It is unclear Insulin C-peptide 607pmol/L
whether intensive treatment to lower glucose
Insulin 46pmol/L
improves outcomes, although high glucose at
presentation is associated with increased
She was commenced on diazoxide which
mortality.
resulted in a reduction in frequency of
hypoglycaemic episodes.
An oral glucose tolerance test (OGTT) should
not be routinely offered to patients with Which of the following statements is correct?
normal fasting glucose and HbA1c. Screening
1- CT pancreas with intravenous contrast is
for diabetes mellitus in patients who have had
the imaging modality of choice and has a
hyperglycaemia after ACS should be performed
sensitivity of 80%
at least annually, or if symptoms of diabetes
develop. 2- Diazoxide promotes hyperpolarisation
of ATPsensitive potassium channels
Further reading (KATP) in smooth muscle and may cause
Norhammar A, et al. Glucose metabolism in hypertension
patients with acute myocardial infarction and
no previous diagnosis of diabetes mellitus: a 3- Diazoxide inhibits generation of beta-cell
prospective study. Lancet 2002;359:2140–4. membrane action potential by binding to
ATPsensitive potassium channels (KATP)
National Institute for Health and Clinical
Excellence. NICE Clinical Guideline 130. 4- Surgical resection is the treatment of
Hyperglycaemia in acute coronary syndromes. choice for all patients because of the risk
October 2011. of malignancy
www.nice.org.uk/nicemedia/live/13589/5681
7.pdf [Accessed 15 Oct 2012]. 5- 80% of patients develop this condition in
association with MEN-1
Q 104
Answer & Comments
A 97-year-old woman was admitted to a care of
3- Diazoxide inhibits generation of beta-cell
the elderly ward with unsteadiness, and
membrane action potential by binding to
treated empirically for a urinary tract infection.
ATPsensitive potassium channels (KATP)
While an inpatient, she was noted to have
asymptomatic hypoglycaemia on capillary

69 | Dr. Khalid Yusuf (FB: Sohag Endocrine Group)


SCE Endocrinology (sce.practicaldiabetes.com), 2017

Diazoxide was initially developed as an anti- 4- Approximately 50% of patients with type
hypertensive agent and later found to have 1 diabetes and microalbuminuria will
anti-hypoglycaemic properties. Diazoxide progress to overt proteinuria
inhibits insulin release by binding to ATP-
5- Microalbuminuria often precedes the
sensitive potassium channels (KATP) in the
onset of retinopathy
beta-cell membrane, causing them to remain
open. This hyperpolarises the cell membrane
and prevents the generation of cell membrane Answer & Comments
action potentials, thus inhibiting exocytosis of
1- Elevated urinary microalbumin excretion
insulin-containing vesicles. A similar
predicts an increased risk of
mechanism of action occurs in smooth muscle
atherosclerotic cardiovascular disease
and accounts for the drug’s hypotensive
independent of blood pressure, duration
effects.
of type 1 diabetes or glycaemic control
Insulinomas are usually solitary and benign,
Patients with type 1 diabetes and urinary
but 80% of multifocal insulinomas are
albumin excretion of 30–300mg/24 hours have
associated with multiple endocrine neoplasia
a higher risk of atherosclerotic vascular disease
type 1 (MEN-1). Radiological imaging of
than those with lower albumin excretion rates,
insulinomas is difficult, and contrast enhanced
and the presence of microalbuminuria appears
CT has a sensitivity of 50–80%. Intraoperative
to predict this risk independent of other risk
ultrasound offers the highest sensitivity, and
factors.
surgical resection is the intervention of choice
except in the elderly or debilitated. Normal urinary albumin excretion is <20mg/24
hours and values 30–300mg/24 hours are
Further reading consistent with microalbuminuria. Other
George P, McCrimmon R. Diazoxide. Pract
causes of microalbuminuria include
Diabetes 2012;29:36–7.
hypertension and post-streptococcal
Shin JJ, et al. Insulinoma: pathophysiology, glomerulonephritis. Retinopathy usually
localization and management. Future
precedes the onset of microalbuminuria and
Oncology 2010;6:229–37.
nephropathy, and other causes should be
sought if retinopathy is not present.
Q 105
Further reading
A young patient with newly diagnosed type 1 Arun CS, et al. Significance of microalbuminuria in
diabetes asks why annual screening for urinary long-duration type 1 diabetes. Diabetes Care
microalbuminuria is performed. 2003;26:2144–9.

Which of the following statements is true?


Q 106
1- Elevated urinary microalbumin excretion
predicts an increased risk of A 58-year-old male head teacher with type 2
atherosclerotic cardiovascular disease diabetes has been advised by his general
independent of blood pressure, duration practitioner that he should commence on
of type 1 diabetes or glycaemic control cholesterol-lowering therapy because he is at
2- Normal urinary albumin excretion is less increased risk of cardiovascular disease. He is
than 300mg in 24 hours reluctant to take statin therapy because he has
heard there is a high incidence of adverse
3- Microalbuminuria is pathognomonic of effects and he has read in the national press
diabetic nephropathy
that the use of statins is associated with the
development of Alzheimer’s disease.

70 | Dr. Khalid Yusuf (FB: Sohag Endocrine Group)


SCE Endocrinology (sce.practicaldiabetes.com), 2017

How might you explain the benefits of statin Further reading


therapy to this patient? National Institute for Health and Clinical
Excellence. Statins for the prevention of
1- Long-term statin therapy is protective cardiovascular events. Technology Appraisal
against Alzheimer’s dementia 94. January 2006.
www.nice.org.uk/nicemedia/pdf/TA094guida
2- Myalgia is a rare complication of statin
nce.pdf [Accessed 16 Oct 2012].
therapy and usually only occurs at high
doses
Q 107
3- The number needed to treat (NNT) to
avoid either death from coronary heart
Current guidelines recommend administration
disease or a nonfatal myocardial
of 15–20g of rapid-acting carbohydrate to
infarction is 25
adult patients with hypoglycaemia who are
4- Statin therapy reduces the risk of death alert, orientated and able to swallow.
from heart disease by approximately
Which of the following is not equivalent to 15g
20%
of rapid acting carbohydrate?
5- Statin therapy reduces the risk of death
from stroke by 25% 1- 5 Jelly Babies
2- 3 teaspoons of sugar or jam
Answer & Comments 3- 5 glucose tablets
4- Statin therapy reduces the risk of death 4- 150ml of fruit juice or non-diet soft drink
from heart disease by approximately 5- 150ml of original Lucozade™
20%

Statins are a commonly prescribed medication Answer & Comments


for the treatment of hyperlipidaemia.
5- 150ml of original Lucozade™
However, it is important to remember that the
reason for such treatment is to reduce the risk All of the suggested options contain rapid-
of cardiovascular disease in patients with a 20% acting carbohydrate except 150ml of original
or higher risk of developing cardiovascular LucozadeTM which provides 25g of
disease in the next 10 years. A meta-analysis of carbohydrate (15g is equivalent to 90ml).
all placebo-controlled trials of statin therapy in Lucozade drinks are commonly recognised by
people without known cardiovascular disease both patients and health care professionals as
suggests a reduction in mortality from a source of rapid-acting carbohydrate, but the
cardiovascular disease by approximately 20%. carbohydrate content varies according to the
The incidence of non-fatal strokes is reduced type of Lucozade drink consumed.
by 25%, but there is no significant reduction in
stroke mortality. The number needed to treat If administration of 15–20g rapid-acting
(NNT) for primary prevention of non-fatal carbohydrate does not result in a rise in
myocardial infarction or death from coronary capillary blood glucose above 4mmol/L after 15
heart disease is 95. Use of statins is not minutes, a further 15–20g should be
implicated in the development of Alzheimer’s administered for up to 3 cycles before
disease and there is insufficient evidence to considering parenteral therapy, e.g.
support the assertion that statin therapy is intramuscular glucagon or intravenous
protective against dementia. Although dextrose.
rhabdomyolysis is rare, myalgia is a common
complication and is often the main reason for Further reading
non-compliance with therapy. NHS Diabetes. The Hospital Management of
Hypoglycaemia in Adults with Diabetes

71 | Dr. Khalid Yusuf (FB: Sohag Endocrine Group)


SCE Endocrinology (sce.practicaldiabetes.com), 2017

Mellitus. March 2010. 5- Refer for consideration of radioiodine


www.diabetes.nhs.uk/document.php?o=217 therapy
[Accessed 16 Oct 2012].

Answer & Comments


Q 108
3- Stop all treatment and repeat thyroid
A 57-year-old woman is referred to the function tests in 12 weeks or if
endocrine clinic with mild symptoms of symptomatic
thyrotoxicosis and a painless thyroid swelling.
She has no evidence of dysthyroid eye disease This patient has rapidly become hypothyroid
and is a non-smoker. Investigations are as after commencing antithyroid drugs for mild
follows: thyrotoxicosis, thus she is likely to have ‘silent’
or ‘painless’ autoimmune thyroiditis. This
TSH <0.01mU/L condition is characterised by painless goitre
with transient thyrotoxicosis due to release of
Free T4 28.6pmol/L (10–23) pre-formed thyroid hormone from the thyroid,
with subsequent return to euthyroidism or
Free T3 6.8pmol/L (3.5–6.5) hypothyroidism over a period of months.
Thyroid peroxidase antibodies may be positive,
TPO antibodies: positive and thyroid fine-needle aspiration (FNA)
demonstrates lymphocytic infiltration.
USS thyroid: diffusely enlarged thyroid gland Hypothyroidism may be transient or
with no discrete nodules. Left-sided enlarged permanent, requiring short- or long-term
lymph node approximately 1.8cm x 0.5cm thyroxine supplementation. Other forms of
thyroiditis include subacute (de Quervain’s)
FNA neck lump: consistent with reactive lymph thyroiditis, which is characterised by painful
node thyroid swelling, and post-partum thyroiditis. If
thyroiditis is suspected when a patient
She is commenced on carbimazole 10mg once
becomes rapidly hypothyroid on antithyroid
daily and counselled regarding the risk of
medication, treatment should be withdrawn to
agranulocytosis. Four months later she returns
assess whether normal thyroid function has
for review and has the following investigations:
returned.
TSH 18.3 mU/L

Free T4 8.1pmol/L Further reading


Samuels MH. Subacute, silent and postpartum
thyroiditis. Med Clin North Am 2012;96:223–
Free T3 5.2pmol/L
33.
What is the next most appropriate course of
action? Q 109
1- Add levothyroxine 50?g once daily to her
current medication A 37-year-old woman with a 2-year history of
type 2 diabetes mellitus attends clinic. She has
2- Reduce carbimazole to 5mg once daily
a previous medical history of obesity (BMI
and recheck thyroid function tests in 6–
37kg/m2), hypertension, dyslipidaemia and
8 weeks
polycystic ovarian syndrome, for which she
3- Stop all treatment and repeat thyroid takes metformin, pioglitazone, ramipril and
function tests in 12 weeks or if simvastatin. She is a smoker and is keen to lose
symptomatic weight. Her investigations are as follows:
4- Change antithyroid medication to
BP 126/76mmHg
propylthiouracil 50mg twice daily

72 | Dr. Khalid Yusuf (FB: Sohag Endocrine Group)


SCE Endocrinology (sce.practicaldiabetes.com), 2017

HbA1c 60mmol/mol (7.6%) patient’s longterm risk of cardiovascular


disease.
Cholesterol 6.8mmol/L
Further reading
HDL 0.8mmol/L YHPHO, NHS Diabetes. Diabetes with Kidney
Disease: Key Facts. March 2011.
Creatinine 71?mol/L www.yhpho.org.uk/resource/item.aspx?RID=
105786.
eGFR >90ml/min/1.73m2 National Institute for Health and Clinical
Excellence. CG73. Chronic kidney disease:
ACR 165mg/mmol early identification and management of
What is the next most important step in this chronic kidney disease in adults in primary and
secondary care. Sept 2008.
patient’s management?
www.nice.org.uk/cg73.
1- Commence on GLP-1 agonist therapy
and discontinue pioglitazone Q 110
2- Increase dose of simvastatin or consider
atorvastatin aiming for a total A 66-year-old man with type 2 diabetes for 15
cholesterol of 4mmol/l or 25% years was admitted to the stroke unit via
reduction, whichever is the greater accident and emergency having suffered a
middle cerebral artery (MCA) infarct. He was
3- Measure urinary protein:creatinine ratio
ineligible for thrombolysis due to late
after exclusion of urinary tract infection
presentation and has a residual dense
and consider referral to nephrologists
hemiparesis with dysphasia and dysphagia
4- Refer for support with smoking cessation despite supportive care. Prior to admission, his
5- Repeat early morning urinary diabetes was well controlled on metformin,
albumin:creatinine ratio after exclusion gliclazide and sitagliptin. HbA1c on admission
of urinary tract infection and maximise was 46mmol/mol (6.4%). A decision is made by
ACE inhibitor therapy as tolerated the stroke multidisciplinary team to commence
nasogastric feeding after 1 week. His capillary
blood glucose is persistently >12mmol/L during
Answer & Comments feeding.
3- Measure urinary protein:creatinine ratio What is the most appropriate way to manage
after exclusion of urinary tract infection his diabetes?
and consider referral to nephrologists
1- Administer soluble or liquid forms of his
The important thing to note in this case is the usual oral hypoglycaemic agents via the
presence of significant proteinuria despite ACE nasogastric tube
inhibitor therapy in a patient with a relatively 2- Commence a variable rate intravenous
short history of diabetes. This raises the insulin infusion (VRIII) until his capillary
possibility that there is an alternative cause of glucose is in the target range of 6–
proteinuria other than diabetic nephropathy. If 12mmol/L
there is persistent proteinuria with no
evidence of urinary tract infection, the patient 3- Commence on a long-acting analogue
should be referred to a nephrologist for review insulin (glargine or detemir) and titrate
and consideration of renal biopsy. Measures to until fasting capillary glucose <6mmol/L
support weight loss, reduction in total 4- Use a continuous glucose-potassium-
cholesterol and smoking cessation are insulin (GKI) infusion and increase insulin
important interventions to reduce this dose during naso - gastric feeding

73 | Dr. Khalid Yusuf (FB: Sohag Endocrine Group)


SCE Endocrinology (sce.practicaldiabetes.com), 2017

5- Continue metformin in the form of presentation, and had been applying


resuspended metformin powder via absorbent dressings while at work, leaving the
nasogastric tube and consider isophane wound open at home. On examination he has
or pre-mixed human insulin at start and diminished sensation to 10g monofilament
mid-point of feed if capillary blood bilaterally. His left 5th toe is swollen and
glucose persistently >12mmol/L erythematous, and a 1.2cm diameter ulcer
with sloughy base is noted on the lateral aspect
Answer & Comments of the left 5th toe, approximately 3mm in
depth.
5- Continue metformin in the form of What is the recommended next step in this
resuspended metformin powder via
patient’s management?
nasogastric tube and consider isophane
or pre-mixed human insulin at start and 1- Commence on oral antibiotics according
mid-point of feed if capillary blood to local antibiotic policy and review in 72
glucose persistently >12mmol/L hours
2- Liaise with local diabetes footcare team
There is currently considerable variability in
for advice and urgent review
the management of hyperglycaemia in patients
with newly-diagnosed or known type 2 3- Perform foot X-ray and refer for
diabetes mellitus receiving enteral feeding. orthopaedic opinion if osteomyelitis
Metformin powder, resuspended and confirmed
administered via nasogastric tube, should be
4- Refer to community podiatry for review
considered for patients with well-controlled
regarding appropriate dressings
type 2 diabetes (capillary blood glucose 6–
12mmol/L) in whom capillary blood glucose 5- Refer urgently for inpatient hospital
rises to >12mmol/L during feeding. Patients admission
with type 2 diabetes whose capillary blood
glucose is >12mmol/L despite metformin Answer & Comments
therapy should be commenced on isophane or
pre-mixed (30/70) human insulin administered 2- Liaise with local diabetes footcare team
at the start and mid-point of nasogastric for advice and urgent review
feeding. Basal analogue insulins may be
continued if already established on this type of Local services may vary but it is recommended
insulin prior to admission, although additional that patients with diabetes who present with
boluses of soluble insulin may be required. active foot ulceration are reviewed by the
diabetes multidisciplinary footcare team
Further reading within 24 hours. The assessment should
Joint British Diabetes Societies (JBDS) for inpatient include samples for microbiological culture, X-
care. Glycaemic management during the rays where osteomyelitis is suspected,
inpatient enteral feeding of stroke patients antibiotic therapy, review of dressings,
with diabetes. June 2012. appropriate footwear and offloading
www.diabetes.nhs.uk/document.php?o=3766 techniques, glycaemic control, management of
. neuropathy and vascular risk factors, and
vascular or orthopaedic opinion where
Q 111 appropriate. The clinical features described
may be consistent with osteomyelitis, but it
A 62-year-old civil engineer with type 2 would be unwise to defer intervention until
diabetes presents with a non-healing wound radiological confirmation.
on the lateral aspect of his left 5th toe. He first
noticed a blister at the site 4 months prior to It may be acceptable in some situations to
commence on antibiotics according to local

74 | Dr. Khalid Yusuf (FB: Sohag Endocrine Group)


SCE Endocrinology (sce.practicaldiabetes.com), 2017

policy after taking appropriate samples for A meta-analysis of studies comparing therapies
microbiological culture, or refer for immediate for type 2 diabetes suggests that metformin
inpatient admission, for example if access to monotherapy is more beneficial than
specialist diabetes footcare services is likely to sulphonylurea monotherapy in terms of
be delayed. glycaemic control, low-density lipoprotein
cholesterol and triglyceride levels in patients
Further reading with type 2 diabetes.
National Institute for Health and Clinical
Excellence. CG10. Type 2 diabetes: prevention The United Kingdom Prospective Diabetes
and management of foot problems. Jan 2004. Study trials demonstrated that intensive
http://guidance.nice.org.uk/CG10. glycaemic control with metformin had a
greater beneficial effect than conventional
Q 112 non-intensive dietary control,
chlorpropramide, glibenclamide or insulin in
The oral biguanide metformin is recommended terms of diabetes-related outcomes and all-
as first-line therapy for overweight patients cause mortality, but there were no significant
with type 2 diabetes mellitus. differences with respect to myocardial
infarction, stroke or peripheral vascular
Which of the following statements is true disease.
regarding metformin use in patients with type
2 diabetes? The bioavailability of metformin is
approximately 60% in the fasting state and
1- Metformin should be administered with
absorption is delayed by ingestion with food.
meals to enhance absorption and
Ingestion of metformin with food is
increase efficacy
recommended to reduce gastrointestinal
2- Lactic acidosis is a significant risk of intolerability in susceptible patients.
metformin therapy and occurs in
approximately 1:1000 patients There have been no reported cases of fatal or
nonfatal lactic acidosis associated with
3- Metformin is absolutely contraindicated
metformin therapy, and there is emerging
during pregnancy
evidence that continuation of established
4- Metformin is more beneficial than metformin therapy in pregnant women is safe
sulphonylurea therapy in terms of low- and effective.
density lipoprotein cholesterol and
triglyceride levels Further reading
National Collaborating Centre for Chronic
5- Intensive glycaemic control with
Conditions. Type 2 diabetes: national clinical
metformin therapy results in a reduction guideline for management in primary and
in myocardial infarction, stroke and secondary care (update). London: Royal
peripheral vascular disease when College of Physicians, 2008.
compared to intensive glycaemic control www.nice.org.uk/nicemedia/live/11983/4080
with sulphonylureas or insulin 3/40803.pdf.
National Institute for Health and Clinical
Answer & Comments Excellence. CG63. Diabetes in pregnancy:
Management of diabetes and its
4- Metformin is more beneficial than complications from pre-conception to the
sulphonylurea therapy in terms of low- postnatal period. March 2008.
density lipoprotein cholesterol and http://publications.nice.org.uk/diabetes-in-
triglyceride levels pregnancycg63/guidance/.

75 | Dr. Khalid Yusuf (FB: Sohag Endocrine Group)


SCE Endocrinology (sce.practicaldiabetes.com), 2017

Q 113 The Endocrine Society. Management of thyroid


dysfunction during pregnancy and
postpartum. 2012. www.endo-
A 30-year-old pregnant woman is referred to
society.org/guidelines/upload/Thyroid-Exec-
the medical disorders in pregnancy clinic. Her
Summ.pdf.
thyroid function tests are as follows.

TSH 7.6mU/L (normal range: 0.2–6.0) Q 114

Free T4 12pmol/L (normal range: 10–23.0) Which enzyme deficiency causes Von Gierke
disease?
Which of the following statements is true
regarding treatment of subclinical 1- Acid maltase
hypothyroidism in pregnancy? 2- Glucose-6-phosphatase
1- Treatment improves obstetric outcome 3- Hepatic phosphorylase
and is associated with improved long-
4- Muscle phosphorylase
term neurological development in
offspring 5- Phosphofructokinase deficiency
2- Treatment does not improve obstetric
outcome but is associated with Answer & Comments
improved long-term neurological
2- Glucose-6-phosphatase
development in offspring
3- Treatment has no effect on obstetric Von Gierke disease is also known as type 1
outcome or neurological development in glycogen storage disease. It is an autosomal
offspring recessive condition associated with deficiency
of glucose-6- phosphatase, which leads to
4- Treatment improves obstetric outcome
abnormal build up of glycogen in the liver and
but has no effect on neurological
causes significant end-organ disease. It usually
development in offspring
presents in infancy with fatiguability, irritability
5- Subclinical hypothyroidism has no effect and failure to thrive despite hunger and
on obstetric outcome or neurological increased appetite. Infants may have a
development in offspring protuberant abdomen with thin limbs and
chest.
Answer & Comments
Management is with supplemental nasogastric
4- Treatment improves obstetric outcome feeding in addition to frequent intake of
but has no effect on neurological carbohydrate-containing foods. Patients with
development in offspring this condition cannot metabolise fructose or
lactose and should avoid foods containing
Subclinical hypothyroidism (serum TSH these.
concentration above the upper limit of the
reference range with a normal free T4) is Table 1 lists enzyme deficiencies associated
associated with adverse outcomes for both with glycogen storage disorders. Types II, III, V
mother and offspring. Levothyroxine therapy and VII are associated with clinically significant
has been shown to improve obstetric outcome, myopathy.
but there are no proven beneficial effects on Eponymo
long-term neurological development in the GSD
us Enzyme
offspring. type
name deficiency

Further reading

76 | Dr. Khalid Yusuf (FB: Sohag Endocrine Group)


SCE Endocrinology (sce.practicaldiabetes.com), 2017

Von Glucose-6- NICE guidance recommends that capillary


I blood glucose levels are monitored hourly
Gierke phosphatase
during labour, with the aim of maintaining
II Pompe Acid maltase blood glucose between 4 and 7 mmol/litre.
Capillary blood glucose should be measured
Forbes Debranching every 30 minutes if a general anaesthetic is
III
Cori enzyme required. Intravenous dextrose and insulin
should be considered if blood glucose falls
Andersen Transglucosidase outside these values.
IV
disease deficiency
Further reading
Muscle National Institute for Health and Clinical
V McArdle Excellence. CG63 Diabetes in Pregnancy.
phosphorylase
Reissued July 2008.
Hepatic http://guidance.nice.org.uk/CG63/NICEGuida
VI Hers nce/pdf/English.
phosphorylase

Phosphofructoki Q 116
VII Tarui
nase
A 53-year-old male parcel courier attends the
diabetes outpatient clinic for review. He
Further reading manages his diabetes with a combination of
Bali DS, et al. Glycogen Storage Disease Type I. diet, metformin therapy and a DPP-IV inhibitor,
Includes: Glycogen Storage Disease Type Ia,
but his control has deteriorated of late. His
Glycogen Storage Disease Type Ib. Initial
HbA1c has increased from 58mmol/mol (7.5%)
posting: 19 April 2006; last update: December
23, 2010. to 65mmol/mol (8.1%). He has a history of
www.ncbi.nlm.nih.gov/books/NBK1312/. hypertension, mixed dyslipidaemia,
osteoarthritis and newly diagnosed
background retinopathy. He has discussed a
Q 115
weight reducing diet with the dietician and has
been advised to increase his activity levels,
A 35-year-old lady with gestational diabetes is which he finds difficult because of the time he
admitted to the labour ward at 38 weeks for spends driving for his employer. He is
induction of labour. concerned about increasing weight and has
According to NICE guidelines, how frequently marked central obesity with a BMI of 31.4.
should her blood sugars be monitored during Which of the following therapeutic
labour? interventions is most appropriate?
1- Every 15 minutes 1- Consider use of a GLP-1 agonist and
2- Every 30 minutes discontinue DPP-IV inhibitor

3- Every hour 2- Commence on a twice-daily


sulphonylurea with advice to eat regular
4- Every two hours carbohydrate containing meals and
5- If she develops symptoms of snacks
hypoglycaemia 3- Use a once-daily long acting insulin with
oral hypoglycaemic agents
Answer & Comments 4- Commence on twice daily premixed
3- Every hour biphasic insulin with advice to eat

77 | Dr. Khalid Yusuf (FB: Sohag Endocrine Group)


SCE Endocrinology (sce.practicaldiabetes.com), 2017

regular carbohydrate containing meals and he has stable graft function with the
and snacks following additional investigations:
5- Continue lifestyle measures and dietary
Creatinine 110µmol/l
restriction with current medication
eGFR 68ml/min/1.73m2
Answer & Comments
Hb 11g/dl
1- Consider use of a GLP-1 agonist and
discontinue DPP-IV inhibitor Calcium (adj) 2.30mmol/l

This gentleman’s occupation requires that he is What should you advise regarding
able to drive and thus avoidance of management of his diabetes?
hypoglycaemia is an important consideration. 1- Refer to diabetes specialist nurses to
In addition, weight loss is likely to benefit his commence insulin therapy in view of
other medical conditions and overall renal dysfunction
cardiovascular risk. GLP-1 agonists are
recommended for people with type 2 diabetes 2- Commence on low dose metformin
and BMI <35 if insulin therapy has significant therapy and monitor levels
occupational implications and weight loss may 3- Stop prednisolone in view of
benefit other co-morbidities. Continued hyperglycaemia and increase tacrolimus
lifestyle intervention is important but
4- Advise regarding dietary intervention,
improvement in glycaemic control is necessary
weight loss and exercise
in view of newly diagnosed microvascular
complications. 5- Target antihypertensive therapy to BP
<130/80 and check urine
Further reading microalbumin:creatinine ratio
National Institute for Health and Clinical
Excellence. CG 87 Type 2 Diabetes – newer
Answer & Comments
agents (partial update of CG 66). Issued May
2009.
4- Advise regarding dietary intervention,
http://guidance.nice.org.uk/CG87/NICEGuida
weight loss and exercise
nce/pdf/English.
New Onset Diabetes Mellitus After Transplant
Q 117 (referred to as NODAT) is increasingly
recognised and has a cumulative incidence of
The renal registrar contacts you for advice 25% three years post renal transplant. It is
regarding a 55-year-old male who developed associated with a significant reduction in graft
end-stage renal disease secondary to chronic and patient survival. It is more common in
glomerulonephritis and hypertension. He older patients and those of Afro-Caribbean
underwent cadaveric renal transplant four race. Other risk factors include obesity, use of
years previously and is maintained on prednisolone therapy, graft rejection episodes
prednisolone 5mg once daily, tacrolimus 2mg and hepatitis C infection. Calcineurin inhibitors,
twice daily and mycophenolate 1g twice daily. particularly tacrolimus, are associated with
He also takes doxazosin and ramipril for development of NODAT and his
hypertension. He is found to have glycosuria immunosuppressant regime should be
when he attends renal outpatient clinic, thus reviewed if there is difficulty in achieving
HbA1c is requested and found to be glycaemic control. Although control of blood
57mmol/mol (7.4%). He is overweight with a pressure is important in both renal disease and
BMI of 29. His BP is 148/88 with no proteinuria, diabetes, changes to antihypertensive
medication would not usually be

78 | Dr. Khalid Yusuf (FB: Sohag Endocrine Group)


SCE Endocrinology (sce.practicaldiabetes.com), 2017

recommended on the basis of a single clinic practice guideline. Journal of Clinical


measurement. Endocrinology and Metabolism 2010;
95(9):4133–60. doi 10.1210/jc.2009-2631.
Further reading
Davidson J, Wilkinson A et al. NODAT. Q 119
Transplantation 2003; 75:S23–24.
A 69-year-old man presents to the accident and
Q 118 emergency department with severe back pain
following a fall from standing height. Thoracic
A 32-year-old woman with mild facial hirsutism spine X-ray confirms a vertebral wedge
has been referred to the endocrine clinic with fracture. He is a non-smoker, denies excess
a suspected diagnosis of polycystic ovarian alcohol consumption and has no personal or
syndrome. She is not distressed by this family medical history of significance.
symptom, and waxes less than once weekly. Investigations are as follows:
She has two healthy children and is taking an
oral contraceptive pill as she has no current Calcium (adj) 2.32mmol/L (2.20–2.60)
plans for further pregnancy. Her 17-OHP levels
are found to be elevated and you have arrived ALP 40 iU/L (40–125)
at a diagnosis of non-classical Congenital
PTH 4.3 pmol/L (1–6.1)
Adrenal Hyperplasia. She does not have any
other symptoms.
TSH 1.2 mU/L (0.5–6)
What treatment would you advise?
Free testosterone 7.0 nmol/L (9–42)
1- None
2- Hydrocortisone LH 8.6 U/L (3–8)

3- Dexamethasone FSH 7.2 U/L (0.5–5)


4- Prednisolone
Prolactin 120 mU/L (<450)
5- Spironolactone
MRI Pituitary No abnormality seen
Answer & Comments Which of the following treatments have not
1- None been shown to reduce the risk of fragility
fracture in men?
Endocrine Society guidelines do not 1- Calcium and vitamin D replacement
recommend treatment of non-classical
Congenital Adrenal Hyperplasia (NCCAH) in 2- Bisphosphonate therapy
asymptomatic adults. Indications for treatment 3- Testosterone replacement
of adults with NCCAH include subfertility or
hyperandrogenism which is of significance to 4- Teriparatide
the patient. Recommendations for treatment 5- Regular weight bearing exercise
of children with NCCAH include overt
virilisation, inappropriately early onset of
Answer & Comments
pubarche, rapid progression of pubarche and
advanced bone age. 3- Testosterone replacement

Further reading Bisphosphonates, calcitonin and teriparatide


Speiser PW, Azziz R et al. Congenital adrenal have been shown to reduce the risk of
hyperplasia due to steroid 21-hydroxylase vertebral fragility fractures in men.
deficiency: an Endocrine Society clinical

79 | Dr. Khalid Yusuf (FB: Sohag Endocrine Group)


SCE Endocrinology (sce.practicaldiabetes.com), 2017

Teriparatide is used as a second line therapy for Free T4 19.3 pmol/L (10-23.0)
patients who are unable to tolerate
bisphosphonates or continue to have fractures Free T3 3.5 pmol/L (3.5-7.0)
after 1 year of bisphosphonate therapy. Which is the most appropriate intervention?
Although there is evidence that testosterone
supplementation increases bone mineral 1- Check thyroid peroxidase antibodies and
density, there is no evidence of a reduction in consider thyroid hormone replacement
fragility fractures. Use of calcium with vitamin if positive
D supplementation is also recommended, 2- Commence on levothyroxine therapy to
although the evidence suggests only a modest improve lipid profile and glycaemic
reduction in the risk of further fractures. control
Regular weight bearing exercise has been
shown to reduce the risk of falls and increase 3- Consider tri-iodothyronine in view of low
bone mineral density in both men and women. free T3
4- Repeat thyroid function tests and
Further reading commence on levothyroxine if TSH
Qaseem A, Snow V et al. Pharmacologic persistently elevated
Treatment of Low Bone Density or
Osteoporosis to Prevent Fractures: A Clinical 5- If thyroid function tests remain stable no
Practice Guideline from the American College further intervention is required
of Physicians. Ann Intern Med 2008; 149(6):
404–15.
Answer & Comments

Q 120 5- If thyroid function tests remain stable no


further intervention is required
A 79-year-old man with type 2 diabetes attends
the outpatient clinic for review. He has a Amiodarone inhibits 1 5’-deiodinase enzyme
previous medical history of a myocardial activity, which is responsible for peripheral
infarction with subsequent angina despite conversion of thyroxine (T4) to
coronary artery bypass grafting, severe left triiodothyronine (T3). Consequently, serum
ventricular dysfunction, hypertension and levels of T4 and rT3 increase and serum levels
dyslipidaemia. He has recently been of T3 decrease by 20-25%. TSH levels rise on
commenced on amiodarone and has an initiation of therapy and usually return to
implantable defibrillator following an out of normal over a period of months.
hospital VF arrest three months ago. His
diabetes has previously been well controlled Further reading
Bogazzi F, Tomisti L et al. Amiodarone and the
using metformin and premixed biphasic
thyroid: a 2012 update. J Endocrinol Invest
insulin. Investigations are as follows: 2012; 35(3):340–348. doi:10.3275/8298. Epub
2012 Mar 19.
HbA1c 60 mmol/mol

Total Chol 4.9 mmol/L Q 121

HDL 1.0 mmol/L A 19-year-old man with Asperger’s syndrome


attends the diabetes outpatient clinic for
Creat 154 µmol/L review. He lives independently in a supported
living facility, preparing his own meals and
ACR <2.5 mg/mmol administering his own insulin. He has a fixed
routine and keeps meticulous records of his
TSH 7.0 mU/L (0.2-6.0)
dietary intake, capillary blood glucose records,
insulin doses and activity levels. He maintains

80 | Dr. Khalid Yusuf (FB: Sohag Endocrine Group)


SCE Endocrinology (sce.practicaldiabetes.com), 2017

weekly contact with the diabetes specialist Further reading


nurses who help him with titration of his insulin National Institute for Health and Clinical
doses as necessary because he reports he is not Excellence. CG 15: Type 1 diabetes: Diagnosis
ready to undertake a carbohydrate counting and management of type 1 diabetes in
course. He converted from a twice-daily, pre- children, young people and adults. July 2004.
http://www.nice.org.uk/cg15.
mixed insulin regimen to a basal bolus regimen
just over a year ago to see if his control could Heller S, et al. Comparison of insulin detemir and
be improved with greater flexibility of dosing, insulin glargine in a basal-bolus regimen, with
insulin aspart as the mealtime insulin, in
but admits that he forgets his 10pm insulin
patients with type 1 diabetes: a 52-week,
detemir approximately once or twice weekly.
multinational, randomized, open-label,
He has no complications and does not report parallel-group, treat-to-target non-inferiority
hypoglycaemia, but his overall glycaemic trial. Clin Ther 2009;31(10):2086–97.
control has deteriorated since commencing on
basal bolus dosing.
Q 122
What is the next most appropriate step?
1- Arrange for district nurses to administer A 46-year-old man of no fixed abode is
his long-acting insulin in the morning admitted 1 week following an alleged assault
during which he sustained a fracture to his
2- Change his detemir insulin to glargine right hand. He complains of a 2-day history of
insulin dizziness, intermittent confusion and general
3- Encourage him to undertake a malaise. He has a previous medical history of
personalised carbohydrate counting new-onset diabetes after renal transplant
course (NODAT), for which he has been prescribed
pre-mixed biphasic insulin 3 times daily in
4- Refer for consideration of a continuous addition to his immunosuppressant therapy.
subcutaneous insulin infusion pump On examination, he has a GCS of 15 with no
5- Revert to a twice-daily, pre-mixed focal neurological signs. He is unkempt and
biphasic insulin regimen dehydrated but general examination is
otherwise unremarkable. His investigations are
Answer & Comments as follows:

5- Revert to a twice-daily, pre-mixed Sodium: 127mmol/L


biphasic insulin regimen
Potassium: 4.4mmol/L
All of the options should be considered, but the
most appropriate is to simplify this patient’s Urea: 12.5mmol/L
insulin dosing schedule while allowing him to
Creatinine: 186mmol/L
maintain his independence and ability to self-
care. Twice-daily, pre-mixed biphasic insulin
Glucose: 34.5mmol/L
regimens can be very effective for people who
lead relatively regular lifestyles, and reducing Urinary ketones: +
the frequency of injections is likely to improve
his compliance. His reluctance to undertake BP: 143/88mmHg, no postural change
carbohydrate counting despite intensive
support from the specialist nursing team FBC: normal
makes him less likely to benefit from the
flexibility of an insulin pump. There is no LFTs: normal
evidence that switching between long-acting
insulins has any effect on overall glycaemic CRP: 26mg/L
control.

81 | Dr. Khalid Yusuf (FB: Sohag Endocrine Group)


SCE Endocrinology (sce.practicaldiabetes.com), 2017

Chest X-ray: no acute pulmonary pathology regimen resulted in improvement of both


hyperglycaemia and hyponatraemia within 48
CT brain: no acute changes hours.
What is the most likely cause of his
hyponatraemia? Further reading
Roscoe JM, et al. Hyperglycemia-induced
1- Diabetic ketoacidosis requiring hyponatremia: metabolic considerations in
intravenous insulin and urgent calculation of serum sodium depression. Can
replacement of fluid and electrolytes Med Assoc J 1975;112:452–3.

2- Hyperglycaemic hyperosmolar state,


requiring intravenous insulin and Q 123
cautious replacement of fluid and
electrolytes A 67-year-old woman with poorly-controlled
type 2 diabetes complicated by retinopathy,
3- SIADH (syndrome of inappropriate obesity, hypertension and dyslipidaemia is
diuretic hormone) following head injury, admitted for treatment of an acute
requiring fluid restriction exacerbation of COPD. She also complains of an
4- Compensatory hyponatraemia due to acutely painful, red right eye. You note
non-compliance with insulin and rubeosis iridis on ophthalmoscopic
subsequent hyperglycaemia examination and refer her urgently for
ophthalmology review for suspected
5- Acute adrenal insufficiency with stress-
neovascular glaucoma.
induced hyperglycaemia
Which of the following features of diabetic
Answer & Comments retinopathy is most strongly associated with
development of rubeosis iridis and neovascular
4- Compensatory hyponatraemia due to glaucoma?
non-compliance with insulin and
subsequent hyperglycaemia 1- Microaneurysms
2- Dot haemorrhages
Risk factors and approach to management of
NODAT (new-onset diabetes after transplant) 3- Blot haemorrhages
are similar to type 2 diabetes. The degree of 4- Cotton wool spots
ketonuria is insufficient to diagnose diabetic
5- Intra-retinal microvascular anomalies
ketoacidosis which is usually associated with
(IRMAs)
insulin deficiency states. Similarly, the patient’s
osmolality is too low for a diagnosis of
hyperglycaemic hyperosmolar state, which is Answer & Comments
diagnosed when calculated or measured serum
3- Blot haemorrhages
osmolality is >320mOsmol/kg. (Calculated
osmolality 2 x [Na+ + K+] + urea + glucose =
Blot haemorrhages are considered to
309.8mOsmol/kg. Some calculations disregard
represent deep retinal infarcts caused by
the serum potassium, in which case calculated
capillary occlusion. Large, peripheral blot
osmolality will be lower.) Acute adrenal
haemorrhages are a common feature of ocular
insufficiency usually results in hypoglycaemia.
ischaemia and may be associated with rubeosis
iridis and neovascular glaucoma.
The patient admitted non-compliance with
insulin due to pain, swelling and reduced
Dot haemorrhages, which are more superficial
function of his dominant right hand. Oral
than blot haemorrhages, are features of non-
rehydration and assistance with administration
proliferative diabetic retinopathy. They cannot
and titration of his insulin using a twice-daily

82 | Dr. Khalid Yusuf (FB: Sohag Endocrine Group)


SCE Endocrinology (sce.practicaldiabetes.com), 2017

always be differentiated from microaneurysms patient takes Adcal D3 Forte 1 tablet twice-
on clinical examination, thus the term dot daily for osteopaenia.
haemorrhage/microaneurysm (H/Ma) may be
What is the next most appropriate diagnostic
applied.
step?
Cotton wool spots are thought to represent 1- Stop her calcium supplement and repeat
swollen axons due to interruption of calcium and PTH levels
axoplasmic flow by retinal infarct. They are not
exclusive to diabetic retinopathy and do not 2- Measure 24-hour total urinary calcium
appear to increase the risk of new vessel excretion
formation; thus, unless extensive areas are 3- Measure spot urinary calcium level
found, they are considered to represent non-
4- Measure 24-hour urinary calcium,
proliferative retinopathy.
creatinine and oxalate excretion
The collective term intra-retinal microvascular 5- Take a full family history to ascertain the
abnormalities or ‘IRMAs’ describes the presence of familial hypercalcaemia
tortuous microvascular abnormalities thought
to be dilated capillary remnants which occur Answer & Comments
due to occlusion of the capillary network. An
alternative explanation is that IRMAs are a 4- Measure 24-hour urinary calcium,
variant of collateral formation in response to creatinine and oxalate excretion
an ischaemic stimulus.
Although stopping calcium supplements is an
Further reading appropriate therapeutic intervention, excess
The Royal College of Ophthalmologists. Diabetic exogenous calcium should cause suppression
retinopathy guidelines. December 2012. of parathyroid hormone levels; thus, the most
www.rcophth.ac.uk/core/core_picker/downl likely cause of this patient’s hypercalcaemia is
oad.asp?id=1533. primary hyperparathyroidism.

Q 124 The most sensitive and specific diagnostic test


is calculation of the calcium:creatinine ratio in
A 72-year-old woman presents to accident and a 24-hour urine collection. Total urinary
emergency with dysuria, fever and loin pain. calcium is elevated in only 40% of patients with
Growth of Escherichia coli is demonstrated on primary hyperparathyroidism, and some
a mid-stream sample of urine and she is patients with familial hypocalciuric
commenced on appropriate antimicrobial hypercalcaemia (FHH) will have normal or even
therapy. She is also under the care of the elevated total 24-hour urinary calcium levels. A
urology team, having recently undergone single on-the-spot urinary calcium:creatinine
bilateral vesico-ureteric junction stenting to ratio is likely to have similar sensitivity and
allow passage of multiple renal calculi because specificity to a 24-hour urinary
of recurrent renal colic. Review of her blood calcium:creatinine ratio, but there is currently
investigations demonstrates persistent mild insufficient evidence to support routine use.
elevation of corrected serum calcium between
2.67 and 2.95mmol/L for several months Both primary hyperparathyroidism and FHH
(range 2.13–2.63). Her parathyroid hormone exhibit a familial tendency; thus, family history
(PTH) level is 4.2pmol/L (range 1.1–6.9), and would not reliably differentiate the 2
vitamin D levels are 66nmol/L (48–144). conditions.
Alkaline phosphatase, serum albumin and
serum phosphate levels are normal. The Further reading
Shinall MC, et al. Differentiating familial
hypocalciuric hypercalcemia from primary

83 | Dr. Khalid Yusuf (FB: Sohag Endocrine Group)


SCE Endocrinology (sce.practicaldiabetes.com), 2017

hyperparathyroidism. Endocr Pract than 2.5mU/L in the first trimester or 3.0mU/L


2013;20:1–19. [Epub ahead of print.] in the second and third trimester, or to
trimester-specific normal ranges for TSH.
Q 125 Thyroid function tests should be re-measured
within 30–40 days.
You are contacted by the obstetrics team for
If a woman is diagnosed with hypothyroidism
advice regarding a 32-year-old woman who is
prior to pregnancy, her thyroxine dose should
28 weeks’ pregnant. Her thyroid function tests
be adjusted to reach a TSH level not greater
were checked due to complaints of cold
than 2.5mU/L prior to conceiving.
intolerance and constipation. She was noted to
be bradycardic with a regular heart rate of
54bpm. She has no skin changes nor goitre. Her Further reading
Abalovich M, et al. Management of thyroid
TSH is 6.3mU/L (normal range 0.5–6.0) and her
dysfunction during pregnancy and
free T4 6.0pmol/L (normal range 10.0–23.0). postpartum: an Endocrine Society Clinical
What would you advise? Practice Guideline. J Clin Endocrinol Metab
2007;92(8 Suppl):S1–S47.
1- Commence on levothyroxine with the
intention of rapidly titrating TSH to not
Q 126
higher than 3.0mU/L and recheck
thyroid function tests in 30–40 days
A patient with confirmed osteoporosis admits
2- Commence on levothyroxine with the that she has not been taking her oral
intention of rapidly titrating TSH to not bisphosphonate as she did not understand the
higher than 3.0mU/L and recheck advice on how to take it nor the possible side
thyroid function tests in 8–10 weeks effects.
3- Commence on levothyroxine with the Which of the following is not a recognised
intention of gradually titrating TSH to complication of bisphosphonate therapy?
not higher than 2.5mU/L and recheck
thyroid function tests in 6–8 weeks 1- Oesophageal erosions
4- Measure thyroid peroxidase antibodies 2- Oesophageal cancer
and commence on levothyroxine only if 3- Osteonecrosis of the jaw
positive
4- Atrial fibrillation
5- Repeat thyroid function tests in 6–8
weeks and commence on levothyroxine 5- Atypical femoral fractures
if biochemical picture persistent
Answer & Comments
Answer & Comments
2- Oesophageal cancer
1- Commence on levothyroxine with the
intention of rapidly titrating TSH to not There is insufficient evidence from
higher than 3.0mU/L and recheck epidemiological studies to support a causal
thyroid function tests in 30–40 days association between oral bisphosphonate use
and oesophageal cancer. However, as a causal
If overt hypothyroidism is diagnosed during relationship cannot be excluded, caution is
pregnancy, thyroid function tests should be advised in patients with known Barrett’s
normalised as soon as possible to prevent oesophagus. Oral bisphosphonates are
adverse obstetric and fetal outcomes. associated with severe oesophageal adverse
Thyroxine dosage should be titrated rapidly to reactions including oesophagitis, oesophageal
maintain serum TSH concentrations of less ulcers, oesophageal strictures, and
oesophageal erosions. The risk of oesophageal

84 | Dr. Khalid Yusuf (FB: Sohag Endocrine Group)


SCE Endocrinology (sce.practicaldiabetes.com), 2017

side effects is reduced if the patient adheres to a family history of type 2 diabetes and her BMI
dosing instructions. The risk of is 31kg/m2.
bisphosphonate-associated osteonecrosis is
What would you advise?
greatest in those taking intravenous
preparations, but it is recommended that any 1- Request a fasting blood glucose and
patient with poor oral health should undergo offer an oral glucose tolerance test if
dental review prior to starting oral elevated
bisphosphonates.
2- Arrange an urgent oral glucose tolerance
test
A review of clinical trial data demonstrates an
increased risk of atrial fibrillation in patients 3- Offer self-monitoring of blood glucose or
taking zoledronic acid, pamidronate and an oral glucose tolerance test between
possibly alendronate. However, the risk is low 16–18 weeks’ gestation
and the consensus of opinion is that benefits of
4- Arrange an oral glucose tolerance test
bisphosphonate therapy are likely to outweigh
between 24–28 weeks’ gestation
the risk of atrial fibrillation. This association
remains under review. 5- Monitor fasting capillary blood glucose
and 1-hour postprandial capillary blood
Bisphosphonate use is associated with atypical glucose with lifestyle intervention as
femoral fractures which may occur unilaterally appropriate
or bilaterally with no history of trauma. The
patient should be advised to report any new, Answer & Comments
persistent thigh, hip or groin pain and both hips
should be examined if an atypical fracture is 4- Arrange an oral glucose tolerance test
suspected. between 24–28 weeks’ gestation

Further reading NICE recommends screening for gestational


Medicines and Healthcare Products Regulatory diabetes in the healthy population based on
Agency (MHRA). Information on assessment of the following risk factors:
bisphosphonates for healthcare professionals.
Oct 2012. • BMI >30kg/m2.
www.mhra.gov.uk/Safetyinformation/Genera
lsafetyinformationandadvice/Product- • Previous macrosomic baby (>4.5kg).
specificinformationandadvice/Product-
specificinformationandadvice-A- • Previous gestational diabetes.
F/Bisphosphonates/index.htm#l2.
• First-degree relative with diabetes.
Q 127
• Family origin with a high prevalence of
You are contacted for advice by the obstetric diabetes, e.g. South Asian, Black Caribbean or
team regarding a 29-year-old Pakistani woman Middle Eastern.
who has been referred to the antenatal clinic.
Women with any one of these risk factors
Her ultrasound scan confirms that she is
should be offered testing for gestational
pregnant with a live, single fetus of
diabetes.
approximately 10 weeks’ gestation. This is her
second pregnancy, having previously delivered
Women who have had gestational diabetes
a healthy baby boy weighing 3.1kg with no
in a previous pregnancy should be offered early
complications. She has no previous medical
self-monitoring of blood glucose or an oral
history of note and takes no regular medication
glucose tolerance test at 16–18 weeks, and a
except for folic acid during pregnancy. She has

85 | Dr. Khalid Yusuf (FB: Sohag Endocrine Group)


SCE Endocrinology (sce.practicaldiabetes.com), 2017

further oral glucose tolerance test at 28 weeks Urine ketones ++


if the results are normal.
Capillary ketones: unavailable
Women with any of the other risk factors for
gestational diabetes should be offered an oral He was commenced on a variable rate
glucose tolerance test at 24–28 weeks. intravenous insulin infusion and transferred to
a medical ward where you have been asked to
Screening for gestational diabetes using fasting review him. His capillary blood glucose is now
plasma glucose, random blood glucose, in the range of 12–15mmol/L and he is eating
glucose challenge test and urinalysis for and drinking. He has no ketonuria.
glucose should not be undertaken.
What is the next most appropriate step?
Further reading 1- Commence on a fixed-rate, glucose-
National Institute for Health and Clinical potassium-insulin infusion
Excellence. CG 63: Diabetes in pregnancy.
March 2008.
2- Commence on basal bolus insulin
www.nice.org.uk/nicemedia/pdf/CG063Guida regimen
nce.pdf. 3- Commence on pre-mixed biphasic
insulin regimen
Q 128 4- Commence on oral hypoglycaemic
agents
A 57-year-old HGV driver is admitted to the
medical assessment unit by his GP after he 5- Stop all glucose-lowering therapies and
presented with osmotic symptoms and was monitor
found to have a capillary glucose reading of ‘HI’
at the surgery. He denies any recent weight Answer & Comments
loss or abdominal symptoms. He has a previous
medical history of hypertension which is well 4- Commence on oral hypoglycaemic
controlled on amlodipine, does not take agents
alcohol and is a non-smoker. He has annual
checks with his GP and had been found to have This patient has features of the metabolic
a marginally elevated fasting blood glucose syndrome including obesity and hypertension
level 4 months previously for which he received and is most likely to have type 2 diabetes, thus
advice regarding lifestyle intervention. His metformin is likely to be the most useful first-
initial investigations were as follows: line therapy.

BMI 35kg/m2 In this case, a combination of metformin and


a sulphonylurea resulted in acceptable
BP 128/74mmHg lowering of his capillary blood glucose levels
with no further ketonuria. It is worth
Sodium 132mmol/L remembering that other causes of ketonuria
include high-protein/low-carbohydrate diets,
Potassium 4.2mmol/L high alcohol intake, fasting states and
conditions associated with increased
Urea 8.4mmol/L metabolism including fever, pregnancy,
lactation and hyperthyroidism.
Creatinine 87mmol/L
Further reading
Bicarbonate 35mmol/L National Institute for Health and Clinical
Excellence. CG 87: Type 2 diabetes: the
Glucose 35mmol/L management of Type 2 diabetes. May 2009.

86 | Dr. Khalid Yusuf (FB: Sohag Endocrine Group)


SCE Endocrinology (sce.practicaldiabetes.com), 2017

http://publications.nice.org.uk/type-2- including pancreatic cancer, account for


diabetes-cg87. approximately half of cases.

Q 129 Although this patient is likely to be relatively


insulin deficient, it is not appropriate to
commence her on a variable rate intravenous
A 91-year-old, frail woman is commenced on
insulin infusion as she does not have clinical
metformin for newly-diagnosed type 2
features consistent with diabetic ketoacidosis;
diabetes. However, she becomes unwell on
however, she may benefit from subcutaneous
treatment and is admitted to hospital for
insulin rather than oral hypoglycaemic agents
further evaluation. She is dehydrated,
if she becomes hyperglycaemic after
nauseous and vomiting and her capillary
management of her initial presentation. She
glucose is 5.8mmol/L with mild ketonuria. You
may have an elevated lactate due to
note she is cachectic with recent weight loss,
dehydration and peripheral hypoperfusion.
and on examination of her abdomen you find it
However, initial treatment of an elevated
is slightly protuberant though soft and non-
lactate would include intravenous fluid therapy
tender with a palpable firm, irregular nodule
and basic resuscitation measures which should
causing protrusion of the umbilicus.
precede any assessment for admission to
What is the next most appropriate step? critical care – a decision which should be
1- Give antiemetic and intravenous fluid tempered by the high likelihood of confirming
rehydration and assess response advanced metastatic malignancy.

2- Advise that she is kept ‘nil-by-mouth’ Further reading


and request an urgent surgical opinion Urbano FL. Sister Joseph’s Nodule. Hosp Physician
2001;44:33–5.
3- Commence a variable rate intravenous
insulin infusion with fluid and electrolyte
replacement and monitoring of capillary Q 130
glucose and ketones
4- Substitute usual preparation of A 65-year-old man is referred to the endocrine
metformin with slow-release metformin clinic following a recent presentation to the
at a lower dose medical assessment unit with lethargy and
muscle weakness. He has a previous medical
5- Check lactate levels and refer for urgent history of well-controlled type 2 diabetes
critical care opinion (HbA1c 46mmol/mol [6.4%]), CKD 3,
hypertension, gastro-oesophageal reflux
Answer & Comments disease and hypercholesterolaemia for which
he takes irbesartan 300mg, omeprazole 20mg,
1- Give antiemetic and intravenous fluid simvastatin 40mg, gliclazide 80mg twice daily
rehydration and assess response and bendroflumethiazide 2.5mg. His
investigations were as follows:
It is unusual for a non-obese elderly patient to
develop type 2 diabetes as a consequence of Sodium 136mmol/L
insulin resistance or metabolic syndrome, thus
underlying pancreatic pathology should be Potassium 4.1mmol/L
considered. The firm nodule in the umbilicus
represents a ‘Sister Mary Joseph’ nodule, Urea 9.8mmol/L
which is a palpable metastatic deposit usually
associated with a pelvic or abdominal Creatinine 142mmol/L
malignancy. Gastrointestinal malignancies,
Corrected calcium 1.36mmol/L

87 | Dr. Khalid Yusuf (FB: Sohag Endocrine Group)


SCE Endocrinology (sce.practicaldiabetes.com), 2017

Parathyroid hormone 5.4pmol/L Malabsorptive states such as coeliac disease


may be associated with hypocalcaemia, but
Magnesium 0.21mmol/L there is usually a compensatory rise in
parathyroid hormone.
Vitamin D levels: awaited
Further reading
While in the medical assessment unit his Hess MW, et al. Systematic review:
electrolytes were replaced intravenously and hypomagnesaemia induced by proton pump
he was discharged with a provisional diagnosis inhibition. Aliment Pharmacol Ther
of primary hypoparathyroidism. On reviewing 2012;36:405–13.
his previous blood tests you note that both his
calcium and magnesium levels have been Q 131
slightly below the normal reference range for
approximately 12 months. The patient tells you
A 36-year-old woman presents to her GP, with
he had a bout of winter vomiting virus about 2
weight loss, tremor and palpitations. Her
weeks before admission.
thyroid function tests were consistent with
What is the most appropriate next step in his thyrotoxicosis and thyroid receptor antibodies
management? were positive. She was commenced on carbim-
azole, but presented to her GP 2 weeks later
1- Check coeliac antibodies
complaining of discomfort and a sensation of
2- Commence on additional therapy to grittiness in her eyes.
improve glycaemic control
Which of the following clinical signs is an
3- Give high-dose oral calcium and vitamin indication for urgent referral to a specialist in
D supplementation thyroid eye disease?
4- Reassure patient that electrolyte losses 1- Chemosis
were probably due to gastroenteritis
2- Corneal opacity
5- Switch omeprazole and
bendroflumethiazide to alternative 3- Diplopia
therapies and monitor electrolyte levels 4- Periorbital swelling
5- Restriction of eye movements
Answer & Comments

5- Switch omeprazole and Answer & Comments


bendroflumethiazide to alternative
therapies and monitor electrolyte levels 2- Corneal opacity

Graves’ ophthalmopathy is usually mild and


Hypomagnesaemia inhibits release of
self-limiting, with only 3–5% of cases
parathyroid hormone which can result in
progressing to sight-threatening disease.
hypocalcaemia. Both thiazide diuretics and
Urgent referral to a specialist in thyroid eye
proton-pump antagonists may cause
disease is recommended for any patient with
hypomagnesaemia which in this case was
sudden, unexplained deterioration in vision or
exacerbated by increased gastrointestinal
colour vision, globe subluxation, obvious
losses
corneal opacity, papilloedema or visible cornea
during acute illness. Although poorly- with the eyelids closed. A non-urgent referral is
controlled diabetes may also cause appropriate for patients with light sensitivity,
hypomagnesaemia due to osmotic diuresis and grittiness despite topical lubricants, and
increased renal losses, this man’s HbA1c progressive change in appearance or diplopia.
indicates good control.

88 | Dr. Khalid Yusuf (FB: Sohag Endocrine Group)


SCE Endocrinology (sce.practicaldiabetes.com), 2017

Further reading mineral density in high-risk individuals is not


Bartalena L, et al. Consensus statement of the necessary prior to starting therapy. In
European Group on Graves’ Orbitopathy individuals <65 years of age without a previous
(EUGOGO) on management of GO. Eur J fragility fracture, assessment of bone mineral
Endocrinol 2008;158:273–85. density and fracture risk is recommended
before starting therapy, and treatment is
Q 132 recommended for individuals with a T-score of
-1.5 or less.
A 67-year-old woman is referred by a
respiratory physician for advice regarding Further reading
primary prevention of osteoporosis while on Glucocorticoid induced osteoporosis: Guidelines
long-term prednisolone. for prevention and treatment. London: Royal
College of Physicians, 2002.
Which of the following is recommended?
1- Measure bone mineral density and treat Q 133
with bisphosphonate if T-score <2.5
2- Measure bone mineral density and treat A 58-year-old man presented to A&E with
with bisphosphonate if T-score <1.5 central, crushing chest pain and was diagnosed
with a non-ST-elevation myocardial infarction
3- Commence bisphosphonate without (NSTEMI). He had no previous medical history
assessment of bone mineral density of diabetes, but his venous glucose on
4- Commence on calcium and vitamin D admission was 16mmol/L, and remained
supplementation and measure bone persistently elevated over the next 24 hours.
mineral density if glucocorticoid Which of the following would you recommend?
treatment continues for >1 year
1- Intensive glucose control with a variable
5- Give advice regarding lifestyle rate insulin infusion
modification and measure bone mineral
density if glucocorticoid treatment 2- Intensive glucose control with an
continues for >1 year intravenous infusion of insulin, glucose
and potassium
Answer & Comments 3- Oral metformin therapy and dietary
advice
3- Commence bisphosphonate without
assessment of bone mineral density 4- Further monitoring and observation of
probable stress-induced hyperglycaemia
Loss of bone mineral density associated with 5- Subcutaneous insulin therapy with dose
oral glucocorticoid use is greatest in the first adjustment according to capillary blood
few months of glucocorticoid administration. glucose
Glucocorticoid-induced osteoporosis is also
associated with an increased fracture risk over
Answer & Comments
and above the effects of bone mineral density.
Individuals at high risk, for example, those aged 1- Intensive glucose control with a variable
over 65 years or with a previous history of rate insulin infusion
fragility fracture, should commence on bone-
protective therapy at the time of commencing NICE guidelines recommend first-line use of a
on glucocorticoid therapy. Bisphosphonates dose-adjusted intravenous insulin infusion for
are recommended as first-line therapy unless hyperglycaemia in the context of acute
there are contraindications; calcium and cornonary syndromes (ACS), which includes ST-
vitamin D supplementation is considered an elevation myocardial infarction (STEMI),
adjunct to treatment. Assessment of bone

89 | Dr. Khalid Yusuf (FB: Sohag Endocrine Group)


SCE Endocrinology (sce.practicaldiabetes.com), 2017

NSTEMI and unstable angina. The aim of such 1- Continue variable rate insulin infusion at
therapy is to maintain blood glucose levels a reduced rate with concurrent dextrose
below 11mmol/L, with avoidance of infusion
hypoglycaemia. NICE does not recommend
2- Stop variable rate insulin infusion and
routinely offering ‘intensive insulin therapy’
commence on fixed rate glucose-
(i.e. intravenous infusion of insulin and glucose
potassium-insulin (GKI) infusion
with or without potassium) in the context of
ACS. 3- Stop variable rate insulin infusion and
recommence oral metformin therapy
Further reading 4- Stop variable rate insulin infusion and
National Institute for Health and Clinical commence on pre-mixed biphasic
Excellence. CG 130. Management of
subcutaneous insulin therapy at 15 units
hyperglycaemia in acute coronary syndromes.
www.nice.org.uk/nicemedia/live/13589/5681 twice daily
7/56817.pdf. 5- Stop variable rate insulin therapy and
monitor capillary blood glucose with
Q 134 measurement of ketones if capillary
glucose >15mmol/L
You are asked to review an 82-year-old woman
48 hours after she was admitted under the care Answer & Comments
of the surgical team with sepsis due to hepatic
abscess, which is responding well to antibiotic 5- Stop variable rate insulin therapy and
therapy and percutaneous drainage. She had monitor capillary blood glucose with
been discharged from hospital 1 month measurement of ketones if capillary
previously following an episode of gallstone glucose >15mmol/L
induced pancreatitis, and has a previous
medical history of type 2 diabetes complicated This patient has developed stress-induced
by hypertension and background retinopathy. hyperglycaemia as a consequence of severe
She was reviewed in the diabetes outpatient infection, which has responded well to
clinic 2 weeks prior to admission and had been intravenous insulin infusion. She is at risk of
noted to have poor control on metformin relative insulin deficiency as a consequence of
therapy (HbA1c 64mmol/mol [8%]), thus was previous pancreatitis but, as her blood glucose
referred to the diabetes specialist nurse for is well controlled on a relatively low total
education and follow up regarding capillary insulin dose, it would be wise to monitor her
blood glucose monitoring. blood glucose before immediately
commencing on biphasic pre-mixed insulin. A
On admission, her blood glucose levels were fixed rate glucose-potassium-insulin (GKI)
recorded betweet 22mmol/L and ‘HI’ although infusion may be considered if she was fasted or
no ketones were detected, thus metformin was unable to eat. She is at risk of developing sepsis
discontinued and she was commenced on a and consequent lactic acidosis, thus it is
variable rate insulin infusion which resulted in recommended that metformin be withheld in
rapid lowering of her blood glucose levels. She the presence of another condition which may
is now eating small amounts and her blood predispose to lactic acidosis.
glucose is between 5.6 and 6.8mmol/L pre-
meals with ongoing intravenous insulin Further reading
infusion of 1 unit/hour. Magaji V, Johnston JM. Inpatient management of
hyperglycaemia and diabetes. Clin Diabetes
What should you advise the surgical team? 2011;29(1):3–9.

90 | Dr. Khalid Yusuf (FB: Sohag Endocrine Group)


SCE Endocrinology (sce.practicaldiabetes.com), 2017

Q 135 reduces glucose reabsorption by the kidneys


and increases urinary excretion of glucose. The
You review a 49-year-old man with type 2 manufacturer advises consideration of dose
diabetes, dyslipidaemia, spinal stenosis and reduction of sulphonylureas or insulin when
chronic back pain in the diabetes outpatient dapagliflozin is used in combination with these
clinic. He is obese with a BMI of 37, although agents due to the risk of hypoglycaemia.
his diabetes is relatively well controlled on Dapagliflozin increases diuresis, thus is not
once-daily long-acting insulin, twice-daily recommended in patients using loop diuretics.
sulphonylurea, slow-release metformin and a Temporary interruption of treatment is
DPP-4 inhibitor. He reports that he adheres to recommended for patients suffering volume
a low-calorie diet, and has previously been depletion who are at risk of acute kidney injury.
unable to tolerate orlistat or GLP-1 agonist Due to a lack of long-term data, and concerns
therapy due to bowel disturbance. He has about safety including increased incidence of
heard about a new medication for diabetes genito-urinary infection and bladder cancer, it
which is associated with weight loss and wishes is not currently recommended for use by NICE.
to know if it will be of benefit for him.
Further reading
Which of the following statements is not true of Regional Drug & Therapeutics Centre.
dapagliflozin? Dapagliflozin. New Drug Evaluation Dec 2012;
No 121.
1- Dapagliflozin reduces renal glucose
absorption and increases renal glucose
excretion by inhibiting the sodium- Q 136
glucose co-transporter 2 (SGLT 2)
2- Use of dapagliflozin with A 33-year-old Vietnamese ship maintenance
thiazolidinediones is not recommended worker is admitted with sudden onset of
due to the increased risk of bladder bilateral leg weakness and grade 3/5 power of
cancer with both drugs both lower limbs with no sensory disturbance
or sphincter dysfunction; deep tendon reflexes
3- Dapagliflozin is associated with an in his lower limbs are present but reduced. He
increase in urinary tract and genital has no previous medical or family history, and
infections has been visiting friends in the area. He is
4- During clinical trials, patients receiving afebrile with normal inflammatory markers
dapagliflozin for 24 weeks demonstrated although tachycardic with a resting heart rate
weight loss of 1.0–2.0kg of 104bpm. He has a small, soft, non-tender
goitre with no bruit. His investigations are as
5- Patients treated concurrently with follows.
insulin or sulphonylureas do not need
dose adjustment of their existing TSH <0.05mU/L (0.5–6mU/L)
medication
Free T4 74.3pmol/L (10–23.0pmol/L)
Answer & Comments
Thyroid peroxidase antibodies: Positive
5- Patients treated concurrently with
insulin or sulphonylureas do not need CSF white cell count <5 (<5/mm3)
dose adjustment of their existing
medication CSF protein 0.2g/L (0.15–0.45g/L)

Dapagliflozin is an orally active, competitive, CSF glucose 3.4mmol/L (2.2–4.5mmol/L)


reversible inhibitor of the human sodium-
Plasma glucose 4.5mmol/L (3.6–7.8mmol/L)
glucose cotransporter 2 (SGLT 2), which

91 | Dr. Khalid Yusuf (FB: Sohag Endocrine Group)


SCE Endocrinology (sce.practicaldiabetes.com), 2017

Which is the most appropriate intervention? medication, radioiodine or surgery to achieve


euthyroidism.
1- Treat thyroiditis with non-steroidal anti-
inflammatory drugs and supportive care Positive thyroid peroxidase antibodies are not
2- Seek and correct electrolyte disorders if pathognomonic of thyroiditis. FVC monitoring
necessary and commence on and IVIg would be appropriate for Guillain–
propranolol with antithyroid medication Barré syndrome (acute inflammatory
demyelinating polyneuropathy) which is
3- Give intravenous and oral potassium
characterised by ascending rather than abrupt
replacement and advise to avoid triggers
onset paralysis, and high CSF protein.
such as strenuous exercise, high
Thyrotoxic myopathy tends to affect proximal
carbohydrate or high-salt meals and
musculature but is usually of gradual onset.
extreme changes in temperature
4- Perform regular measurement of forced Further reading
vital capacity (FVC) and commence on Lam L, et al. Thyrotoxic periodic paralysis. Proc
intravenous immunoglobulins (IVIg) (Bayl Univ Med Cent) 2006;19(2):126–9.

5- Request electromyography to confirm


diagnosis of thyrotoxic myopathy Q 137

Answer & Comments The psychiatric SHO requests advice regarding


further investigation and management of a 50-
2- Seek and correct electrolyte disorders if year-old woman with mania and psychosis
necessary and commence on receiving treatment in an inpatient psychiatric
propranolol with antithyroid medication facility. She has a previous medical history of
obesity (BMI 35), alopecia and hypothyroidism
Thyrotoxic periodic paralysis (TPP) is for which she takes levothyroxine 150?g once
characterised by abrupt onset of paralysis of daily. Thyroid function tests were taken in view
the lower extremities, often associated with of her complaints of further hair loss and lower
hypokalaemia, with thyrotoxicosis. It is most limb swelling, and are as follows:
commonly seen in Asian men in their third to
fifth decades, and is curable once euthyroidism TSH 8.51mU/L (0.2–6.0mU/L)
is achieved. The neuromuscular presentation
of both TPP and autosomal dominant familial Free T4 12.5pmol/L (10–23pmol/L)
periodic paralysis (FPP) is identical, thus What would you advise?
confirmation of family history and features of
hyperthyroidism should be sought. FPP is more 1- Check Free T3 level
common in Caucasian populations and may be 2- Increase dose of levothyroxine and
triggered by strenuous exercise followed by repeat thyroid function in 6–8 weeks
abrupt rest, high-carbohydrate or high-salt
meals, temperature change and emotional 3- Continue current dose of levothyroxine
stress. and repeat thyroid function in 6–8 weeks
4- Recommend MR pituitary to exclude
Treatment of TPP with a non-selective beta- TSH-oma
blocker such as propranolol blunts the
hyperadrenergic stimulation of Na+/K+-ATPase 5- Commence on an oral triiodothyronine
and thus prevents intracellular shift of preparation
potassium and phosphate. Potassium
replacement should be given in low dose to Answer & Comments
reduce the chance of rebound hyperkalaemia.
Definitive therapy is with antithyroid 3- Continue current dose of levothyroxine
and repeat thyroid function in 6–8 weeks

92 | Dr. Khalid Yusuf (FB: Sohag Endocrine Group)


SCE Endocrinology (sce.practicaldiabetes.com), 2017

This pattern of thyroid function tests is Oestradiol <17pmol/L (17–260pmol/L,


consistent with poor compliance with therapy, follicular)
which is probable given the history of severe
mental health disorder requiring inpatient Testosterone 2.8nmol/L (1–2.5nmol/L)
admission and treatment. Thyroid function
tests should be rechecked after a period of 6–8 Sex hormone binding globulin (SHBG)
weeks; if TSH is static or increasing, the 32nmol/L (22–126nmol/L)
levothyroxine dose should be increased. It is
important to enquire whether the patient is Dehydroepiandrosterone (DHEAS) 9.2µmol/L
being treated with lithium therapy as lithium (4.9–9.4µmol/L)
inhibits thyroidal iodine uptake and thyroid
Androstenedione 4.3nmol/L (<3.8nmol/L)
hormone secretion, which would produce a
similar biochemical picture. Measurement of
17-hydroxyprogesterone (17-OHP) 32nmol/L
Free T3 level is not useful in management of
(<5nmol/L)
hypothyroidism. Patients with TSH-oma usually
have features of hyperthyroidism due to failure What is the next most appropriate
of negative feedback on TSH production. intervention?
1- Explain that she is entering the
Further reading
menopause and commence on hormone
England ML, Hershman JM. Serum TSH
concentration as an aid to monitoring replacement therapy (HRT) if
compliance with thyroid hormone therapy in symptomatic
hypothyroidism. Am J Med Sci 1986; 2- Request an MR pituitary
292(5):264–6.
Bochetta A, Loviselli A. Lithium treatment and
3- Stop antidepressant and repeat
thyroid abnormalities. Clin Pract Epidemiol prolactin
Ment Health 2006;2:23. Published online 2006 4- Commence on oral steroids to suppress
September 12. doi:10.1186/1745-0179-2-23. ACTHdriven production of androgens
and 17-OHP
Q 138
5- Perform pelvic ultrasound to confirm
suspected alternative diagnosis of
A 44-year-old woman with non-classic polycystic ovary syndrome (PCOS)
congenital adrenal hyperplasia (CAH) is being
followed up in the endocrine clinic. She has a
previous medical history of severe depression Answer & Comments
which has recently relapsed, although her 2- Request an MR pituitary
symptoms are improving with use of a selective
serotonin re-uptake inhibitor which she is The prolactin level is significantly elevated
extremely reluctant to discontinue. She does which may be consistent with a pituitary
not take any treatment for CAH and is tumour. As serum prolactin level generally
amenorrhoeic although otherwise correlates with the size of the tumour, this
asymptomatic. Her investigations are as level of hyperprolactinaemia may indicate a
follows: macroprolactinoma. Ideally, any drug with the
potential to cause hyperprolactinaemia should
FSH <0.5U/L (0.5–5.0U/L, follicular) be discontinued for 3 days prior to repeat
measurement of prolactin. However, in
LH <3U/L (3.0–12.0U/L, follicular)
practice, as in this case, discontinuation of
causative drugs is not always possible, thus MR
Prolactin 6624mU/L (<450mU/L)
imaging of the pituitary to identify a pituitary
TSH 1.9mU/L (0.2–6.0mU/L) adenoma is an acceptable alternative.

93 | Dr. Khalid Yusuf (FB: Sohag Endocrine Group)


SCE Endocrinology (sce.practicaldiabetes.com), 2017

Selective serotonin re-uptake inhibitors are TSH 1.59mU/L


less often associated with hyperprolactinaemia
than other typical and atypical antipsychotic Free T4 15.3pmol/L
medications. There may be considerable
overlap between phenotypic appearances of Urine ACR <0.3mg/mmol
patients with polycystic ovary syndrome
(PCOS) and congenital adrenal hyperplasia He reports diminishing awareness of
(CAH), including hyperandrogenism, hypoglycaemia, and has recorded capillary
subfertility and polycystic ovaries on pelvic blood glucose readings as low as 2.1mmol/L
ultrasound, thus this investigation will not with no symptoms.
differentiate the two conditions. Mild What would you advise?
hyperprolactinaemia may be seen with PCOS,
but is not a feature of non-classic CAH. 1- Commence on theophylline to enhance
Treatment is not recommended for subjective symptoms of hypoglycaemia
asymptomatic patients with non-classic CAH. 2- Measurement of capillary blood glucose
at 2-hourly intervals throughout the day
Further reading and at night
Speiser PW, et al.; Endocrine Society. Congenital
adrenal hyperplasia due to steroid 21- 3- Use of a continuous blood glucose
hydroxylase deficiency: An Endocrine Society monitoring device with integrated alarm
Clinical Practice Guideline. J Clin Endocrinol 4- Regular carbohydrate snacks at intervals
Metab 2010;95(9):4133–60. throughout the day
Melmed S, et al. Diagnosis and treatment of
hyperprolactinemia: An Endocrine Society 5- Reduction in insulin doses to achieve
Clinical Practice Guideline. J Clin Endocrinol higher target blood glucose levels with
Metab 2011;96(2):273–88. avoidance of hypoglycaemia

Q 139 Answer & Comments

5- Reduction in insulin doses to achieve


A 62-year-old man with type 1 diabetes of 43
higher target blood glucose levels with
years’ duration attends the outpatient clinic.
avoidance of hypoglycaemia
He has a history of hypertension,
hypercholesterolaemia, background diabetic Hypoglycaemia unawareness is more common
retinopathy, age-related macular degeneration in patients with intensively controlled diabetes
and a quiescent Charcot’s joint affecting the of long duration, but can occur in any person
left ankle. He manages his diabetes with a basal with diabetes as a consequence of diminished
bolus regimen and is a non-driver. His counter-regulatory responses to recurrent
investigations are as follows: hypoglycaemia.
BMI 24kg/m2 Appropriate management includes
identification of the cause through thorough
HbA1c 43mmol/mol (6.1%)
history, review of medication use and
examination of injection sites. Individualised
Cholesterol (total) 3.9mmol/L
educational support may then be offered with
HDL 2.2mmol/L the intention of achieving adequate glycaemic
control without hypoglycaemia. This is likely to
Creatinine 74mmol/L require reduction in insulin doses in the short
term, then further support with carbohydrate
eGFR 93ml/min/1.73m2 counting, monitoring and titration of insulin
doses. Increasing carbohydrate intake may

94 | Dr. Khalid Yusuf (FB: Sohag Endocrine Group)


SCE Endocrinology (sce.practicaldiabetes.com), 2017

cause unwanted weight gain. Intensive glucose 5- Encourage intake of additional


monitoring with or without a continuous carbohydrate-containing snacks on the
glucose monitoring device may be necessary in days he attends for dialysis
patients with recurrent or disabling
hypoglycaemia despite these interventions to Answer & Comments
aid identification and avoidance of
hypoglycaemic episodes. Although 1- Reduce dose of gliclazide
theophylline and other stimulants have been
shown to improve counter-regulatory Good glycaemic control has been shown to
responses and perception of hypoglycaemia in have long-term benefits in terms of reducing
experimental settings, they are not the incidence of micro- and macrovascular
recommended for routine clinical use. complications in individuals with diabetes.

Further reading Poor glycaemic control (HbA1c >64mmol/mol


Cranston I, et al. Restoration of hypoglycaemia or 8%) is associated with decreased survival in
awareness in patients with long-duration patients with diabetes receiving renal
insulin-dependent diabetes. Lancet replacement therapy in the form of
1994;344:283–7. haemodialysis.
De Galan BE, et al. Theophylline improves
hypoglycemia unawareness in type 1 diabetes. However, there is some evidence that tight
Diabetes 2002;51:790–6. glycaemic control is also associated with
increased mortality, possibly due to increased
Q 140 frequency of hypoglycaemia. Another potential
explanation is that a low HbA1c level is a
surrogate marker of protein-energy wasting,
You are asked to review a 70-year-old man
which is a well-known predictor of mortality in
receiving haemodialysis 3 times weekly for
maintenance haemodialysis.
end-stage renal failure due to diabetic
nephropathy. He has a history of hypertension
In this case, the patient’s hypoglycaemia
and myocardial infarction. His current
unawareness is consistent with frequent
medication includes gliclazide 80mg twice
hypoglycaemic episodes and thus his
daily, doxazosin 4mg once daily, 1-alfacalcidol
medication should be reviewed in order to
500ng once daily, aspirin 75mg once daily and
eliminate hypoglycaemic episodes. This may
bisoprolol 5mg twice daily. His HbA1c is
require a reduction in gliclazide dose, or a
51mmol/mol (6.8%) and he was noted to have
substitution of insulin for gliclazide rather than
a capillary blood glucose reading of 3mmol/L
use of additional therapeutic agents.
prior to commencing on dialysis. He is
asymptomatic.
Further reading
Which of the following interventions is most Ricks J, et al. Glycemic control and cardiovascular
appropriate? mortality in hemodialysis patients with
diabetes. A 6-year cohort study. Diabetes
1- Reduce dose of gliclazide 2012;61:708–15. Published online 2012
February 13. doi: 10.2337/db11-1015.
2- Increase dose of gliclazide aiming for an
HbA1c of 48mmol/mol (6.5%)
Q 141
3- Add linagliptin to his current regimen
aiming for an HbA1c of 48mmol/mol
A 48-year-old man with longstanding,
(6.5%)
moderately well-controlled type 1 diabetes
4- Add insulin to his current regimen attends the outpatient clinic. He has a previous
aiming for an HbA1c of 48mmol/mol medical history of retinopathy, peripheral
(6.5%) neuropathy, chronic kidney disease stage 3,

95 | Dr. Khalid Yusuf (FB: Sohag Endocrine Group)


SCE Endocrinology (sce.practicaldiabetes.com), 2017

hypertension, hyperlipidaemia, and ischaemic infection, which may lead to further


heart disease. He uses a basal bolus insulin complications such as ulceration,
regimen and takes antiplatelet therapy, a osteomyelitis, and amputation. The cause is
statin, iron supplements and a proton pump unknown, although as they tend to occur on
inhibitor, and has no known allergies. He the acral surfaces a number of theories are
complains of multiple, painless blisters postulated including trauma and
appearing on his lower shins with no history of microangiopathy. Correct management is to
trauma, some of which have healed but later leave the blister intact to reduce the risk of
recurred in the same place. He has not noted secondary infection.
blistering lesions elsewhere. On examination,
you note a number of tense, non-tender Topical and systemic steroids with other
blisters of varying sizes on the anterior aspect immunosuppressant agents are used in the
of his lower legs. The blisters contain clear fluid management of bullous pemphigoid and
and the surrounding skin is non-erythematous. epidermolysis bullosa acquisita. Porphyria
cutanea tarda may be treated with
You arrange for him to be reviewed by a
hydroxychloroquine and avoidance of
dermatologist, but what management would precipitating factors such as sun exposure,
you advise in the interim? alcohol, oestrogens and drugs.
1- Avoid sun exposure and consider
therapy with hydroxychloroquine after Further reading
appropriate investigation Poh-Fitzpatrick MB. Bullous disease of diabetes. e-
Medicine. Available from:
2- Leave blisters intact and advise patient www.emedicine.com/derm/topic62.htm.
to seek advice if fluid becomes
discoloured or cloudy
Q 142
3- Recommend drainage of blisters and
cover with a soft dressing to prevent A 27-year-old woman was diagnosed with
further trauma Cushing’s syndrome during pregnancy at 26
4- Treat with topical steroids weeks’ gestation. She was commenced on
metyrapone as an interim measure pending
5- Treat with systemic steroid therapy definitive treatment.
Metyrapone acts by inhibition of which
Answer & Comments
enzyme?
2- Leave blisters intact and advise patient 1- 11 ?-hydroxylase
to seek advice if fluid becomes
discoloured or cloudy 2- 3 ?-hydroxysteroid dehydrogenase
3- 21-hydroxylase
Bullous disease of diabetes or ‘bullosis
diabeticorum’ describes the occurrence of 4- 17 ?-hydroxylase
recurrent, dermal blisters containing sterile 5- 17,20-desmolase
proteinaceous fluid with no inflammatory
component, in patients with diabetes. It is rare,
Answer & Comments
with a prevalence of 0.5%, and is twice as
common in males as in females. It tends to 1- 11 ?-hydroxylase
occur in patients with longstanding diabetes
and multiple complications, many of whom will Metyrapone can be used as short-term
have neuropathy and nephropathy. Blisters treatment for Cushing’s syndrome before
usually heal spontaneously after 2–6 weeks, definitive treatment. Metyrapone blocks
although they often recur in the same place cortisol synthesis by inhibiting 11 β-
and can be complicated by secondary

96 | Dr. Khalid Yusuf (FB: Sohag Endocrine Group)


SCE Endocrinology (sce.practicaldiabetes.com), 2017

hydroxylase. This blockade can be measured by 5- Allow patient to decide whether to


the urinary increase of the metabolites of continue on treatment
cortisol precursors in the urine (17-
hydroxycorticosteroids [17-OHCS] and 17- Answer & Comments
ketogenic steroids [17-KGS]).
3- Stop rhGH therapy for 2–3 months and
Further reading undertake biochemical assessment of
Engelhardt D, Weber MM. Therapy of Cushing’s growth hormone secretion
syndrome with steroid biosynthesis inhibitors.
J Steroid Biochem Mol Biol 1994;49:261–7. Most individuals with idiopathic, isolated
Gormley MJ, et al. Cushing’s syndrome in growth hormone deficiency in childhood have
pregnancy – treatment with metyrapone. Clin normal growth hormone secretion during late
Endocrinol (Oxf) 1982;16:283–93. adolescence or young adulthood, presumably
because of the stimulatory effects of gonadal
Q 143 steroid hormones on the hypothalamic-
pituitary axis for growth hormone secretion. At
A 19-year-old male was reviewed in the completion of linear growth (growth rate
endocrine clinic. He has been treated with <2cm/year), growth hormone replacement
recombinant human growth hormone (rhGH) therapy can be safely discontinued if
therapy since childhood for isolated growth biochemical assessment demonstrates normal
hormone (GH) deficiency. He has achieved his growth hormone secretion. However, if severe
final height and has normal secondary sexual growth hormone deficiency persists,
characteristics. Investigations reveal: continuation of growth hormone therapy at
adult doses may be considered until adult peak
IGF-1 40.2nmol/L (28–50) bone mass is achieved, usually around 25 years
of age. Continuation beyond this point is only
Testosterone 18nmol/L (9–30) recommended if the individual has ongoing
severe growth hormone deficiency and a
LH 1.6mU/L (2–10) perceived impairment of quality of life which
improves with rhGH treatment.
FSH 2.1mU/L (2–10)
Further reading
Free T4 15.2pmol/L (9–22) National Institute for Health and Clinical
Excellence. Human growth hormone in adults
TSH 1.8mU/L (0.5–5.0) with growth hormone deficiency. Clinical
Guideline TA64. 2003. www.nice.org.uk/TA64.
Prolactin 400mU/L (50–450)
What is the next most appropriate step in this Q 144
patient’s management?
An 89-year-old woman with no significant
1- Continue lifelong rhGH treatment
previous medical history was referred to the
2- Continue rhGH treatment only if acute medical assessment unit with sudden-
receiving replacement therapy for onset severe headache. According to her
deficiency of other pituitary hormones granddaughter, she had experienced milder
3- Stop rhGH therapy for 2–3 months and headaches, nausea, malaise and lethargy in the
undertake biochemical assessment of 2 weeks preceding admission. On examination,
growth hormone secretion she was too drowsy to assess visual fields or
acuity, but she had a complete right ptosis with
4- Continue rhGH until adult peak bone dilation of the pupil and depression and
mass achieved and then review abduction of the globe. She had no other focal

97 | Dr. Khalid Yusuf (FB: Sohag Endocrine Group)


SCE Endocrinology (sce.practicaldiabetes.com), 2017

neurological deficit and her vital signs were for the management of pituitary apoplexy. Clin
stable. A CT brain was performed and a Endocrinol (Oxf) 2011;74:9–20.
possible mass lesion or aneurysm was noted in
the region of the optic chiasm, but the scan was Q 145
curtailed due to the patient becoming
hypotensive and unwell. A 73-year-old man with type 2 diabetes is
What is the next most important step in this brought to the accident and emergency
patient’s management? department having been involved in a road
traffic accident during which he collided with
1- Assess fluid and electrolyte balance and the central reservation of a dual carriageway.
give intravenous hydrocortisone He sustained no significant injury except mild
2- Discuss with local neurosurgical unit concussion, but the paramedics recorded his
with a view to arranging urgent transfer capillary blood glucose as 1.6mmol/L at the
for further imaging and intervention scene of the accident. He denies any previous
episodes of hypoglycaemia and remains
3- Draw blood for prolactin, FSH, LH, asymptomatic while an inpatient. He continues
oestradiol, TSH, Free T4, random cortisol on his usual dose of metformin 850mg tds and
and IGF-1 premixed, biphasic analogue insulin 83 units
4- Urgent MRI brain and pituitary with with breakfast and 78 units with his evening
contrast enhancement meal. His capillary blood glucose monitoring is
as follows:
5- Urgent neuro-ophthalmology
assessment including visual field testing Before breakfast

Answer & Comments 7.8mmol/L

1- Assess fluid and electrolyte balance and 8.1mmol/L


give intravenous hydrocortisone
7.7mmol/L
This patient has pituitary apoplexy with an
underlying non-functioning pituitary adenoma. 6.1mmol/L
A diagnosis of pituitary apoplexy should be
considered in all patients who present with Before evening meal
acute onset severe headache, with or without
neuro-ophthalmic signs. Indications for 3.2mmol/L
empirical steroid therapy in patients with
pituitary apoplexy are haemodynamic 4.1mmol/L
instability, altered consciousness level,
2.9mmol/L
reduced visual acuity and severe visual field
defects. Prompt administration of
2.8mmol/L
glucocorticoids may be life-saving, and should
be initiated pending results of pituitary What is the next most important step?
investigations. Further imaging to confirm the
1- Advise the patient he must refrain from
diagnosis should be performed prior to
driving and inform the DVLA
transfer to a neurosurgical unit for
consideration of surgery. 2- Ask the diabetes specialist nurses to
review injection sites and technique
Further reading 3- Increase capillary glucose monitoring to
Rajasekaran S, et al.; Pituitary Apoplexy 4 times daily
Guidelines Development Group. UK guidelines
4- Increase evening insulin by 2 units

98 | Dr. Khalid Yusuf (FB: Sohag Endocrine Group)


SCE Endocrinology (sce.practicaldiabetes.com), 2017

5- Stop metformin and review capillary 2- Perform an OGTT 6–8 weeks after the
blood glucose subsequently acute episode
3- Request a fasting blood glucose no
Answer & Comments sooner than 4 days after the acute
episode and check HbA1c before
1- Advise the patient he must refrain from discharge
driving and inform the DVLA
4- Request an HbA1c before discharge and
This man has documented hypoglycaemia a fasting blood glucose 6–8 weeks after
(capillary blood glucose <4mmol/L) but does the acute episode
not report symptoms, thus he has at least 5- Request an HbA1c before discharge and
diminished awareness of hypoglycaemia. If a an OGTT 6–8 weeks after the acute
lack of subjective awareness with capillary episode
blood glucose <3mmol/L is characteristic of the
patient’s hypoglycaemia experience, he may
be defined as having complete hypoglycaemia Answer & Comments
unawareness, which will require revocation of
3- Request a fasting blood glucose no
his driving licence by the DVLA. It would be
sooner than 4 days after the acute
more appropriate to reduce rather than
episode and check HbA1c before
increase his insulin doses or discontinue
discharge
metformin; and, although review of his
injection sites and technique is appropriate Hyperglycaemia is common in people admitted
and important in establishing the cause of to hospital with acute coronary syndrome
hypoglycaemia, the occurrence of severe (ACS) and is a powerful predictor of poorer
hypoglycaemia and hypoglycaemia survival and increased risk of complications
unawareness while driving poses a significant while in hospital, regardless of whether or not
risk to the patient and other road users, and the patient has diabetes. NICE guidance
needs to be immediately addressed. recommends that all patients with
hyperglycaemia after ACS and without a known
Further reading diagnosis of diabetes should be offered
Gallen I, et al. Driving and hypoglycaemia: measurement of HbA1c before discharge and
questions and answers. Pract Diabetes
measurement of fasting blood glucose no
2012;29:13–4.
earlier than 4 days after the acute episode,
although these tests should not delay
Q 146 discharge. Oral glucose tolerance tests should
not be routinely offered to patients with
A 62-year-old man with no previous history of hyperglycaemia and ACS if HbA1c and fasting
diabetes was noted to have blood glucose glucose levels are within the normal range.
levels between 14 and 20mmol/L following an
inpatient admission for a non-ST elevation Further reading
myocardial infarction. His hyperglycaemia was National Institute for Health and Care Excellence.
initially managed using an intravenous insulin Hyperglycaemia in acute coronary syndromes.
infusion and after discontinuation his capillary Clinical Guideline 130. October 2011.
blood glucose levels were normal. http://guidance.nice.org.uk/CG130.

What advice would you give regarding long-


Q 147
term management?
1- Perform an oral glucose tolerance test A 55-year-old man with type 2 diabetes
(OGTT) before discharge mellitus, dyslipidaemia, obesity and
obstructive sleep apnoea complains of erectile

99 | Dr. Khalid Yusuf (FB: Sohag Endocrine Group)


SCE Endocrinology (sce.practicaldiabetes.com), 2017

dysfunction when he attends outpatient small, firm, non-tender nodule in the right lobe
review. This is characterised by low libido, loss of his thyroid with no associated
of early morning erections and poor erections lymphadenopathy.
with stimulation. His current medication
Which of the following statements is correct?
includes metformin 1g bd, sitagliptin 100mg
od, simvastatin 40mg nocte, co-codamol as 1- A ‘hot’ nodule on radioiodine scan
required, and overnight continuous positive makes malignancy less likely
airway pressure (CPAP) ventilation.
2- Medullary thyroid cancer is the most
Which of the following factors is not likely to likely cause
contribute to secondary hypogonadism in this 3- Papillary thyroid cancer has the worst
case? prognosis
1- Insulin resistance 4- Thyroid cancer is more common in men
2- Amyloid deposition in the pituitary than in women

3- Aromatase-induced oestrogen 5- Thyroglobulin levels are a specific


production marker of follicular cell carcinoma

4- Obstructive sleep apnoea


Answer & Comments
5- Increased inflammatory response
1- A ‘hot’ nodule on radioiodine scan
makes malignancy less likely
Answer & Comments

2- Amyloid deposition in the pituitary Thyroid nodules are common, and most are
non-malignant. Suppressed TSH and nodules
Type 2 diabetes and obesity are associated with increased uptake on radioiodine imaging
with secondary (hypogonadotropic) (i.e. ‘hot nodules’) are typically benign,
hypogonadism in males due to a number of although up to 4% of these may harbour
factors inhibiting the hypothalamic-pituitary- malignancy.
gonadal axis including: insulin resistance and
hyperinsulinaemia; increased aromatisation of Thyroid cancer is 3 times more common in
androgens to oestrogens by adipose tissue; women than men, and papillary carcinoma
subclinical inflammation and increased accounts for most cases. Anaplastic thyroid
inflammatory response; and sleep disorders carcinoma has the worst prognosis, followed
such as obstructive sleep apnoea which by Hürthle cell carcinoma (an aggressive form
increase insulin resistance and may also have of follicular carcinoma) and medullary thyroid
an effect on leptin production. carcinoma. Papillary and follicular carcinomas
have well-differentiated neoplastic cells which
Further reading are usually TSH sensitive, take up iodine and
Aftab ASA, et al. The role of obesity and type 2 produce thyroglobulin, features which may be
diabetes mellitus in the development of male exploited diagnostically and therapeutically.
obesity-associated secondary hypogonadism.
Clin Endocrinol (Oxf) 2013;78:330–7. Further reading
British Thyroid Association, Royal College of
Physicians. Guidelines for the management of
Q 148
thyroid cancer, 2nd edn. (Perros P, ed). Report
of the Thyroid Cancer Guidelines Update
A 72-year-old man with neck swelling and Group. London: Royal College of Physicians,
weight loss is referred by his GP. He has no 2007.
previous medical or family history and appears
clinically euthyroid. On examination, he has a

100 | Dr. Khalid Yusuf (FB: Sohag Endocrine Group)


SCE Endocrinology (sce.practicaldiabetes.com), 2017

Q 149 expression of aldosterone synthase and novel


steroids in the adrenal zona fasciculata under
An 18-year-old male with a family history of the regulation of adrenocorticotropic hormone
haemorrhagic strokes and early onset (ACTH). Treatment with physiologic doses of a
hypertension is found to have a blood pressure glucocorticoid will correct the overproduction
of 168/94mmHg. The results of biochemical of aldosterone by suppressing ACTH.
tests are as below:
Further reading
Sodium 142mmol/L Funder JW, et al; Endocrine Society. Case
detection, diagnosis, and treatment of
Potassium 4.0mmol/L patients with primary aldosteronism: An
Endocrine Society Clinical Practice Guideline. J
Urea 6.5mmol/L Clin Endocrinol Metab 2008;93:3266–81.

Creatinine 90mmol/L Q 150

Plasma renin activity 0.3nmol/L (0.5–2.2)


A 39-year-old woman is referred for advice
regarding management of vasomotor
Aldosterone 782pmol/L (100–500)
symptoms, excessive sweating and mood
Detailed work up confirmed glucocorticoid- swings following a hysterectomy and bilateral
remediable aldosteronism (GRA). oophorectomy for severe endometriosis. She
does not take any regular medication and has
What is the mode of inheritance of this no previous medical history of venous
condition? thromboembolism or breast cancer. She
1- Autosomal dominant enquires about the use of hormone
replacement therapy (HRT) and the potential
2- Autosomal recessive side effects and complications associated with
3- X-linked dominant it.

4- X-linked recessive Which of the following statements is correct?

5- Mitochondrial inheritance 1- A combination of oestrogen and


progesterone is preferable
Answer & Comments 2- The use of oestrogen alone is less likely
to increase the risk of breast cancer
1- Autosomal dominant compared to a combined oestrogen and
progesterone preparation
Glucocorticoid-remediable aldosteronism
(GRA) is a rare, autosomal dominantly 3- Selective oestrogen receptor modulator
inherited condition and a family history of early (SERM) medications are breast cancer
hypertension and haemorrhagic strokes is risk neutral
characteristic. The plasma potassium 4- Use of tibolone may reduce the
concentration is normal in more than one-half incidence of breast cancer
of cases of GRA in contrast to the hypokalaemia
frequently seen in primary aldosteronism. 5- Use of HRT increases the risk of
Individuals have a chimeric gene which results ischaemic heart disease and stroke
from fusion of the ACTH-responsive promoter
region of the gene for 11b-hydroxylase Answer & Comments
production (CYP11B1) and the coding
sequences of the aldosterone synthase gene 2- The use of oestrogen alone is less likely
(CYP11B2). This duplication results in ectopic to increase the risk of breast cancer

101 | Dr. Khalid Yusuf (FB: Sohag Endocrine Group)


SCE Endocrinology (sce.practicaldiabetes.com), 2017

compared to a combined oestrogen and peripheral neuropathy, hypertension and


progesterone preparation chronic kidney disease stage 3. She uses a
twice-daily pre-mixed biphasicinsulin regimen
Use of HRT increases the risk of breast cancer with metformin and linagliptin and her most
and combined oestrogen and progesterone recent HbA1c was 62mmol/mol. She also takes
preparations increase breast cancer risk more an antiplatelet agent, 3 antihypertensives, and
than oestrogen-only preparations; the risk statin therapy. On the morning of surgery her
increases with duration of therapy. In view of blood glucose is found to be 19mmol/L. She has
the fact that the patient has had a already taken her metformin,
hysterectomy, she does not need combined antihypertensives and half her usual insulin
HRT as the progesterone element is used to dose.
prevent endometrial hyperplasia and reduce
What is the next most appropriate
the risk of uterine cancer in women with an
intact uterus. management step?
1- Give her the remaining half dose of her
Selective oestrogen receptor modulators usual morning insulin and recheck her
(SERMs) may have a role in young women with blood glucose in 1 hour aiming for blood
an increased risk of breast cancer and reduced glucose of <10mmol/L
bone density at menopause as they produce an
oestrogen-like effect on bone and lipids and 2- Commence her on a variable rate insulin
reduce the risk of breast cancer. However, they infusion (VRII) until after surgery
are associated with an increased risk of stroke, 3- Give her usual dose of linagliptin
uterine cancer and venous thromboembolism,
although second generation agents such as 4- Check for ketones and give 0.1 units/kg
raloxifene have a better safety profile. rapid acting insulin if negative
Tibolone is a synthetic steroid which mimics 5- Cancel surgery and advise team to
the actions of oestrogen, progesterone and rearrange when blood glucose readings
testosterone and may be associated with an are stabilised
increased risk of breast cancer. The association
between HRT, ischaemic heart disease and Answer & Comments
stroke is unclear and studies are ongoing.
4- Check for ketones and give 0.1 units/kg
Further reading rapid acting insulin if negative
Rossouw JE, et al.; Writing Group for the Women’s
Health Initiative Investigators. Risks and In the first instance, patients with
benefits of estrogen plus progestin in healthy hyperglycaemia should have capillary blood
postmenopausal women: principal results ketones or urinary ketones measured. If there
from the Women’s Health Initiative
is evidence of ketosis (blood ketones >3mmol/L
randomized controlled trial. JAMA
or urinary ketones +++) surgery will need to be
2002;288:321–33.
cancelled and intravenous insulin and fluid
Hickey M, et al. Hormone replacement therapy. rehydration given according to diabetic
BMJ 2012;344:44–9.
ketoacidosis guidelines. If blood glucose is
12mmol/L or greater and there is no evidence
of ketosis, patients with type 1 diabetes should
Q 151 be given subcutaneous rapid-acting analogue
insulin assuming that 1 unit will drop blood
A 56-year-old woman is admitted to the glucose by 3–5mmol/L. Recheck the blood
surgical unit for a day case incisional hernia glucose 1 hour later to ensure it is falling. In
repair. She has a history of type 2 diabetes patients with type 2 diabetes give 0.1 units/kg
complicated by stable retinopathy, bilateral of subcutaneous rapid-acting analogue insulin,
and recheck blood glucose 1 hour later to

102 | Dr. Khalid Yusuf (FB: Sohag Endocrine Group)


SCE Endocrinology (sce.practicaldiabetes.com), 2017

ensure it is falling. If surgery cannot be delayed, Urea 11.0mmol/L


then commence on variable rate insulin
infusion. The aim is to maintain capillary blood Creatinine 98?mol/L
glucose in the 6–10mmol/L range where this Which is the next most appropriate step in this
can be achieved safely. A range of 4–12mmol/L
patient’s management?
is acceptable. If additional pre-mixed biphasic
insulin is administered, the patient is at greater 1- Measure C-peptide levels
risk of late hypoglycaemia. Linagliptin is not 2- Perform a short synacthen test with
useful in acute hyperglycaemia and it is usually ACTH measurement
recommended that DPP-IV inhibitors and GLP-
1 analogues are withheld on the day of surgery. 3- Recommend further weight loss with
further reduction in insulin dosage
Further reading 4- Educate patient and spouse regarding
Joint British Diabetes Societies for Inpatient Care management of hypoglycaemia and
Group. Management of adults with diabetes
provide with intramuscular glucagon
undergoing surgery and elective procedures:
injection
improving standards (April 2011).
www.diabetes.org.uk/Documents/Professionals/ 5- Check FSH/LH and testosterone levels
Reports%20and%20statistics/Management% and measure ferritin
20of%20adults%20with%20diabetes%20unde
rgoing%20surgery%20and%20elective%20pro
Answer & Comments
cedures%20-%20improving%20standards.pdf
2- Perform a short synacthen test with
Q 152 ACTH measurement

A 42-year-old male chef with obesity, While weight loss is often desirable in obese
hypertension and type 2 diabetes is admitted patients with type 2 diabetes, spontaneous
to hospital following an episode of severe weight loss with unpredictable and
hypoglycaemia. He has experienced unexplained hypoglycaemia warrants further
increasingly frequent and unpredictable investigation, and in this case the patient was
hypoglycaemic episodes over the last 2 found to have adrenal insufficiency.
months, despite reducing his insulin doses by Suppressed C-peptide levels are indicative of
approximately one-third with the support of exogenous insulin use, but C-peptide levels
the diabetes specialist nurse. On direct may be difficult to interpret due to the
questioning he admits approximately 8kg hyperinsulinaemic state associated with the
weight loss over this period, associated with metabolic syndrome and type 2 diabetes. Low
general malaise and lethargy. He is also ACTH levels should prompt further pituitary
concerned by erectile dysfunction of recent evaluation in view of his erectile dysfunction,
onset. The results of investigations are: but this symptom is likely to be of multi-
factorial aetiology and is best re-evaluated
BMI 38kg/m2 after management of the acute episode.

HbA1c 7.9% Further reading


Oelkers W. Adrenal Insufficiency. N Engl J Med
Hb 11.9g/dl 1996;335:1206–12.

TSH 2.2mU/L Q 153


Sodium 135mmol/L
A 24-year-old woman with type 1 diabetes is
Potassium 4.9mmol/L admitted to a gastroenterology ward for

103 | Dr. Khalid Yusuf (FB: Sohag Endocrine Group)


SCE Endocrinology (sce.practicaldiabetes.com), 2017

investigation and management of chronic Answer & Comments


diarrhoea. She has a history of type 1 diabetes,
depression and previous episodes of self-harm. 4- She should be allowed to self-manage
She reports a number of recent episodes of her diabetes but the blood glucose
hypoglycaemia as low as 2.2mmol/L with readings, dose and time of the insulin
reduced warning symptoms, and has been administered should be clearly recorded
adjusting her insulin according to her on the insulin chart by a member of staff
carbohydrate intake to combat this with a
reduction in frequency of episodes. Where the facility is available, patients should
Investigations showed the following: be allowed to self-manage their diabetes after
agreement between the patient and the
BMI 19kg/m2 responsible nurse on admission, and an
agreement form should be signed by both the
HbA1c 96mmol/L patient and a registered nurse. Diabetes
specialist nurse staffing levels should be
BP 130/86mmHg sufficient to support this intervention as
necessary. Patients may be allowed to self-
TSH 1.6mU/L monitor their blood glucose but should make
the results available to hospital staff, and the
Coeliac screen: negative insulin dose administered by the patient
should be recorded on the prescription chart.
She is keen to continue monitoring her blood The diabetes specialist team should be
glucose and self-adjusting her insulin during involved if there is disagreement about the
her admission. patient’s ability to self-manage or if there are
Which of the following is correct? difficulties with diabetes control, and in this
case the patient is likely to benefit from a
1- She should not be allowed to self- referral to the diabetes team. Poor glycaemic
manage her insulin because of poor control may be due to a diarrhoeal illness, but
glycaemic control and recurrent there may be a number of other causes
hypoglycaemia including autonomic neuropathy as a
2- She should be allowed to self-manage consequence of poor glycaemic control, which
her diabetes, but only after consulting may improve with intervention to normalise
with the diabetes team glycaemia. Previous self-harm and depression
are a caution rather than an absolute
3- Her recurrent hypoglycaemia and poor contraindication to insulin self-administration.
glycaemic control are likely to be due to
diarrhoeal illness, hence she should be
Further reading
allowed to self-manage and consulting Joint British Diabetes Societies for Inpatient Care
the diabetes team is not mandatory Group. Self-management of diabetes in
4- She should be allowed to self-manage hospital (March 2012).
her diabetes but the blood glucose www.diabetes.nhs.uk/document.php?o=3859..
readings, dose and time of the insulin
administered should be clearly recorded Q 154
on the insulin chart by a member of staff
5- She should not be allowed to self- A 42-year-old woman presented to the
manage her diabetes because of the endocrine clinic with an asymptomatic, long-
history of depression and self-harm standing lump on the right side of her neck.
Examination revealed a 2cm firm, thyroid
nodule on the right lobe. Thyroid function tests
are as follows:

104 | Dr. Khalid Yusuf (FB: Sohag Endocrine Group)


SCE Endocrinology (sce.practicaldiabetes.com), 2017

TSH 2.2mU/L (0.5–5) differentiated thyroid cancer with


radiotherapy/chemotherapy for
Free T4 15.1pmol/L (9–21) anaplastic carcinoma, lymphoma or
metastatic tumor.
She undergoes an ultrasound-guided fine
needle aspiration (FNA) of the nodule, and the Further reading
cytology report categorises the specimen as British Thyroid Association, Royal College of
Thy-1. Physicians. Guidelines for the management of
thyroid cancer, 2nd edn. Perros P (ed). Report
What is the appropriate action?
of the Thyroid Cancer Guidelines Update
1- Reassure and discharge patient Group. London: Royal College of Physicians,
2007.
2- Repeat FNA
3- Repeat FNA after 3–6 months Q 155
4- Offer lobectomy
A 45-year-old man is referred to the
5- Offer thyroidectomy
endocrinology clinic with painful
gynaecomastia noted over the last 12 months.
Answer & Comments He is otherwise fit and well and does not take
any regular medication, and examination is
2- Repeat FNA unremarkable apart from bilateral tender
gynaecomastia. He smokes 10 cigarettes per
The Thy classification adopted by the Royal
day and marijuana on a weekly basis. Initial
College of Physicians for thyroid cytology is as
investigations showed:
follows.
Testosterone 12nmol/L (9–42)
o Thy-1. Non-diagnostic or inadequate.
FNA should be repeated. LH 5U/L (3–8)
o Thy-2. Non-neoplastic. Two diagnostic
benign results 3-6 months apart are U&Es: normal
required to exclude neoplasia.
o Thy-3. (i) Follicular lesion. Lobectomy LFTs: normal
advised, with complete thyroidectomy if
histology proves malignant; (ii) Other Adj calcium 2.23mmol/L (2.20–2.60)
suspicious findings. Discuss with
cytopathologist and multidisciplinary TSH 2.4mU/L (0.20–6.0)
team to decide appropriate course of
action. hCG 95U/L (0–5)
o Thy-4. Abnormal, suspicious of
malignancy (suspicious, but not USS testes: no abnormality detected
diagnostic, of papillary, medullary or What is the next most appropriate
anaplastic carcinoma or of lymphoma). investigation?
Surgical intervention indicated for
differentiated tumour. Further 1- Serum prolactin level
treatment dependent upon pathology 2- Pituitary magnetic resonance imaging
report.
3- Serum oestrogen level
o Thy-5. Diagnostic of malignancy
(unequivocal features of papillary, 4- CT thorax, abdomen and pelvis
medullary or anaplastic carcinoma, or of
5- Mammogram
lymphoma or of metastatic tumour).
Surgical intervention indicated for

105 | Dr. Khalid Yusuf (FB: Sohag Endocrine Group)


SCE Endocrinology (sce.practicaldiabetes.com), 2017

Answer & Comments Plasma renin activity 0.2ng/ml/h

4- CT thorax, abdomen and pelvis Aldosterone 800pmol/L

First-line investigations for gynaecomastia Aldosterone:renin ratio 4000


would usually include serum prolactin in
addition to those given above. However, as the His ramipril is substituted for an increased dose
patient has been found to have an elevated of doxazosin 16mg od and he undergoes saline
hCG and a testicular germ cell tumour has been infusion testing. Aldosterone post-infusion:
excluded by a normal testicular ultrasound, the 334pmol/L
next most appropriate investigation would be
a CT thorax/abdomen and pelvis to look for an CT abdomen: 1cm right-sided adrenal
extragonadal germ cell tumour (EGGCT), the adenoma
most common sites being the mediastinum
(50–70%), retroperitoneum (30–40%), the You discuss the findings with the patient who
pineal gland (5%), and the sacrococcygeal area states that he does not wish to pursue surgery.
(<5%). Human chorionic gonadotrophin may What is the next most appropriate
also be secreted by several non-trophoblastic management step?
neoplasms including colorectal, breast, gastric,
bronchogenic, and bladder cancers. 1- Repeat measurement of
aldosterone:renin ratio following
Mammography is recommended if a suspicious discontinuation of ramipril for 2 weeks
breast mass is found on clinical assessment and
2- Refer for adrenal venous sampling
is relatively accurate in distinguishing between
malignant and benign male breast diseases, 3- Refer for consideration of right-sided
although the positive predictive value for adrenalectomy
malignant conditions is low (55%) because of 4- Medical management with
the low prevalence of malignancy in patients spironolactone
presenting with gynaecomastia.
5- Iodocholesterol scintigraphy
Further reading
Johnson RE, et al. Gynaecomastia: Answer & Comments
pathophysiology, evaluation and
management. Mayo Clin Proc 2009;84:1010– 4- Medical management with
15. spironolactone

Q 156 The aldosterone:renin ratio (ARR) for this


patient is suggestive of primary
hyperaldosteronism and failure of suppression
A 35-year-old man presents with hypertension
of aldosterone with intravenous saline infusion
and hypokalaemia. His medications include
is consistent with this diagnosis. As ACE
ramipril 10mg od and doxazosin 4mg od.
inhibitors may lead to an increase in plasma
Investigations showed:
renin activity and reduced aldosterone due to
Sodium 142mmol/L inhibition of the renin-angiotensin-aldosterone
axis, the sensitivity of the elevated ARR is
Potassium 3.2mmol/L increased and thus re-testing off ramipril is not
necessary. As the patient has made an
Urea 2.4mmol/L informed decision to avoid surgery, there is no
justification for invasive investigation or
Creatinine 65?mol/L isotope scans with a view to localising the

106 | Dr. Khalid Yusuf (FB: Sohag Endocrine Group)


SCE Endocrinology (sce.practicaldiabetes.com), 2017

tumour; thus, he should be maintained on


aldosterone agonist therapy.

Further reading
Funder JW, et al. The Endocrine Society’s clinical
guidelines. Case detection, diagnosis, and
treatment of patients with primary
hyperaldosteronism. JCEM 2008;93:3266–81.

107 | Dr. Khalid Yusuf (FB: Sohag Endocrine Group)

You might also like